Mayo Clinic Neurology Board Review - (Section IV Neurologic Intensive Care Disorders)

You might also like

Download as pdf or txt
Download as pdf or txt
You are on page 1of 78

Section

Neurologic Intensive IV
Care Disorders
Copyright © 2021. Oxford University Press, Incorporated. All rights reserved.

Mayo Clinic Neurology Board Review, edited by Kelly D. Flemming, Oxford University Press, Incorporated, 2021. ProQuest Ebook Central, http://ebookcentral.proquest.com/lib/cuhk-ebooks/detail.action?docID=6746419.
Created from cuhk-ebooks on 2023-09-02 09:10:26.
Copyright © 2021. Oxford University Press, Incorporated. All rights reserved.

Mayo Clinic Neurology Board Review, edited by Kelly D. Flemming, Oxford University Press, Incorporated, 2021. ProQuest Ebook Central, http://ebookcentral.proquest.com/lib/cuhk-ebooks/detail.action?docID=6746419.
Created from cuhk-ebooks on 2023-09-02 09:10:26.
Impaired Consciousness and Comaa
55 MAXIMILIANO A. HAWKES, MD; EELCO F. M. WIJDICKS, MD, PHD

Introduction Anatomy of Consciousness

T
he diagnosis of impaired consciousness or coma Arousal is governed primarily by the ascending reticular
requires a rapid but thorough evaluation. Although activating system (ARAS). The ARAS (Figure 55.1) is a col-
impaired consciousness has many causes, clues lection of nuclei that extend throughout the pons and mid-
can be gleaned from history and examination factors to brain tegmentum and whose input is derived mainly from
deduce potential causes. Ancillary tests, including brain somatic and special sensory afferents of the spinal nerves
imaging and laboratory studies, may further aid in the and cranial nerves (CNs). Output of the ARAS is primarily
diagnosis of impaired consciousness. Coma has various to the reticular nucleus of the thalamus via the central teg-
gradations. When all brain and brainstem functions cease mental tract. The ARAS and the reticular nucleus of the
and a known, irreversible brain injury is responsible, the thalamus through its interactions with the cerebral cortex
condition is referred to as brain death. act as a switch and limit information that reaches the corti-
This chapter reviews the anatomy of altered conscious- cal structures. The thalamic reticular nucleus participates
ness and provides historical and examination clues to in cortico-​thalamo-​cortical interactions.
determine its cause. The definition and diagnosis of brain In addition to thalamic output, the ARAS has connec-
death conclude the chapter. tions with the hypothalamus and the descending output
to nuclei involved in promotion of the wakeful state, such
as raphe nuclei and locus ceruleus. These nuclei have dif-
fuse projections throughout the cortex through noradren-
Consciousness ergic and serotonergic synapses that serve to reinforce the
Definition wakefulness.
Any pathologic process that affects these anatomical
Consciousness can be defined practically as the presence structures can result in a disruption of consciousness. For
Copyright © 2021. Oxford University Press, Incorporated. All rights reserved.

of awareness and being awake, or arousal. When conscious, instance, bilateral cerebral hemispheric dysfunction
a person is awake, alert, vigilant, and aware and has resulting from an acute toxicity or acute metabolic derange-
thoughts and intentions. ment will likely result in altered awareness. Unilateral
Awareness and being awake can be separate compo- ­cortical lesions (eg, a stroke) are unlikely to reduce con-
nents. A person can be aware and awake (for instance, the sciousness if the absence of considerable mass effect
reader of this book), not aware and not awake (coma), or results in opposite hemispheric dysfunction because of
awake but not aware (persistent vegetative state). Aware diffuse bilateral projections from the arousal centers.
but not awake occurs when a person dreams. Similarly, bilateral thalamic lesions are typically required

a
Portions previously published in Moore SA, Wijdicks EF. The acutely comatose patient: clinical approach and diagnosis. Semin Neurol.
2013 Apr;33(2):110-​20. Epub 2013 Jul 25 and Wijdicks EF. The bare essentials: coma. Pract Neurol. 2010 Feb;10(1):51–60; used with
permission.
The editors and authors acknowledge the contributions of S. Arthur Moore, MD, to the previous edition of this chapter.
Abbreviations: ARAS, ascending reticular activating system; CN, cranial nerve; FOUR, Full Outline of Unresponsiveness

453

Mayo Clinic Neurology Board Review, edited by Kelly D. Flemming, Oxford University Press, Incorporated, 2021. ProQuest Ebook Central, http://ebookcentral.proquest.com/lib/cuhk-ebooks/detail.action?docID=6746419.
Created from cuhk-ebooks on 2023-09-02 09:10:26.
454 Section IV. Neurologic Intensive Care Disorders

Cerebral cortex

Thalamus

Cerebral
Cerebral aqueduct
cortex Fourth
ventricle

Hypothalamus

Raphe nuclei

Medial nuclei
(magnocellular) D.F.
© MAYO
Lateral nuclei 2014
(parvicellular)

Figure 55.1 Neuroanatomy of Consciousness. The ascending reticular activating system is responsible for arousal and
projects to the thalamus, hypothalamus, raphe nuclei, and locus ceruleus. The cerebral hemispheres interpret content of
awareness.
(From Wijdicks EFM. Catastrophic neurologic disorders in the emergency department. 2nd ed. Oxford [UK]: Oxford University Press;
c2004. Chapter 8, Altered arousal and coma; p. 59–​93; used with permission of Mayo Foundation for Medical Education and Research.)

to affect consciousness. In contrast, a single lesion may disorder of the consciousness. The patient can be briefly
result in disruption of the reticular formation in the mid- aroused by painful stimulation.
brain. Comatose patients cannot voluntarily respond even
Awareness is the more diffuse cerebral process involv- with noxious stimulation. Coma is usually a transient state
ing the bilateral cerebral hemispheres and their interac- from which patients die or their consciousness evolves to
Copyright © 2021. Oxford University Press, Incorporated. All rights reserved.

tions with the thalamus (particularly the reticular nucleus), other levels, such as a vegetative state or minimally con-
hypothalamus, and brainstem. scious state. Persistent vegetative state is uncommon
because many patients improve to a minimally conscious
Disorders of Consciousness state—​a state where most stay. Young patients (age <45
years), however, may improve to a disabled state without
The classification of disorders of consciousness is arbi-
full dependence.
trary. The emergence of functional magnetic resonance
Delirium is a state of fluctuating alterations of con-
imaging for patients with disordered consciousness
sciousness and cognition due to a general medical condi-
has led to new findings and may lead to reclassification.
tion that results in a global disruption in cognition and
Table 55.1 outlines the current spectrum of disorders of
awareness of surroundings. Although many of the same
­consciousness.
conditions that lead to coma can result in delirium, the
The term somnolence, or drowsiness, refers to a mild
most common causes of delirium are acute metabolic
decrease in the level of alertness. A person goes to sleep
derangements, infection, and a certain toxin or withdrawal
without constant stimulation. Cognition is slow, but the
from the toxin. Factors that predispose a patient to devel-
individual can respond appropriately. Stupor is a deeper
opment of delirium include advanced age (>65 years),

Mayo Clinic Neurology Board Review, edited by Kelly D. Flemming, Oxford University Press, Incorporated, 2021. ProQuest Ebook Central, http://ebookcentral.proquest.com/lib/cuhk-ebooks/detail.action?docID=6746419.
Created from cuhk-ebooks on 2023-09-02 09:10:26.
Chapter 55. Impaired Consciousness and Coma 455

Table 55.1 • Disorders of Consciousness Clinical Approach to Coma and Impaired


State Description
Consciousness
Somnolence/​ Mild-​to-​moderate decreased level of
Historical Evaluation
drowsiness alertness and interest in the As with any medical condition, the approach to the patient
surrounding environment
Needs prodding to do a task
in a coma begins with the history. Timely availability of
May have avoidance reactions to family members or other witnesses can be invaluable for
noxious stimuli refining the differential diagnosis (Box 55.1). Questions
Stupor Decreased level of alertness should focus on the circumstances surrounding the event.
Can be aroused with only vigorous For instance, how was the patient found? What were the
stimuli immediate events before the onset of an alteration in con-
Coma Eyes closed sciousness? Was the alteration abrupt, or did it have grad-
No response to noxious stimuli other ual decline? Family members are especially helpful in
than reflexive, such as extensor or detailing the patient’s recent and past medical history and
flexor posturing
medications, all of which could be contributors to the cur-
No perception of external stimuli
rent state.
Minimally Makes eye contact or orients and tracks Other questions include whether the patient had access
conscious state stimuli
to pills or over-​the-​counter drugs or herbs. Has the patient
Abulic, emotionless
May mouth words or have some had a psychiatric consultation or admission and made a
intelligible speech prior suicide attempt? Is there a history of drug or drinking
May withdraw from and fend off habits? The next question should focus on the possibility
noxious stimuli without an of a central nervous system infection. Did the patient use
appreciable emotional reaction
antibiotics for an infection and did rapid onset of fever and
Persistent vegetative Transitioning from coma to headache occur? Was the patient confused while having
state prolonged coma
fever, and did the patient have difficulty getting their
Arousal present, including sleep-​wake
cycles, but no awareness words out?
Eyes may be open, especially during
the day Physical Examination
Roving eye movements
After a history is obtained, further refining of the differen-
Delirium Rapidly fluctuating change in cognition
Agitated and wild behavior
tial diagnosis is accomplished through a physical examina-
(occasionally subdued) tion. The primary goals of the physical examination are to
Commonly attributable to a general gauge the level of consciousness and to identify focal or
medical condition general findings that may help localize a lesion or perhaps
Frequent findings include indicate an underlying cause.
disorientation, misperception of
sensory stimuli, including
Scales can be used to define the level of consciousness,
hallucinations one of which is the Glasgow Coma Scale. This scale is
divided into 3 responses: eye, verbal, and motor. The max-
Locked-​in syndrome Paralysis of extremities and cranial
nerves with retained vertical eye imal total score is 15 (Box 55.2). A second, frequently pre-
Copyright © 2021. Oxford University Press, Incorporated. All rights reserved.

movements ferred coma scale is the Full Outline of Unresponsiveness


No impairment of sensation or (FOUR) score. With a maximum score of 16, the FOUR
consciousness (may fluctuate) score has 4 elements: motor responses, eye responses,
Often due to large pontine lesions that
brainstem reflexes, and respiratory pattern (Figure 55.2).
spare the reticular activating system
These coma assessment tools are generally performed
Psychogenic Considered only after excluding other before a more comprehensive neurologic examination.
unresponsiveness potential causes of unresponsiveness
Forced gaze that changes in direction
Nonepileptic abnormal movements General Observations
Characteristic responses to hand-​drop General observations are part of a comprehensive physical
test or eye opening and fixating with examination of a patient in a coma. For instance, obvious
tickling of nose hairs signs of trauma such as Battle sign (bruising of the mastoid
bone) or raccoon eyes (periorbital ecchymosis) may each
suggest skull base fractures (see Chapter 60, “Traumatic
underlying mild cognitive impairment or dementia, and Brain Injury”). Hypothermia is typically associated with
prolonged hospitalization or a recent surgical procedure alcohol intoxication, an overdose of barbiturate, or
under anesthesia.

Mayo Clinic Neurology Board Review, edited by Kelly D. Flemming, Oxford University Press, Incorporated, 2021. ProQuest Ebook Central, http://ebookcentral.proquest.com/lib/cuhk-ebooks/detail.action?docID=6746419.
Created from cuhk-ebooks on 2023-09-02 09:10:26.
456 Section IV. Neurologic Intensive Care Disorders

Box 55.1 • Classification and Major Causes of Coma

Structural brain injury Cerebellum (with displacement of brainstem)


Hemispheric Cerebellar infarct
Unilateral (with displacement) Cerebellar hematoma
Intraparenchymal hematoma Cerebellar abscess
Middle cerebral artery occlusion Cerebellar glioma
Hemorrhagic contusion
Cerebral abscess Acute metabolic-​endocrine derangement
Brain tumor Hypoglycemia
Bilateral Hyperglycemia (nonketotic hyperosmolar)
Penetrating traumatic brain injury Hyponatremia
Multiple traumatic brain contusions Hypernatremia
Anoxic-​ischemic encephalopathy Addison disease
Aneurysmal subarachnoid hemorrhage Hypercalcemia
Multiple cerebral infarcts Acute hypothyroidism
Bilateral thalamic infarcts Acute panhypopituitarism
Cerebral venous thrombosis Acute uremia
Lymphoma Hyperbilirubinemia
Encephalitis Hypercapnia
Gliomatosis
Diffuse physiologic brain dysfunction
Acute disseminated encephalomyelitis
Generalized tonic-​clonic seizures
Cerebral edema
Poisoning, illicit drug use
Multiple brain metastases
Hypothermia
Acute hydrocephalus
Gas inhalation
Acute leukoencephalopathy
Acute (lethal) catatonia, malignant neuroleptic
Brainstem syndrome
Pontine hemorrhage
Basilar artery occlusion Psychogenic unresponsiveness
Central pontine myelinolysis Hysterical
Brainstem hemorrhagic contusion Malingering

From Wijdicks EFM. The practice of emergency and critical care neurology. 2nd ed. New York (NY): Oxford University Press; c2016.
Chapter 12, Comatose. p. 104-​36; used with permission of Mayo Foundation for Medical Education and Research.
Copyright © 2021. Oxford University Press, Incorporated. All rights reserved.

hypothyroidism. Hyperthermia can occur with cocaine, The Eyes: Pupil Size and Light Response
tricyclic antidepressant, phencyclidine, and salicylate The eyes are first evaluated by lifting the lids and assessing
intoxication. Presence of fever may also suggest an infec- the pupils, both initial size and response to light.
tious source. Dryness of skin indicates barbiturate poison- Midpositioned, fixed pupils indicate disruption of both
ing or use of anticholinergic agents. parasympathetic and sympathetic tone and disruption of
The classic foul breaths should be known and are occa- the efferent pathway of the pupillary light reflex (CN III).
sionally helpful. They are dirty toilet odor with uremia, They can be indicative of severe midbrain dysfunction
fruity sweat odor with ketoacidosis, fishy or musty odor such as from an infarct or transtentorial herniation. Other
with acute hepatic failure, onion odor with paraldehyde, causes such as medications (atropine) or eye drops must
and garlic odor with organophosphates. Myxedema coma also be excluded.
should be suspected for patients who present with hypo- Small or pinpoint pupils unresponsive (or minimally
tension, bradycardia, hyponatremia, hypoglycemia, and responsive) to light correlate with interruption of the sympa-
hypoventilation; a puffy face; and a thickened nose, swol- thetic supply to the eye and lead to unopposed parasympa-
len lips, and enlarged tongue. thetic activation through CN III. The central nervous system

Mayo Clinic Neurology Board Review, edited by Kelly D. Flemming, Oxford University Press, Incorporated, 2021. ProQuest Ebook Central, http://ebookcentral.proquest.com/lib/cuhk-ebooks/detail.action?docID=6746419.
Created from cuhk-ebooks on 2023-09-02 09:10:26.
Chapter 55. Impaired Consciousness and Coma 457

lesion most frequently responsible for this phenomenon is a


Box 55.2 • Components and Scoring of the Glasgow large pontine hemorrhage. However, heroin or other opioids
Coma Scale can cause a similar clinical picture and should be consid-
ered. Lesions in the upper brainstem (above the red nucleus)
Eye response
can cause pinpoint pupils through disruption of the descend-
4 = Eyes open spontaneously ing sympathetic outflow from the hypothalamus.
3 = Eyes open to verbal command When sympathetic activation of the pupils is unop-
2 = Eyes open to pain posed, the result is fixed and dilated pupils. Midsize light-​
1 = No eyes open fixed pupils are due to a midbrain lesion and always
Motor response indicate more severe injury and loss of brainstem function.
6 = Obeys commands Maximally dilated pupils (>8 mm) are due to a lesion of
5 = Has localizing pain CN III nuclei in the mesencephalon or compression of the
4 = Withdrawal from pain peripheral fibers. Poorly reactive pupils can indicate a
3 = Flexion response to pain lesion in either the afferent (CN II) or the efferent (CN III)
pathways and can be differentiated by observing both the
2 = Extension response to pain
direct and the consensual responses in each eye.
1 = No motor response
A unilateral fixed pupil is due to a CN III lesion from
Verbal response
compression of the midbrain, retraction of CN III, or pres-
5 = Oriented sure of the nerve against the clivus due to mass effect.
4 = Confused
3 = Inappropriate words The Eyes: Position and Movement
2 = Incomprehensible sounds Tracking a finger requires a complex neuronal connectiv-
1 = No verbal response ity. Certain situations lead to deviation of the eyes in 1
direction, known as a gaze preference. A horizontal gaze
Data from Teasdale G, Jennett B. Assessment of coma and preference can result from a lesion involving the pons or
impaired consciousness: a practical scale. Lancet. 1974 Jul
13;2(7872):81-​4. the frontal cortex, or it can be a sign of ongoing seizure
activity. With a pontine lesion in the center of the horizon-
tal gaze (aka, paramedian pontine reticular formation), a
Copyright © 2021. Oxford University Press, Incorporated. All rights reserved.

D.F.
© MAYO
2013

Figure 55.2 The Full Outline of Unresponsiveness (FOUR) Score. The FOUR has 4 elements, each with 4 points possible:
eye (E) and motor (M) responses, brainstem reflexes (B), and respiratory pattern (R). Maximum score is 16. This coma
assessment tool is generally performed before a more comprehensive neurologic examination.
(From Wijdicks EFM. The comatose patient. 2nd ed. New York [NY]: Oxford University Press; c2014. Chapter 3, Neurologic examination of
the comatose patient and localization principles; p. 81–110; used with permission of Mayo Foundation for Medical Education and Research.)

Mayo Clinic Neurology Board Review, edited by Kelly D. Flemming, Oxford University Press, Incorporated, 2021. ProQuest Ebook Central, http://ebookcentral.proquest.com/lib/cuhk-ebooks/detail.action?docID=6746419.
Created from cuhk-ebooks on 2023-09-02 09:10:26.
458 Section IV. Neurologic Intensive Care Disorders

patient will have contralateral gaze deviation, looking Other CN reflexes that can be tested as part of the coma
toward the side of a hemiparesis if the lesion also includes examination and their clinical implications are listed in
the descending corticospinal tract. (See Chapter 11, Table 55.2 and Figure 55.3.
“Brainstem and Cranial Nerves: Longitudinal Pathways of
the Brainstem,” Figure 11.3.) The oculocephalic maneuver The Motor Examination
(discussed below) does not overcome this gaze preference. For a comatose patient, motor responses are typically eval-
With a lesion involving the frontal cortex (specifically, the uated by the application of painful stimulus to the supraor-
frontal eye fields), patients look away from the hemiparesis bital nerve, the temporomandibular joint, or the nail bed.
if the corticospinal tract is involved. The oculocephalic Motor responses are generally described as localizing or
maneuver is able to overcome a gaze preference due to a reflexive.
lesion in the frontal cortex because the pons and midbrain A localizing response is one in which the patient
structures responsible for the reflex remain intact. reaches toward the noxious stimulus (ie, the patient local-
The oculocephalic, or vestibulo-​ ocular, reflex (doll’s izes to the stimulus). For instance, when pressure is
eyes) can be elicited to assess the integrity of multiple applied to the supraorbital nerve, the patient’s hand may
brainstem structures (see Chapter 11, “Brainstem and reach toward the stimulated eye. Reflexive responses
Cranial Nerves: Longitudinal Pathways of the Brainstem”). include extensor or flexor posturing or a lack of response.
The maneuver itself involves rotating the head horizon- Decorticate posturing and decerebrate posturing are
tally or vertically while observing the eyes for movement. reflexive motor responses (Figure 55.4). Decorticate pos-
Because of the head movement involved, a cervical spine turing is characterized by slow flexion at the elbow, wrist,
injury should be conclusively ruled out before this maneu- and fingers. Conventional wisdom is that decorticate pos-
ver is attempted. In a patient with a normally functioning turing results from a lesion above the red nucleus because
brainstem, eyes move in an opposite direction to the head cortical influence has been interrupted but the rubrospi-
movement and appear to remain fixated on a point in space nal tracts, pontine reticulospinal tracts, and medullary
(much like a doll’s eyes, for which the reflex is named). If vestibulospinal tracts stay intact. However, many coma-
a lesion exists, the eyes will move along with the head, tose patients have both decorticate and decerebrate pos-
remaining in midposition with respect to the bony orbits. turings simultaneously, and these motor responses can be
A normal oculocephalic reflex implies intact function nonlocalizing.
within the pontine and midbrain pathways involving CN Decerebrate posturing is characterized by extension of
III and CN IV (horizontal movements), CN III and CN VI the upper extremities at the elbow, wrist, and fingers,
(vertical movements), and the medial longitudinal fascicu- accompanied by pronation. In decorticate and decerebrate
lus connecting CN III with CN VI. posturing, the lower extremities extend.
Caloric testing similarly tests the integrity of multiple
brainstem nuclei, including the vestibular nuclei. The test
is performed by first raising the head of the bed to 30° and
then instilling cold water in 1 ear (ensure the eardrum is
intact first). If the brainstem is intact, the current within
the semicircular canals through CN VIII results in tonic Table 55.2 • Cranial Nerve (CN) Reflexes
deviation of the eyes toward the ear with the cold water Afferent Central Lesiona
through pathways involving CN III, CN VI, and the medial Reflex Limb Efferent Limb Location
Copyright © 2021. Oxford University Press, Incorporated. All rights reserved.

longitudinal fasciculus.
Pupillary light CN II CN III Upper
Spontaneous eye movements, rather than those elicited
midbrain
with testing, occasionally occur in comatose patients.
Corneal CN V CN VII Lower
Roving eye movements are slow, conjugate, lateral excur-
midbrain to
sions of the eyes that can be seen in normal sleep or upper pons
patients with diffuse cortical dysfunction due to toxic or
Oculocephalic CN VIII CN III, IV, VI Upper
metabolic threats. Although not etiologically diagnostic, midbrain to
the presence of roving eye movements signifies structural lower pons
integrity of the brainstem.
Gag CN IX CN X Medulla
Ocular bobbing, which involves a rapid downward eye
movement followed by slow upward deviation, indicates a Cough CN X CN X Medulla to
Cervical spinal cervical
pontine lesion. The opposite—​ ocular dipping—​ can be
nerves 3, 4, spinal cord
present in varied brainstem lesions without a clear local- and 5
izing value. Skew deviation, in which the eyes are no lon- a
Lesions are those within the central nervous system that can produce
ger conjugate, suggests an acute brainstem injury, typically an absent reflex. Peripheral lesions involving the individual CN can
to CN III and CN IV in the midbrain. lead to an absent reflex, but these are not discussed herein.

Mayo Clinic Neurology Board Review, edited by Kelly D. Flemming, Oxford University Press, Incorporated, 2021. ProQuest Ebook Central, http://ebookcentral.proquest.com/lib/cuhk-ebooks/detail.action?docID=6746419.
Created from cuhk-ebooks on 2023-09-02 09:10:26.
Chapter 55. Impaired Consciousness and Coma 459

II
A
III

VII

III
B
VI
E
VIII

IX
C X

V
Copyright © 2021. Oxford University Press, Incorporated. All rights reserved.

VII

D.F.
© MAYO
2010

Figure 55.3 Cranial Nerve Reflexes. A, Pupillary light reflex. B, Vestibular response to water. C, Grimace response to pain.
D, Corneal response. E, Gag reflex. The solid line represents the cranial nerve involved in the afferent pathway. The dotted
line represents the cranial nerve involved in the efferent pathway. (See text and Table 55.2.)
(From Wijdicks EFM. Brain death. 3rd ed. New York [NY]: Oxford University Press; c2017. Chapter 2, Neurology of brain death; p. 25-​73;
used with permission of Mayo Foundation for Medical Education and Research.)

Mayo Clinic Neurology Board Review, edited by Kelly D. Flemming, Oxford University Press, Incorporated, 2021. ProQuest Ebook Central, http://ebookcentral.proquest.com/lib/cuhk-ebooks/detail.action?docID=6746419.
Created from cuhk-ebooks on 2023-09-02 09:10:26.
460 Section IV. Neurologic Intensive Care Disorders

B
A

D.F.
© MAYO
D.F. 2014
© MAYO
2014

Decorticate posturing Decerebrate posturing (lesion between


red nucleus and vestibular nuclei)
Figure 55.4 Decorticate (A) and Decerebrate (B) Posturing.
(From Flemming KD. Principles of critical care neurology. In: Mowzoon N, Flemming KD, editors. Neurology board review: an illustrated
study guide. Rochester [MN]: Mayo Clinic Scientific Press and Florence [KY]: Informa Healthcare USA; c2007. p. 401-​34; used with
permission of Mayo Foundation for Medical Education and Research.)

Respiratory Patterns • Midposition, fixed pupils indicate disruption of


The respiratory pattern of comatose patients is often diffi- parasympathetic and sympathetic tone and disruption
cult to observe because many of these patients require of the efferent pathway of the pupillary light reflex (CN
mechanical ventilation. Even though a spontaneous venti- III) and can be indicative of severe midbrain
lator mode can be used to observe a patient’s intrinsic dysfunction, such as dysfunction from an infarct or
breathing pattern, the localization value of the neurogenic transtentorial herniation.
respiratory patterns is not firmly established. However, the • Small or pinpoint pupils that are unresponsive (or
classic patterns are reviewed below and in Table 55.3. minimally responsive) to light are associated with
Cheyne-​Stokes breathing is a respiratory pattern char- interruption of the sympathetic supply to the eye,
acterized as a progressive increase in respiratory volume leading to unopposed parasympathetic activation
and rate followed by a gradual decrease in volume and rate through CN III.
that may lead to a brief apnea. This pattern can be associ- • Because of the head movement involved in the
ated with any cause of reduced responsiveness and, as oculocephalic maneuver, a cervical spine injury
such, is generally considered to be poorly localizing. By should be conclusively ruled out before it is
comparison, short-​cycle Cheyne-​Stokes breathing may be attempted.
seen among patients whose condition is deteriorating to a • In caloric testing, if the brainstem is intact, the current
lower level of consciousness. within the semicircular canals via CN VIII results in
Apneustic breathing may occur when a lesion is at the tonic deviation of the eyes toward the ear with the
pontine level. It is characterized by gasping inspiration, a cold water through pathways involving CN III, CN VI,
Copyright © 2021. Oxford University Press, Incorporated. All rights reserved.

pause at full inspiration, and then release. Cluster breath- and the medial longitudinal fasciculus.
ing and central neurogenic hyperventilation can each indi- • Decorticate posturing is characterized as slow flexion
cate bihemispheric or pontine lesions. Cluster breathing is at the elbow, wrist, and fingers with lower extremity
characterized by periods of apnea followed by a cluster of extension.
progressively more shallow breaths until another period of • Decerebrate posturing is characterized by extension of
apnea. Central neurogenic hyperventilation is distin- the upper extremities at the elbow, wrist, and fingers,
guished by a considerable increase in respiratory rate with typically accompanied by pronation and lower
relatively low tidal volumes. extremity extension.
Ataxic breathing generally indicates a lesion of the lat-
eral tegmentum within the lower pons. The distinguishing
feature of this respiratory pattern is that no true pattern
exists. Respirations tend to be erratic in timing and vol-
Treatment of Patients With Impaired
ume. Periods of ataxic breaths are often preceded or fol- Consciousness
lowed by brief periods of apnea and commonly deteriorate The first steps in the care of patients with impaired con-
into agonal respirations. sciousness are to stabilize vital signs and determine

Mayo Clinic Neurology Board Review, edited by Kelly D. Flemming, Oxford University Press, Incorporated, 2021. ProQuest Ebook Central, http://ebookcentral.proquest.com/lib/cuhk-ebooks/detail.action?docID=6746419.
Created from cuhk-ebooks on 2023-09-02 09:10:26.
Chapter 55. Impaired Consciousness and Coma 461

Table 55.3 • Breathing Abnormalities of Patients With Coma


Breathing Description of
Abnormality Respiratory Patterns Breathing Localization

Cheyne-​Stokes Crescendo-​decrescendo Bihemispheric


breathing pattern (unilateral or
followed by apnea bilateral) or
D.F. or hypopnea; persists brainstem
© MAYO in sleep
2014
Cheyne-Stokes
Cluster Irregular clusters of Bihemispheric or
breaths followed by pons
apneic periods of
variable duration
D.F.
© MAYO
2014

Cluster (lower pons)


Ataxic or irregular Irregular respiratory rate, Nonlocalizing or
rhythm, and dorsomedial
amplitudes medulla
interrupted by apnea
D.F.
One minute © MAYO
2014

Ataxic
Apneustic Prolonged inspiration Lateral tegmentum
with a 2-​or 3-​second of lower pons
pause, then expiration
D.F.
© MAYO
Apneustic (pons) 2014

Central neurogenic Sustained Bihemispheric,


hyperventilation hyperventilation, pons, midbrain
exceeds respiratory
D.F. rates of 40 breaths per
© MAYO
2014 minute

Central neurogenic hyperventilation


From Wijdicks EFM. The comatose patient. 2nd ed. New York (NY): Oxford University Press; c2014. Chapter 3, Neurologic examination of the comatose
patient and localization principles; p. 81–​110; used with permission of Mayo Foundation for Medical Education and Research.
Copyright © 2021. Oxford University Press, Incorporated. All rights reserved.

Illustrations from Flemming KD. Principles of critical care neurology. In: Mowzoon N, Flemming KD, editors. Neurology board review: an illustrated
study guide. Rochester (MN): Mayo Clinic Scientific Press and Florence (KY): Informa Healthcare USA; c2007. p. 401–​34; used with permission of
Mayo Foundation for Medical Education and Research.

whether the patient needs intubation and blood pressure Emergency computed tomography of the head without
support or treatment. While the patient’s status is stabi- contrast medium also should be obtained, and the cervical
lized, laboratory values are obtained immediately to rule spine should be cleared. If the patient’s history is sugges-
out potentially reversible causes of coma. Testing should tive of meningitis, empirical broad-​ spectrum antibiotic
include a point-​ of-​
care glucose, arterial blood gas, and therapy should be initiated before a lumbar puncture but
ammonia tests and a full electrolyte panel and drug screen. after specimens for peripheral blood cultures have been
Certain emergency medications may reverse coma. obtained. Any clinical suspicion of ongoing seizure activity
These include naloxone, flumazenil, and glucose in 50% should be treated emergently with an intravenous benzodi-
dextrose fluid. The patient without intravenous access can azepine such as intravenous lorazepam. If evidence shows
be given 1 to 2 mg of intramuscular glucagon. increased intracranial pressure, mannitol or hypertonic

Mayo Clinic Neurology Board Review, edited by Kelly D. Flemming, Oxford University Press, Incorporated, 2021. ProQuest Ebook Central, http://ebookcentral.proquest.com/lib/cuhk-ebooks/detail.action?docID=6746419.
Created from cuhk-ebooks on 2023-09-02 09:10:26.
462 Section IV. Neurologic Intensive Care Disorders

Table 55.4 • Clinical Care of Comatose Patients


Assessment Care Process

Airway, breathing, circulation Assess airway


Intubate when deoxygenation, airway obstruction, tachypnea, or apneic
episodes are present
Check oxygen saturation, administer oxygen as needed
Assess blood pressure and pulse
Place cardiac monitor
Immediate laboratory tests Check glucose with finger stick
Assess arterial blood gas
Check electrolytes, complete blood cell count, calcium, magnesium,
ammonia, drug levels, PT, and PTT
Consider toxin screen
Supportive measures Administer thiamine 50-​100 mg IV
Administer naloxone 1 ampule IV
Administer glucose D-​50 IV (≥25 mL)
Examination If signs or symptoms are suggestive of meningitis (altered mental status,
fever, meningismus), consider empirical use of antibiotics and perform
lumbar puncture if safe
Abbreviations: D-​50, dextrose 50% solution; IV, intravenous; PT, prothrombin time; PTT, partial thromboplastin time.
Modified from Flemming KD. Principles of critical care neurology. In: Mowzoon N, Flemming KD, editors. Neurology board review: an
illustrated study guide. Rochester (MN): Mayo Clinic Scientific Press and Florence (KY): Informa Healthcare USA; c2007. p. 401–34;
used with permission of Mayo Foundation for Medical Education and Research.

saline should be administered. Table 55.4 summarizes clin- eliminated or corrected before testing. Severe hypothermia
ical management recommendations. Table 55.5 lists clues and hypotension should be reversed with a core tempera-
to the cause of coma based on the initial history, physical ture of more than 36oC and systolic blood pressure of more
examination, and basic laboratory studies. Table 55.6 out- than 100 mm Hg or a mean arterial pressure of 65 mm Hg.
lines causes of impaired consciousness due to poisoning. Severe metabolic derangements should be corrected with
replacement therapy or removal of the harmful metabolites
• For patients with impaired consciousness, emergency as necessary. Sedating or paralytic medications should be
medications given to potentially reverse coma include out of the patient’s system (typically about 5 half-​lives,
naloxone, flumazenil, and 50% dextrose. although this can be longer for patients with renal or
hepatic failure). Extraneous toxins should be ruled out.
Testing can proceed only after these criteria are met.
The brain death examination begins for a comatose
Brain Death patient with an attempt to elicit a motor response. The
Definition absence of motor responses, except those attributable to
Copyright © 2021. Oxford University Press, Incorporated. All rights reserved.

spinal cord reflexes, is consistent with brain death. Any


Brain death is the irreversible cessation of all brain and
decorticate or decerebrate posturing implies an intact
brainstem function due to a known, irreversible, and
brainstem and excludes brain death. Spinal cord reflexes
mostly supratentorial injury. The American Academy of
can include several movements seemingly inconsistent
Neurology issued a guideline in 2010 that can be
with death, including rapid flexion of the arms, multifo-
followed in patient evaluation. Its principle is to
­
cal myoclonic movements, and even the raising of 1
­determine a cause, exclude confounders, and do a com-
limb or all limbs off the bed and sitting up (the Lazarus
prehensive clinical examination and an apnea test. The
sign). These should not be confused with purposeful
determination of brain death in the pediatric popula-
movements.
tion typically requires 2 examinations with a fixed
A CN examination normally follows evaluation of the
interval between them.
motor responses. Presence of any CN reflexes excludes a
Clinical Examination diagnosis of brain death. Those tested include the pupil-
lary light response, corneal reflexes, the cold-​water caloric
In all cases of brain death, an irreversible cause of brain oculocephalic reflex, the gag reflex, and the cough reflex
dysfunction should be proven and confounding factors with tracheal suctioning.

Mayo Clinic Neurology Board Review, edited by Kelly D. Flemming, Oxford University Press, Incorporated, 2021. ProQuest Ebook Central, http://ebookcentral.proquest.com/lib/cuhk-ebooks/detail.action?docID=6746419.
Created from cuhk-ebooks on 2023-09-02 09:10:26.
Chapter 55. Impaired Consciousness and Coma 463

Table 55.5 • Hints to the Cause of Impaired Table 55.6 • Selected Causes of Impaired Consciousness
Consciousness Based on Initial Clinical Due to Poisoning
Evaluation
Poisoning Type Characteristics
Clinical Characteristic Possible Cause
Organophosphate Mnemonic MUDDLES: miosis,
Battle sign, periorbital Trauma urination, diarrhea, diaphoresis,
ecchymosis lacrimation, excitation, and
salivation
Hyperthermia Cocaine, tricyclic antidepressants,
phencyclidine (PCP) (also called Methanol Can cause blindness, metabolic
angel dust), salicylate acidosis, headache, confusion, and
intoxication coma. Delayed parkinsonism may
develop because of effects on
Hypothermia Cold exposure, alcohol intoxication, putamen
barbiturate overdose
Carbon monoxide Headache, dizziness, seizures,
Metabolic acidosis Lactic acidosis, ketoacidosis, impaired consciousness, heart
organic acid intoxication (eg, arrhythmia
salicylates, ethanol, methanol,
formaldehyde, ethylene glycol, Salicylate (aspirin) Tinnitus, deafness, tachycardia,
paraldehyde), massive nausea, vomiting, sweating,
rhabdomyolysis respiratory alkalosis (early)
followed by metabolic acidosis
Dry skin Barbiturate poisoning,
anticholinergic agents, botulism Acetaminophen Nausea, vomiting, hepatic necrosis,
confusion, encephalopathy
Pinpoint pupils Pontine lesion, opioid toxicity,
organophosphate poisoning Iron Nausea, vomiting, diarrhea, abdominal
pain, jaundice, coma
Ophthalmoplegia Botulism poisoning, Wernicke
encephalopathy Tricyclic Hallucinations, hyperreflexia,
antidepressant myoclonus, seizures, impaired
Dilated pupils Theophylline poisoning, lesion with overdose consciousness, dilated pupils,
rostrocaudal deterioration to hyperpyrexia, dry mouth, flushing,
level of medulla, bilateral third bowel or bladder paralysis,
cranial nerve palsy, serotonergic tachyarrhythmias, conduction
psychedelics (lysergic acid abnormalities
diethylamide [LSD], mescaline),
anticholinergics (atropine,
scopolamine), stimulants
(cocaine, amphetamines)
of attempted respiration. Given that the Paco2 level
increases 3 to 6 mm Hg per minute, an arterial blood gas
test is repeated at 8 minutes. The test is considered posi-
Diagnostic Testing tive if there is no breathing effort when the arterial Pco2 is
more than 60 mm Hg or if there is an increase of 20 mm Hg
The final test to be performed as part of the brain death
in Pco2 above a normal baseline value. When performing
examination is the apnea test (Figure 55.5). Requirements
the apnea test, 1) allow enough time for the Paco2 to
Copyright © 2021. Oxford University Press, Incorporated. All rights reserved.

for the apnea test, in addition to those listed above for the
increase to the appropriate level and 2) remember that
general brain death examination, include a core tempera-
patients with chronic hypercapnia (such as occurs with
ture of more than 36°C, a positive fluid balance and no
chronic obstructive pulmonary disease) have higher Paco2
recent polyuric episode, a Paco2 of more than 40 mm Hg,
levels at baseline, leading to reduced test reliability and
and a Pao2 of more than 200 mm Hg. The arterial gas levels
the need for further confirmatory testing.
are achieved through preoxygenation and alteration of the
Confirmatory testing is not required for adults, but it
set respiratory rate, if needed. Typically, the patient
can be used if the apnea test cannot be completed. Tests
receives preoxygenation with 100% O2 for 10 minutes
include electroencephalography, cerebral angiography,
before testing.
transcranial Doppler, or cerebral nuclear scanning.
A baseline arterial blood gas value is obtained and the
ventilator is turned off. A catheter inserted close to the
• Brain death is the irreversible cessation of all brain
level of the carina continues to supply oxygen at a rate of 6
and brainstem function due to a known, irreversible,
L/​minute. The patient is closely watched for any evidence
and mostly supratentorial injury.

Mayo Clinic Neurology Board Review, edited by Kelly D. Flemming, Oxford University Press, Incorporated, 2021. ProQuest Ebook Central, http://ebookcentral.proquest.com/lib/cuhk-ebooks/detail.action?docID=6746419.
Created from cuhk-ebooks on 2023-09-02 09:10:26.
464 Section IV. Neurologic Intensive Care Disorders
Copyright © 2021. Oxford University Press, Incorporated. All rights reserved.

Figure 55.5 Apnea Test Procedure. ABP indicates arterial blood pressure; BP, blood pressure; Fio2, fraction of inspired
oxygen; HR, heart rate; PEEP, positive end-​expiratory pressure (cm H2O); RESP, respiration; Spo2, oxygen saturation as
measured by pulse oximetry. (See text for further explanation.)
(From Wijdicks EFM. Brain death. 3rd ed. New York [NY]: Oxford University Press; c2017. Chapter 2, Neurology of brain death; p. 25-​73;
used with permission of Mayo Foundation for Medical Education and Research.)

Mayo Clinic Neurology Board Review, edited by Kelly D. Flemming, Oxford University Press, Incorporated, 2021. ProQuest Ebook Central, http://ebookcentral.proquest.com/lib/cuhk-ebooks/detail.action?docID=6746419.
Created from cuhk-ebooks on 2023-09-02 09:10:26.
Chapter 55. Impaired Consciousness and Coma 465

• Severe hypothermia and hypotension should be • When an apnea test is performed to confirm brain
reversed with a core temperature >36°C and systolic death, the arterial blood gas test is considered positive
blood pressure >100 mm Hg. Severe metabolic if no breathing effort occurs when the arterial Pco2 is
derangements should be corrected with replacement or >60 mm Hg or if there is an increase of 20 mm Hg in
toxin removal as necessary; there should be no effect Pco2 above a normal baseline value.
of sedating or paralytic drugs.
Copyright © 2021. Oxford University Press, Incorporated. All rights reserved.

Mayo Clinic Neurology Board Review, edited by Kelly D. Flemming, Oxford University Press, Incorporated, 2021. ProQuest Ebook Central, http://ebookcentral.proquest.com/lib/cuhk-ebooks/detail.action?docID=6746419.
Created from cuhk-ebooks on 2023-09-02 09:10:26.
Principles and Management of Increased
56 Intracranial Pressure
SHIVRAM KUMAR, MBBS; EELCO F. M. WIJDICKS, MD, PHD

Introduction The brain accounts for 80% of the total intracranial vol-
ume. Brain volume can enlarge suddenly from swelling or

I
ntracranial pressure (ICP) is a reflection of the total spontaneous hemorrhage or in a tumor. A rapid increase of
volume inside the skull. Normal ICP is 5 to 15 mm Hg. more than 10% is clinically consequential.
Intracranial hypertension is defined as sustained ICP CSF accounts for 10% of the intracranial volume. The
of more than 20 mm Hg. Increased ICP may lead to a choroid plexus is the main producer of CSF, at a rate of
reduction of cerebral perfusion pressure (CPP), a shift of approximately 500 mL per day. CSF flows through the ven-
brain tissue, and, as a result, secondary brain or brainstem tricular system and is resorbed by the arachnoid granula-
injury. Early recognition and treatment of elevated ICP are tions into the venous system. Overproduction of CSF (eg,
needed to prevent irreversible damage to the brainstem. choroid plexus papilloma), ventricular obstruction, or
This chapter reviews the clinical presentation, diagno- poor resorption may increase the ICP. However, increased
sis, monitoring, and treatment of elevated ICP. intracranial CSF pressure can be displaced to the spinal
compartment, relieving some of its effects.
Physiologic Factors Lastly, blood accounts for another 10% of the intracra-
The Cranial Contents nial volume. Cerebral blood flow (CBF) may increase with
The 3 constituents of intracranial volume are brain (80%), hypoxia or hypercapnia, and thus the intracranial blood
cerebrospinal fluid (CSF) (10%), and blood (10%). These flow can be manipulated by physiologic factors. If ICP
elements sit within the fixed volume of the skull (approxi- reaches the diastolic pressure, intracranial blood flow
mately 1,400-​ 1,700 mL). If one of these 3 constituents stops at the entry of the skull.
increases in volume, the other 2 must compensate by Generally, if 1 of the 3 intracranial components increases
Copyright © 2021. Oxford University Press, Incorporated. All rights reserved.

reducing volume. If they cannot compensate, ICP increases in volume, compensation must occur. CSF is shifted to spi-
and tissue shifts through preexisting (tentorium or foramen nal subarachnoid spaces, and the venous structures col-
magnum) or iatrogenic (eg, craniotomy) openings. These lapse. Additionally, further increase of ICP can lead to
cranial principles are known as the Monro-​Kellie doctrine herniation of brain tissue. These events may be associated
(Figure 56.1). with clinical signs (ie, change in pupil size and light
response and abnormal motor responses).

The editors and authors acknowledge the contributions of Heidi T. Woessner, MD, and William D. Freeman, MD, to the previous edition
of this chapter.
Abbreviations: CBF, cerebral blood flow; CO2, carbon dioxide; CPP, cerebral perfusion pressure; CSF, cerebrospinal fluid; CT, computed
tomography; CVR, cerebrovascular resistance; ICP, intracranial pressure; MAP, mean arterial pressure; MCA, middle cerebral artery

466

Mayo Clinic Neurology Board Review, edited by Kelly D. Flemming, Oxford University Press, Incorporated, 2021. ProQuest Ebook Central, http://ebookcentral.proquest.com/lib/cuhk-ebooks/detail.action?docID=6746419.
Created from cuhk-ebooks on 2023-09-02 09:10:26.
Chapter 56. Principles and Management of Increased Intracranial Pressure 467

decreases below the ischemic threshold, global brain isch-


emia, infarction, and brain death can occur.

The Blood-​Brain Barrier and Types of Edema


Pressure

Capillary endothelial cells and astrocytes form the most


important barrier between brain and blood. This blood-​
brain barrier is composed of high-​resistance junctions that
limit the movement of large and nonsoluble molecules and
charged substances. Water is able to move somewhat freely,
I II III
but it is largely regulated by electrolyte pumps. This bar-
Volume rier may be damaged by certain toxins, radiation, trauma,
and other central nervous system structural pathologic
Figure 56.1 Intracranial Pressure-​Volume Curve. In zone I, ­factors.
compensatory mechanisms are optimal. In zone II, The 3 major fluids of the brain are blood, CSF, and
compensatory mechanisms fail. A slow increase in pressure interstitial fluid. CSF is similar to plasma but has less pro-
(period of spatial compensation) develops. In zone III, tein and fewer cells. The choroid plexus is the main pro-
increased intracranial pressure and herniations occur and ducer of CSF, and capillaries contribute to its production.
are virtually irreversible. The increase (period of spatial Water moves back and forth through osmotic gradients
decompensation) is rapid. among cells, blood, and interstitial space.
(Modified from Wijdicks EFM. The clinical practice of critical Cerebral swelling, or edema, can be categorized into
care neurology. 2nd ed. Oxford [UK]: Oxford University Press; vasogenic, cytotoxic, or interstitial, although the 3 can
c2003. Chapter 9, Intracranial pressure; p. 107-​25; used with overlap.
permission of Mayo Foundation for Medical Education and Vasogenic edema is typically caused by disruption of
Research.) the blood-​brain barrier wherein fluid leaks out of the cap-
illaries into the extracellular space. This fluid contains
blood and protein, which increase the oncotic pressure
and pull water from surrounding areas. This fluid tends to
Relationship of ICP and CPP
CPP is the driving pressure, or forward flow, that allows
CBF. The relationship of CPP to ICP can be assessed with
the following equation.

CPP = MAP − ICP, where MAP is mean arterial pressure

Normal CPP depends mostly on whether the patient


has normal blood pressure or is hypertensive. For exam-
ple, for a patient with a normal blood pressure of 120/​80
mm Hg (MAP, 93 mm Hg) and an ICP of 5 mm Hg, the cal-
culated CPP is 88 mm Hg. Normal CPP ranges from 50 to
Copyright © 2021. Oxford University Press, Incorporated. All rights reserved.

150 mm Hg, with a mean of 80 to 100 mm Hg.


In normal states, CBF is held constant across a range of
CPP values, termed cerebral autoregulation, because of
varying cerebrovascular resistance (CVR) (ie, CBF = CPP/​
CVR). Normal CBF is typically between 50 and 100 mL/​
100 g per minute. For a patient with chronic hypertension,
Figure 56.2 Cerebral Autoregulation. In normal states, CBF
“normal” CPP is shifted rightward depending on the long-​
is held constant over a range of CPP values because of varying
term MAP, although cerebral autoregulation is maintained
cerebrovascular resistance (CVR). In stroke and brain injury,
(Figure 56.2). For patients with cerebral ischemia or trau-
CVR becomes constant (k), and thus CBF varies linearly with
matic brain injury, autoregulation is lost, CVR becomes
CPP values (in a linear relationship). In patients with chronic
constant, and CBF varies linearly with CPP. The ischemic
hypertension, the curve is shifted right. CBF indicates cerebral
threshold is less than 20 mL/​100 g per minute CBF, and
blood flow; CPP, cerebral perfusion pressure.
gray matter typically has greater requirements (a threshold
(Modified from Rose JC, Mayer SA. Optimizing blood pressure in
of about 80-​100 mL/​100 g/​min) than white matter (about
neurological emergencies. Neurocrit Care 2004;1[3]‌:287-​99; used
50 mL/​100 g/​min). When CPP is compromised and CBF
with permission.)

Mayo Clinic Neurology Board Review, edited by Kelly D. Flemming, Oxford University Press, Incorporated, 2021. ProQuest Ebook Central, http://ebookcentral.proquest.com/lib/cuhk-ebooks/detail.action?docID=6746419.
Created from cuhk-ebooks on 2023-09-02 09:10:26.
468 Section IV. Neurologic Intensive Care Disorders

flow to areas of least resistance and move toward the fiber temporal lobe herniates under tentorium cerebelli into ten-
tracts instead of more cortically. Because no direct dam- torial incisura, displacing the midbrain. Patients pre­sent
age to the cell occurs, edema can resolve with correction with ipsilateral pupillary dilatation due to compression of
of the capillary damage. Vasogenic edema is most com- the third cranial nerve or involvement of its nuclei from
mon among patients with masses such as tumors or mesencephalic shift and rotation. Uncal herniation also
abscesses. may compress the posterior cerebral artery, with subse-
In cytotoxic edema, damage to the cell membrane pre- quent infarction of the occipital lobe.
vents control of the movement of fluid between cells. The In tonsillar (cerebellar cone or foraminal impaction)
death of tissue from stroke, toxins (eg, acute hepatic fail- herniation, downward herniation of cerebellar tonsils
ure), or trauma causes loss of adenosine triphosphatase through the foramen magnum leads to compression of the
function and energy failure. Loss of adenosine triphospha- medulla. Patients present with coma and respiratory arrest,
tase function prevents cell control of the movement of associated with episodic extensor posturing.
fluid between the cell and the surrounding interstitium. Subfalcine (cingulate or supracallosal) herniation is a
Cytotoxic edema occurs in the early stages of cerebral isch- result of the cingulate gyrus herniating under the falx cere-
emia. It also is caused by hypo-​ osmolality, wherein bri. A frontoparietal mass exerts force on the cingulate
increased water content in the bloodstream compared with gyrus, pressing the brain under the falx. This shift may be
intracellular contents results in water flow into the cells mostly asymptomatic, but if severe, it may result in contra-
because of an osmotic gradient. lateral leg weakness, decreased mental status, and even
In interstitial edema, fluid accumulates in the intersti- anterior cerebral artery infarction.
tial spaces because of hydrocephalus. Edema is caused by With acutely increased ICP and brainstem shift, Duret
CSF being pushed into extracellular fluid in the periven- hemorrhage of the brainstem may develop because of
tricular white matter. Interstitial edema may occur in shearing of basilar artery perforators (Figure 56.4). Its pres-
patients with hydrocephalus. ence indicates severe injury with a poor prognosis.
Upward transtentorial herniation occurs when a poste-
• Intracranial hypertension is defined as sustained ICP rior fossa mass creates upward herniation of posterior
>20 mm Hg. fossa contents through the tentorial notch, usually because
• CPP = MAP − ICP. of excessive ventricular CSF drainage. Often, the cerebral
• The ischemic threshold is <20 mL/​100 g/​min CBF. aqueduct or fourth ventricle becomes occluded, and
• Brain edema has 3 types: cytotoxic, vasogenic, and obstructive hydrocephalus results. It is unclear if a dis-
interstitial. tinct clinical picture exists beyond decline of conscious-
ness due to brainstem compression from a new posterior
fossa mass.
Increased ICP One entity, referred to as low pressure herniation,
occurs with herniation or displacement of the brain tissue
Clinical Presentation in the clinical setting of low ICP (a leak in the s­ pinal cord
The neurologic history and examination are the best way is a common cause). Patients clinically deteriorate, and
to determine whether a patient has increased ICP or has their treatment is opposite of the usual treatment, with
early signs of herniation due to increased mass effect. If the Trendelenburg position rather than 30° head ­elevation.
ICP increases slowly, the earliest symptom is headache.
Copyright © 2021. Oxford University Press, Incorporated. All rights reserved.

When ICP becomes critically elevated, symptoms may Diagnosing and Monitoring Increased ICP
include decreased alertness, vomiting, diplopia (often cra- For a patient in whom increased ICP is known or suspected,
nial nerve 6 palsy), and pupillary changes. Untreated, the any clinical deterioration should prompt head computed
patient’s condition will progress to coma. Acute, sudden tomography (CT). CT can be used to assess for worsening
elevations in ICP may result in increased blood pressure, brain shift and edema and for ventricular enlargement.
irregular respirations, and bradycardia—​ part of the so-​ Abnormal CT scans may hint toward increased ICP. Alone,
called Cushing reflex (or Cushing triad). however, CT does not provide a reliable estimation of ICP.
Monitoring ICP can be done through various ways. One
Herniation Syndromes can assess the ICP and the waveforms (Figure 56.5). A reli-
The brain is compartmentalized by the falx and the tento- able way to measure ICP is with an intraventricular cathe-
rium. Herniation is movement of brain tissue into another ter. In addition to its use for quantitative measurement, the
compartment (Figure 56.3). catheter can be used to drain CSF and thus provide a
Uncal herniation, or lateral transtentorial herniation, means to decrease ICP. However, all invasive techniques
occurs when the medial temporal lobe herniates down- have risks, including hemorrhage in the placement track
ward past the tentorium. In this process, the medial and (less common) infection.

Mayo Clinic Neurology Board Review, edited by Kelly D. Flemming, Oxford University Press, Incorporated, 2021. ProQuest Ebook Central, http://ebookcentral.proquest.com/lib/cuhk-ebooks/detail.action?docID=6746419.
Created from cuhk-ebooks on 2023-09-02 09:10:26.
Chapter 56. Principles and Management of Increased Intracranial Pressure 469

     

Figure 56.3 Herniation Syndromes. A, Uncal herniation from a subdural hematoma. Bottom right aspect of figure shows the
uncus of temporal lobe herniating over the tentorium cerebelli, with vascular compression of the PCA and mechanical
compression of the third cranial nerve (III). Upper half of figure shows subfalcine herniation with brain herniating under the
falx cerebri with vascular compression of the ACA. B, Posterior fossa hemorrhage causing downward cerebellar tonsillar
herniation through the foramen magnum. ACA indicates anterior cerebral artery; PCA, posterior cerebral artery.
(From Wijdicks EFM. Catastrophic neurologic disorders in the emergency department. 2nd ed. Oxford [UK]: Oxford University Press;
c2004. Chapter 8, Altered arousal and coma; p. 53-​93; used with permission of Mayo Foundation for Medical Education and Research.)

ICP can be measured with various other types of moni-


tors. An ICP monitor can be placed in the subdural space,
epidural space, or brain parenchyma. These systems do not
allow therapeutic drainage of CSF, and the measured ICP
may not reflect the true CSF pressure because of drift in
values over time. Most major medical institutions in the
world use a fiber optic device in such cases. The device
Copyright © 2021. Oxford University Press, Incorporated. All rights reserved.

includes a mirror that reflects light emission from the fiber


optic cable. The change in light reflection is due to change
in the mirror position generally and to change in its posi-
tion due to compression from brain tissue under pressure.

Treatment
General Treatment
Identifying and treating the underlying cause are impor-
tant. Medical and surgical options are also implemented to
Figure 56.4 Duret Hemorrhages Produced by Severe
reduce ICP until definitive treatment of the underlying
Downward Herniation.
cause. The goals of medical management of increased ICP
(From Flemming KD. Principles of critical care neurology. In:
(Box 56.1) are to maintain brain perfusion and prevent
Mowzoon N, Flemming KD, editors. Neurology board review: an
hypotension.
illustrated study guide. Rochester [MN]: Mayo Clinic Scientific
Patient position can affect ICP. Ensuring that the head of
Press and Florence [KY]: Informa Healthcare USA; c2007. p. 401-​
the bed is raised to 30° allows a decrease in hydrostatic pres-
34; used with permission of Mayo Foundation for Medical
sure, facilitates venous drainage, and therefore may reduce
Education and Research.)

Mayo Clinic Neurology Board Review, edited by Kelly D. Flemming, Oxford University Press, Incorporated, 2021. ProQuest Ebook Central, http://ebookcentral.proquest.com/lib/cuhk-ebooks/detail.action?docID=6746419.
Created from cuhk-ebooks on 2023-09-02 09:10:26.
470 Section IV. Neurologic Intensive Care Disorders

Figure 56.5 ICP Waveform. Upper image (red) is a normal ICP waveform. Bottom image (blue) is a noncompliant waveform.
ICP indicates intracranial pressure with reduced compliance.
(Modified from Chesnut RM, Marshall LF. Management of head injury: treatment of abnormal intracranial pressure. Neurosurg Clin N Am
1991 April;2[2]‌:267-​84; used with permission.)

ICP. In addition, the patient’s neck should be in the neutral Pharmacologic Treatment
position, be facing forward, and not be hyperextended or Among the pharmacologic treatments (Box 56.1), mannitol
flexed. Neck flexion can compress the internal jugular vein can be selected as a temporizing measure for increases in
and decrease venous drainage, subsequently increasing ICP. ICP by increasing serum osmolarity, thereby pulling fluid
For patients who are intubated, temporary measures out of tissue and, in an edematous brain, decreasing ICP.
can be used to decrease ICP. Hyperventilation to decrease Typically, mannitol is administered as a bolus of 1.0 g/​kg,
the carbon dioxide (CO2) concentration can reduce ICP. followed by repeat dosing of 0.5 g/​kg every 6 hours. When
Reduced CO2 leads to arterial vasoconstriction, which low- scheduling mannitol, the clinician should monitor the
ers CBF and cerebral blood volume, thereby decreasing osmolal gap. The main goal is an osmotic gap of less than
ICP. Although reduction in CO2 can be done relatively 20 mOsm/​kg, and mannitol therapy can continue as long
quickly, it should be used for only a short time because as these values are achieved. The diuresis can lead to
vasoconstriction can lead to ischemia or to a later rebound hypotension and electrolyte (eg, potassium, magnesium,
increase in ICP. For a stable, mechanically ventilated phosphorus) depletion. Mannitol is contraindicated for
patient without increased ICP, adequate sedation should patients with severe renal disease.
Copyright © 2021. Oxford University Press, Incorporated. All rights reserved.

be used to prevent coughing, which can increase ICP. Hypertonic saline (23.4%) has an efficacy similar to or
Normothermia should be achieved and maintained better than equiosmolar doses of mannitol. It is a plasma
throughout the patient’s course in the intensive care unit. expander and is preferred for patients with marginal renal
Mild to moderate hypothermia (34°C-​36°C) can be consid- function. Hypertonic saline is a considerable volume load
ered to reduce ICP through a reduction in cerebral meta- for patients with underlying congestive heart failure and
bolic rate of oxygen and CBF if ICP is refractory to must be administered via a central catheter.
conventional osmotherapy and through other interven- Glucocorticoids are most effective in treating vasogenic
tions. Caution should be exercised when implementing edema and are less effective for traumatic brain injury,
hypothermia because whether it improves outcomes com- ischemic stroke, and hemorrhage. Corticosteroids can
pared with normothermia is unknown in any acute neuro- cause hyperglycemia, diabetes mellitus, acute agitation,
logic disorder. Hypothermia is associated with increased tremors, peptic ulcers, or immunosuppression; impair
mortality rates in bacterial meningitis. Other adverse wound healing; or worsen preexisting diabetes mellitus.
effects include infection, coagulation and hemorrhagic Anesthetics, such as propofol and barbiturates, can be
complications, electrolyte abnormalities, and shivering. given in cases of refractory elevated ICP. Propofol decreases
These are rare with mild to moderate hypothermia. cerebral brain metabolism (cerebral metabolic rate of

Mayo Clinic Neurology Board Review, edited by Kelly D. Flemming, Oxford University Press, Incorporated, 2021. ProQuest Ebook Central, http://ebookcentral.proquest.com/lib/cuhk-ebooks/detail.action?docID=6746419.
Created from cuhk-ebooks on 2023-09-02 09:10:26.
Chapter 56. Principles and Management of Increased Intracranial Pressure 471

increased ICP cannot be resected, such as swollen hemi-


Box 56.1 • Treatment of Increased Intracranial spheric and cerebellar infarcts and deep ganglionic and
Pressure pontine hemorrhages. In these situations, options are medi-
cal management or relief of pressure with removal of a large
Medical
bone flap.
Head of bed positioned to 30°
With a ventricular catheter in situ, CSF drainage can be
Normothermia or hypothermia (selected patients) controlled. Normally, the catheter is placed at the level of the
Normoglycemia foramen of Monro, located at the tragus of the ear. The
Normotension of MAP more than 80 mm Hg or CPP manometer can be placed at various levels. When catheter
more than 60 mm Hg pressure exceeds the set value, CSF drains out. The ventricu-
Oxygen saturation greater than 96% and Pao2 lar compartment may become compressed or a major clot
100 mm Hg
may impede drainage. A lumbar drain is contraindicated in
Hyperventilation to a Paco2 30 to 35 mm Hg any patient for whom increased ICP is suspected, because of
(short term)
the risk of further tissue displacement with pressure relief.
Pharmacologic
Mannitol 20%, 1 g/​kg in bolus administration • Acute, sudden elevations in ICP may result in
Hypertonic saline (23.4%), 30 mL in bolus increased blood pressure, irregular respirations, and
administration bradycardia—part of the so-called Cushing reflex (or
Glucocorticoids (vasogenic edema) Cushing triad).
Sedation • Patients with uncal herniation present with ipsilateral
Neuromuscular blockade (rare) pupillary dilatation due to compression of the third
Barbiturates (rare) cranial nerve and decreased mental status.
• Patients with central transtentorial herniation often
Neurosurgical
first have a decline in mental status and consciousness,
Resection of mass lesion
which is followed by respiratory decompensation,
Drainage of CSF by ventriculostomy extensor posturing, bilateral pupil dilatation, and palsy
Hemicraniectomy (selected patients) of the fourth cranial nerve.
• ICP management includes medical, pharmacologic,
Abbreviations: CPP, cerebral perfusion pressure; CSF,
cerebrospinal fluid; MAP, mean arterial pressure. and neurosurgical approaches.
• Among the pharmacologic treatments, mannitol or
hypertonic saline can be used as a temporizing
measure for increased ICP by increasing serum
oxygen) with subsequent reduction in CBF, which reduces osmolality, thereby pulling fluid out of tissue and, in
global ICP (similar to barbiturates). It has the benefit of a an edematous brain, decreasing ICP.
short half-​life, which allows for periods of discontinued
use for intermittent clinical monitoring. Propofol, how-
ever, is not a good long-​term option. Caution should be
Specific Situation: Malignant
used because propofol can cause hypotension that requires
vasopressors, hypertriglyceridemia (pancreatitis and nutri- Ischemic Stroke
Copyright © 2021. Oxford University Press, Incorporated. All rights reserved.

tion adjustments), metabolic acidosis, and seizure. In Large ischemic strokes may result in substantial brain edema
younger patients, it can result in propofol infusion syn- (Figure 56.6). Typically, edema develops within several
drome, which can cause severe acidosis, acute shock syn- hours of an ischemic stroke and peaks at 72 to 120 hours.
drome, and death. Propofol infusion syndrome can be Risk factors for malignant cerebral edema after stroke include
prevented with a protocol in which propofol is not admin- younger age and large distribution of infarction from large
istered at doses of 4 mg/​ kg or greater for more than artery disease, such as the middle cerebral artery (MCA) or
48 hours. internal carotid artery territory (eg, National Institutes of
Health Stroke Scale score >20, early involvement of >50% of
Neurosurgical Interventions MCA territory, involvement of the temporal lobe).
Treatment of increased ICP differs by cause. Intracranial Patients presenting with signs and symptoms of malig-
masses that cause increased ICP should be assessed by a nant stroke should receive medical therapy for cerebral
neurosurgeon for possible resection or craniotomy, or both. edema. Early hemicraniectomy may be considered for
Hematomas, especially epidural hematomas, should be selected patients.
evacuated because they are under arterial pressure. Any In a pooled meta-​analysis of patients with malignant
subdural hematoma with shift on CT scan (usually >10 mm) infarction of the MCA, decompressive hemicraniectomy
should be considered for treatment. Some lesions causing undertaken within 48 hours reduced the mortality rate and

Mayo Clinic Neurology Board Review, edited by Kelly D. Flemming, Oxford University Press, Incorporated, 2021. ProQuest Ebook Central, http://ebookcentral.proquest.com/lib/cuhk-ebooks/detail.action?docID=6746419.
Created from cuhk-ebooks on 2023-09-02 09:10:26.
472 Section IV. Neurologic Intensive Care Disorders

infarction. Patients should be considered on a case-​by-​case


basis. In some instances, hemicraniectomy may be indi-
cated when the edema is severe. A posterior fossa decom-
pression may be indicated for patients with cerebellar
infarction and subsequent edema and mass effect or with
cerebral aqueductal compression.

Specific Situation: Traumatic


Brain Injury
Treatment is often guided with use of CT scan findings.
Any traumatic extradural hematoma may need acute neu-
rosurgical intervention. A comatose patient with traumatic
brain injury without a mass producing contusion or diffuse
axonal injury requires an ICP monitor and treatment with
Figure 56.6 Malignant Middle Cerebral Artery Syndrome. conventional measures (eg, head elevation, temperature
Computed tomographic scan of the head shows a large right and seizure control) and aggressive but careful use of
middle and anterior cerebral artery infarction with severe osmotic agents. Decompressive craniotomy may become
mass effect and midline shift. necessary in later stages for patients with blossoming con-
tusions. This topic is discussed in more detail in Chapter
60 (“Traumatic Brain Injury”).
increased favorable functional outcome for patients age 60
years or younger. One study showed improved survival for
patients older than 60 years who had malignant MCA
Copyright © 2021. Oxford University Press, Incorporated. All rights reserved.

Mayo Clinic Neurology Board Review, edited by Kelly D. Flemming, Oxford University Press, Incorporated, 2021. ProQuest Ebook Central, http://ebookcentral.proquest.com/lib/cuhk-ebooks/detail.action?docID=6746419.
Created from cuhk-ebooks on 2023-09-02 09:10:26.
Status Epilepticus
57 MAXIMILIANO A. HAWKES, MD; SARA E. HOCKER, MD

Introduction seizure termination fail. The endocytic internalization of

S
tatus epilepticus (SE) is a medical and neurologic the inhibitory γ-​aminobutyric acidA and externalization of
emergency defined as persistent seizure activity the excitatory glutamatergic α-​amino-​3-​hydroxyl-​5-​methyl-​
lasting more than 5 minutes or recurrent seizures 4-​isoxazole-​propionate and N-​methyl-​d-​aspartate receptors
without recovery of consciousness in between. When sei- lead to seizure perpetuation and pharmacologic resistance.
zures persist despite adequate doses of first-​and second-​ Expression of proconvulsive neuropeptides (neurokinin B
line antiepileptic agents, the condition is called refractory and substance P) and depletion of inhibitory neuropep-
SE (RSE). Super-​refractory SE (SRSE) occurs when seizure tides (dynorphin and somatostatin) further contribute to
activity continues or recurs 24 hours after the initiation of seizure self-​perpetuation.
therapy with anesthetic agents. After 30 minutes of seizure activity, pathologic changes
This chapter reviews the epidemiologic factors, diagno- occur, including neuronal injury and death and alteration
sis, classification, and treatment of SE. of neuronal networks. Neuronal death results from the
increase in intracellular calcium due to excitotoxicity,
metabolic disturbances (including hypoxia and hypogly-
cemia), and increased intracranial pressure. In prolonged
Epidemiologic and Etiologic Factors cases, changes in gene expression result in a chronic pro-
The incidence of SE is 6.8 to 41 cases per 100,000 persons epileptic state.
annually, with no predilection for any particular sex, race,
or ethnic group. The age distribution follows a bimodal
curve, with the highest incidence in the first year of life
and after age 60 years. Prospective data show that 23% and
Table 57.1 • Precipitating Factors of Status Epilepticus in
9% of SE cases are RSE and SRSE, respectively.
Patients With and Without Epilepsy
Copyright © 2021. Oxford University Press, Incorporated. All rights reserved.

The majority of SE episodes (54%) occur without a


prior history of seizures. Precipitating factors differ among Patients Without
patients with and without epilepsy (Table 57.1). Most SE Epilepsy Patients With Epilepsy
of patients without a history of seizures is triggered by
Acute stroke Low levels of antiepileptic drugs
acute brain injury. Remote symptomatic lesions cause
Brain tumors Abrupt changes in antiepileptic
approximately 30% of SE cases, and 5% are idiopathic. Meningoencephalitis drug regimen
Trauma Drug intoxication or withdrawal
Metabolic derangements Systemic infection
Acute febrile illnesses Metabolic derangement
Pathophysiologic Factors (children) Progression of the underlying
disease responsible for the
Most seizures terminate spontaneously. SE can occur when
seizures
the excitatory and inhibitory mechanisms responsible for

The editors and authors acknowledge the contributions of Matthew T. Hoerth, MD, to the previous edition of this chapter.
Abbreviations: NCSE, nonconvulsive status epilepticus; RSE, refractory status epilepticus; SE, status epilepticus; SRSE, super-​refractory
status epilepticus

473

Mayo Clinic Neurology Board Review, edited by Kelly D. Flemming, Oxford University Press, Incorporated, 2021. ProQuest Ebook Central, http://ebookcentral.proquest.com/lib/cuhk-ebooks/detail.action?docID=6746419.
Created from cuhk-ebooks on 2023-09-02 09:10:26.
474 Section IV. Neurologic Intensive Care Disorders

Clinical Features death. Emergency evaluation should begin with securing


the airway, breathing, and circulation. The glucose value
Diagnosis largely relies on neurologic examination and con- and oxygen status should be checked rapidly. Oxygen
firmation through electroencephalography (Figure 57.1). should be provided and thiamine administered before cor-
The International League Against Epilepsy proposed a clas- rection of hypoglycemia. Initial laboratory evaluation
sification system in 2015 (Box 57.1). Generalized tonic-​ should include arterial blood gases, renal and hepatic
clonic SE is characterized by coma, increased tone, and function, calcium and magnesium values, complete blood
rhythmic jerking of the extremities. These characteristics cell count, toxicology screening, and determination of anti-
are followed by a depressed level of consciousness with or convulsant levels of patients with epilepsy. Correction of
without a focal neurologic deficit and lasting hours to days. metabolic abnormalities and control of hyperthermia are
Nonconvulsive SE (NCSE) has 2 main forms of presen- necessary for treatment of SE and prevention of recurrent
tation: progressive intermittent confusion or stupor with seizures. Electrocardiography and neuroimaging are rec-
or without rhythmic muscle twitches or eye deviation (so-​ ommended for all patients. Lumbar puncture and cerebro-
called subtle SE). NCSE may affect 10% to 15% of patients spinal fluid analysis should be performed if fever is
with altered mental status who are admitted to the hospi- present, recent behavioral or speech changes are reported,
tal, and its diagnosis requires a high level of suspicion. or no cause is readily apparent.
An electroencephalogram should be obtained emer-
gently for patients who do not return to cognitive baseline
Evaluation and Management within 10 minutes after a clinical seizure or SE and those
An algorithmic approach to SE is outlined in Figure 57.2. with persistent altered mental status or coma without a
Following a treatment protocol reduces morbidity and clear cause.

F7-Ab
F3-Ab
Fz-Ab
F4-Ab
F8-Ab

T7-Ab
C3-Ab
Cz-Ab
C4-Ab
T8-Ab

P7-Ab
Copyright © 2021. Oxford University Press, Incorporated. All rights reserved.

P3-Ab
Pz-Ab
P4-Ab
P8-Ab

O1-Ab
Oz-Ab
O2-Ab

Figure 57.1 Nonconvulsive Status Epilepticus. Electroencephalogram shows continuous, diffuse rhythmic activity.
(From Flemming KD. Principles of critical care neurology. In: Mowzoon N, Flemming KD, editors. Neurology board review: an illustrated
study guide. Rochester [MN]: Mayo Clinic Scientific Press and Florence [KY]: Informa Healthcare USA; c2007. p. 401-​34; used with
permission of Mayo Foundation for Medical Education and Research.)

Mayo Clinic Neurology Board Review, edited by Kelly D. Flemming, Oxford University Press, Incorporated, 2021. ProQuest Ebook Central, http://ebookcentral.proquest.com/lib/cuhk-ebooks/detail.action?docID=6746419.
Created from cuhk-ebooks on 2023-09-02 09:10:26.
Chapter 57. Status Epilepticus 475

seizures persist, 20 mg/​kg phenytoin equivalents of fos-


Box 57.1 • Classification of Status Epilepticus (SE) phenytoin should be administered (an additional 5-​10
mg/​kg can be considered if seizures persist) or adminis-
A. With prominent motor symptoms
tration of 40 mg/​kg of valproic acid (maximum, 3,000
A.1. Convulsive SE (CSE), also called tonic-​clonic SE mg), or 60 mg/​kg levetiracetam (maximum, 4,500 mg).
A.1.a. Generalized convulsive For hemodynamically unstable patients, valproic acid
A.1.b. Focal onset evolving into bilateral CSE and levetiracetam may be the drugs of choice. If SE is
A.1.c. Unknown whether focal or generalized caused by drug withdrawal, the withdrawn drug should
A.2. Myoclonic SE (prominent epileptic be replaced immediately with parenteral administration,
myoclonic jerks) if possible.
A.2.a. With coma If seizures persist after adequate doses of benzodiaze-
A.2.b. Without coma pine and of fosphenytoin, valproic acid, or levetiracetam,
A.3. Focal motor consideration should be given to intubation, patient trans-
fer to an intensive care unit, infusion of an anesthetic agent
A.3.a. Repeated focal motor seizures (Jacksonian)
(midazolam, propofol, or pentobarbital), and continuous
A.3.b. Epilepsia partialis continua
electroencephalography. Treatment to burst suppression
A.3.c. Adversive status
should be given for 12 to 48 hours. Sedation should be
A.3.d. Oculoclonic status
withdrawn gradually and guided by the electroencephalo-
A.3.e. Ictal paresis (ie, focal inhibitory SE) gram. Maintenance antiepileptic drug administration
A.4. Tonic status should be continued at therapeutic doses.
A.5. Hyperkinetic SE Alternative antiepileptic agents, including lacosamide
B. Without prominent motor symptoms (ie, and levetiracetam, are currently not recommended as first-​
nonconvulsive SE [NCSE]) or second-​line agents for control of SE. Although they are
B.1. NCSE with coma (including so-​called subtle SE) widely used, their effectiveness and safety are not clearly
B.2. NCSE without coma established in this clinical setting. These agents may be
B.2.a. Generalized selected on a case-​by-​case basis for RSE, such as cases
where endotracheal intubation is not an option. Preferred
B.2.a.a. Typical absence status
medications are those that do not depress the breathing
B.2.a.b. Atypical absence status
drive or result in upper airway collapse.
B.2.a.c. Myoclonic absence status
The evidence-​ based treatment of RSE and SRSE is
B.2.b. Focal
poorly established. The literature provides multiple case
B.2.b.a. Without impairment of consciousness series and case reports. The attempted therapies reported
(aura continua, with autonomic, sensory,
include but are not limited to ketamine, lidocaine, leveti-
visual, olfactory, gustatory, emotional/​
psychic/​experiential, or auditory symptoms) racetam, lacosamide, electroconvulsive therapy, topira-
B.2.b.b. Aphasic status mate, ketogenic diet, and inhalational anesthetics.
B.2.b.c. With impaired consciousness
• The term status epilepticus refers to a seizure that lasts
B.2.c. Unknown whether focal or generalized
>5 minutes or ≥2 seizures with incomplete interictal
B.2.c.a. Autonomic SE
recovery.
Copyright © 2021. Oxford University Press, Incorporated. All rights reserved.

From Trinka E, Cock H, Hesdorffer D, Rossetti AO, Scheffer IE, • Glucose level and oxygen status should be checked
Shinnar S, et al. A definition and classification of status rapidly for patients with seizure.
epilepticus: report of the ILAE Task Force on Classification • Correction of metabolic abnormalities and control of
of Status Epilepticus. Epilepsia. 2015 Oct;56(10):1515-​23.
Epub 2015 Sep 4; used with permission. hyperthermia are necessary for treatment of status
epilepticus and prevention of recurrent seizures.
• Benzodiazepines should be administered while
monitoring the airway.
Details of medications are listed in Table 57.2. • If seizures persist after adequate doses of
Pharmacologic treatment should occur simultaneously benzodiazepine and fosphenytoin, valproic acid, or
with the evaluation detailed above. Benzodiazepines levetiracetam, consideration should be given to
should be administered while monitoring the airway. The intubation, transfer to an intensive care unit, infusion
advantages of benzodiazepines include rapid onset of of an anesthetic agent (midazolam, propofol, or
action efficacy and their versatility (ie, they can be given pentobarbital), continuous electroencephalography,
intravenously, intramuscularly, buccally, or rectally). If and treatment to burst suppression.

Mayo Clinic Neurology Board Review, edited by Kelly D. Flemming, Oxford University Press, Incorporated, 2021. ProQuest Ebook Central, http://ebookcentral.proquest.com/lib/cuhk-ebooks/detail.action?docID=6746419.
Created from cuhk-ebooks on 2023-09-02 09:10:26.
476 Section IV. Neurologic Intensive Care Disorders

Lorazepam IV Alternative: Concurrent management


A: 0.1 mg/kg in divided doses Midazolam IM • ABC
P: 0.05–0.5 mg/kg in divided Weight 13–40 kg: 5 mg • Oxygen by nasal cannula
doses
Weight >40 kg: 10 mg • Establish IV access
• Pulse oximetry, cardiac monitor
• Glucose stick

Phenytoin IV or Valproate sodium IV Levetiracetam IV Laboratory tests


fosphenytoin Loading: 40 mg/kg Loading: 20–60 mg/kg • Arterial blood gases
Loading (A/P): 18–20 mg/kg • CBC, calcium, glucose
• Chemistry
• Liver function tests
• Creatine kinase
• Toxicology screen
• Antiepileptic drug levels
(as indicated)
Brain imaging
• Head CTª
Cerebrospinal fluid
• If cause cannot be determinedª

Pentobarbital IV Propofol IV Midazolam IV Ketamine IV Prepare atropine 0.5 mg if


Loading 5–10 mg/kg 2–5 mg/kg bolus 0.2 mg/kg bolus 0.5–4.5 mg/kg bolus
bradycardiac
1–5 mg/kg/h infusion 5–10 mg/kg/h infusion 0.2–0.4 mg/kg/h infusion 1–5 mg/kg/h infusion Consider transfer to an intensive
care unit
Endotracheal intubation and
mechanical ventilation
Continuous EEG monitoring
Inotrope or vasopressor support
of blood pressure when necessary

Figure 57.2 Management of Status Epilepticus. aThe priority is to stop the status epilepticus; tests to be performed after
patient status is stabilized. A indicates adult; ABC, airway, breathing, and circulation; CBC, complete blood cell count; CT,
computed tomography; EEG, electroencephalography; IM, intramuscularly; IV, intravenously; P, pediatric.
(Modified from Wijdicks EFM. Catastrophic neurologic disorders in the emergency department. 2nd ed. Oxford [UK]: Oxford University
Copyright © 2021. Oxford University Press, Incorporated. All rights reserved.

Press; c2004. Chapter 10, Status epilepticus and recurrent seizures; p. 106-​20; used with permission of Mayo Foundation for Medical
Education and Research.)

Box 57.2 • Systemic Complications of Status


Systemic Complications Epilepticus
Profound physiologic changes triggered and perpetuated by
Hyperthermia Aspiration
SE may result in widespread systemic consequences (Box
Hypoxia Stress-induced cardiomyopathy
57.2). Tachycardia, hyperglycemia, hypertension, and
Acidosis Tongue lacerations
hyperthermia occur in the early stages of SE because of a
catecholaminergic surge (which compensates for a high Mild leukocytosis Long bone and compression fractures
metabolic demand) and vigorous muscle contraction. A Rhabdomyolysis Posterior shoulder dislocation
transient marked acidosis often occurs with respiratory and Trauma
metabolic components. Unlike hyperthermia, acidosis does

Mayo Clinic Neurology Board Review, edited by Kelly D. Flemming, Oxford University Press, Incorporated, 2021. ProQuest Ebook Central, http://ebookcentral.proquest.com/lib/cuhk-ebooks/detail.action?docID=6746419.
Created from cuhk-ebooks on 2023-09-02 09:10:26.
Chapter 57. Status Epilepticus 477

Table 57.2 • Medications Used to Manage Status Epilepticus


Medication Loading Dose, Intravenous Maintenance Dose Therapeutic Level, mcg/​mL

Lorazepam 0.1 mg/​kg (typically, start with 1–2 mg; NA NA


increase if seizures continue)
Diazepam 0.15 mg/​kg NA
Phenytoin/​fosphenytoin 18–​20 mg/​kg 300 mg/​d (adult); 10–​20
4–​6 mg/​kg per d (children)
Valproic acid 40 mg/​kg 1,000–​4,000 mg/​d (adult) 50–​100
Phenobarbital 20 mg/​kg 200–​320 mg/​d (adult); 10–​40
3–​6 mg/​kg per d (children)
Propofol 2–​5 mg/​kg 5–​10 mg/​kg per h NA
Midazolam 0.2 mg/​kg 0.2–​0.4 mg/​kg per h NA
Pentobarbital 5–​10 mg/​kg 1–​5 mg/​kg per h NA
Lacosamide 200–400 mg 200–​400 mg/​d NA
Levetiracetam 20–60 mg/​kg 1,000–​3,000 mg/​d (adult) 12.0–​46.0
Abbreviation: NA, not applicable.
Modified from Flemming KD. Principles of critical care neurology. In: Mowzoon N, Flemming KD, editors. Neurology board review: an illustrated study
guide. Rochester (MN): Mayo Clinic Scientific Press and Florence (KY): Informa Healthcare USA; c2007. p. 401–​34; used with permission of Mayo
Foundation for Medical Education and Research.

A B
Copyright © 2021. Oxford University Press, Incorporated. All rights reserved.

     

Figure 57.3 Cortical Atrophy After Prolonged Status Epilepticus, Shown on Magnetic Resonance Imaging. A, Brain of a
23-​year-​old woman at the onset of status epilepticus. B, Same patient after 4 months of continued status epilepticus despite
aggressive treatment. Cortical atrophy is widespread and severe for her age. Note especially the atrophy of the hippocampus
(arrows).

Mayo Clinic Neurology Board Review, edited by Kelly D. Flemming, Oxford University Press, Incorporated, 2021. ProQuest Ebook Central, http://ebookcentral.proquest.com/lib/cuhk-ebooks/detail.action?docID=6746419.
Created from cuhk-ebooks on 2023-09-02 09:10:26.
478 Section IV. Neurologic Intensive Care Disorders

not require specific treatment and resolves with termina- Outcome and Prognosis
tion of the seizure. Mild leukocytosis from demargination
of neutrophils is common and may be present in the serum The mortality rate related to SE is 10% to 20% and reaches
or cerebrospinal fluid. Convulsive SE can cause hypoxia by 32% in RSE. The rate depends on patient age, history of
mucus plugging, neurogenic pulmonary edema, aspiration seizures, medical comorbidities, electroencephalographic
pneumonitis, or apnea, or a combination of mechanisms. A findings, level of consciousness on presentation, and sei-
reversible myocardial stunning (Takotsubo cardiomyopa- zure type, and mainly on the cause and refractoriness of
thy) should be suspected in patients with hypotension, pul- the seizures.
monary edema, increased cardiac troponin, or cardiogenic Long-​term complications include cortical atrophy with
shock. A substantial proportion of patients present with cognitive impairment, physical deconditioning from pro-
sinus tachycardia or bradycardia. Atrial fibrillation or flut- longed intensive care, and development of epilepsy
ter, ventricular tachycardia or fibrillation, and atrioventric- (mesial temporal sclerosis in 20%-​ 40% of patients)
ular block are less frequent findings. (Figure 57.3).
Rhabdomyolysis results from repeated muscle contrac-
tion with muscle breakdown and can result in acute kidney • The strongest predictors of outcome in status
injury if hydration is inadequate. Musculoskeletal trauma epilepticus are cause and duration.
is a frequent finding in convulsive SE. Common injuries are • The mortality rate is 10%-​20% in status epilepticus,
lateral tongue bites, posterior fractures or dislocations of with an average of 32% in refractory status
the shoulder, and falls resulting in various injuries. epilepticus.
Copyright © 2021. Oxford University Press, Incorporated. All rights reserved.

Mayo Clinic Neurology Board Review, edited by Kelly D. Flemming, Oxford University Press, Incorporated, 2021. ProQuest Ebook Central, http://ebookcentral.proquest.com/lib/cuhk-ebooks/detail.action?docID=6746419.
Created from cuhk-ebooks on 2023-09-02 09:10:26.
Nontraumatic Subarachnoid
58 Hemorrhagea
TIA CHAKRABORTY, MD; JENNIFER E. FUGATE, DO

Introduction Clinical Characteristics

S
ubarachnoid hemorrhage (SAH) is defined as blood The clinical hallmark of aSAH is an instantaneous onset of
in the subarachnoid space. Nontraumatic SAH is a maximally severe headache (ie, thunderclap headache).
most commonly caused by rupture of an aneurysm This symptom may be associated with loss of conscious-
located at the circle of Willis. Patients often present with ness due to massively increased intracranial pressure
acute thunderclap headache but also may lose conscious- (ICP). Other common associated symptoms are nausea,
ness or have focal neurologic deficits. Detection of an vomiting, neck pain, neck stiffness, and photophobia.
aneurysm, if present, and its treatment are needed urgently Generalized tonic-​clonic seizures can occur but are rare
to prevent recurrent bleeding. Patients with nontraumatic (<5%). In comatose patients, adventitious movements such
SAH are prone to numerous complications that require as extensor posturing or myoclonus may be mistaken for
preventative measures, early recognition, and treatment. seizure at presentation.
This chapter reviews the clinical symptoms, diagnosis, Although SAH is most common with aneurysmal rup-
and treatment of patients with nontraumatic SAH. The ture, it may occur with intraparenchymal or arterial sub-
topic of unruptured intracranial aneurysms is covered in dural hematomas and may cause focal neurologic deficits.
Chapter 53, “Unruptured Intracranial Aneurysms and Ocular hemorrhages such as retinal subhyaloid hemor-
Vascular Malformations.” rhages (in about one-​quarter of patients) due to sudden
massively increased ICP can be detected by a funduscopic
examination, which should be performed for all patients
Epidemiologic Factors with aSAH. Patients may have severe visual loss, particu-
Copyright © 2021. Oxford University Press, Incorporated. All rights reserved.

larly in cases of vitreous hemorrhage (Terson syndrome)


Aneurysmal SAH (aSAH) occurs in 2 to 16 per 100,000 (Figure 58.1), or they may be bothered by floating “blobs”
person-​years in most populations globally. The risk of aSAH that obstruct their vision. A third cranial nerve palsy may
increases with age, but about one-​half of patients are younger occur with compression of the third nerve because it exits
than 55 years. Women and persons of black and Hispanic the brainstem and courses between the posterior commu-
ethnicity have a higher incidence of aSAH. Modifiable risk nicating and posterior cerebral arteries.
factors for aSAH include smoking and hypertension. In comatose patients, a downward gaze may indicate
the presence of acute hydrocephalus. If coma is due to
• Modifiable risk factors for aSAH are smoking and hydrocephalus, the placement of a ventriculostomy could
hypertension. dramatically improve the patient’s level of consciousness.

a
Portions previously published in Fugate JE, Rabinstein AA. Intensive care unit management of aneurysmal subarachnoid hemorrhage.
Curr Neurol Neurosci Rep. 2012 Feb;12(1):1-​9; used with permission.
Abbreviations: aSAH, aneurysmal subarachnoid hemorrhage; CPP, cerebral perfusion pressure; CSF, cerebrospinal fluid; CT, computed
tomography; DCI, delayed cerebral ischemia; ICP, intracranial pressure; RBC, red blood cell; SAH, subarachnoid hemorrhage; WFNS,
World Federation of Neurological Surgeons

479

Mayo Clinic Neurology Board Review, edited by Kelly D. Flemming, Oxford University Press, Incorporated, 2021. ProQuest Ebook Central, http://ebookcentral.proquest.com/lib/cuhk-ebooks/detail.action?docID=6746419.
Created from cuhk-ebooks on 2023-09-02 09:10:26.
480 Section IV. Neurologic Intensive Care Disorders

Table 58.2 • Hunt and Hess Grading Scale


Grade Criteria Mortality Rate, %

I Asymptomatic, or mild headache 11


and slight nuchal rigidity
II Cranial nerve palsy, moderate to 26
severe headache, nuchal
rigidity
III Mild focal deficit, lethargy, or 37
confusion
IV Stupor, moderate to severe 71
hemiparesis, possibly early
decerebrate rigidity
V Deep coma, decerebrate rigidity 100

Figure 58.1 Terson Syndrome. Retinal subhyaloid and The rupture of an intracranial aneurysm not only can
peripapillary hemorrhage in the hyperacute stage of a cause dramatic neurologic symptoms and signs but also may
patient with increased intracranial pressure and produce serious systemic alterations. Respiratory failure
subarachnoid hemorrhage. from acute neurogenic pulmonary edema, acute heart failure
(Courtesy of Brian R. Younge, MD, Mayo Clinic, Rochester, due to neurocardiogenic injury, and electrocardiographic
Minnesota; used with permission.) abnormalities (eg, prolonged QT interval, deep inverted
T waves, ST-​segment depression or elevation) may all be
Coma at presentation may be a result of the initial increase seen, particularly in patients with poor-​grade hemorrhage.
in ICP with consequently reduced cerebral perfusion that,
if prolonged, could cause diffuse bihemispheric ischemia. • The clinical hallmark of aSAH is thunderclap
Clinical grading scales such as those of the World headache, which has instantaneous onset and reaches
Federation of Neurological Surgeons (WFNS) and Hunt highest severity within seconds.
and Hess that assess the severity of clinical presentation • Initial extensor posturing or myoclonus associated
have been found to most strongly correlate with outcomes with increased intracranial pressure may be mistaken
(Tables 58.1 and 58.2). The WFNS scale is based on the for seizures.
Glasgow Coma Scale sum score and the presence or • Terson syndrome pertains to vitreous hemorrhage and
absence of motor deficits at presentation. The condition of may cause severe or patchy, or both, visual deficits in
patients who have an aSAH is typically dichotomized into patients with subarachnoid hemorrhage.
good grade (WFNS I-​III) or poor grade (WFNS IV or V) on • Restricted up-​gaze or forced downward gaze of
the basis of WFNS score. comatose patients may indicate the presence of
hydrocephalus.
Copyright © 2021. Oxford University Press, Incorporated. All rights reserved.

Table 58.1 • World Federation of Neurological Surgeons Diagnosis


Grading Scale A history of thunderclap headache should alert the clinician
Grade GCS Sum Score Motor Deficit to the possibility of aSAH. Head computed tomography (CT)
without contrast medium should be performed. With
I 15 Absent
advancing CT techniques, recent studies have suggested
II 13–​14 Absent that CT performed within 6 hours of headache onset should
III 13–​14 Present exclude SAH if negative. Some studies further suggest that
IV 7–12 Either absent or present
the combination of CT and CT angiography is 99% sensitive
for ruling out SAH. Beyond this 6-​hour window or when
V <7 Either absent or present
suspicion is high, lumbar puncture should be considered if
Abbreviation: GCS, Glasgow Coma Scale. the initial CT is negative.
Modified from Report of World Federation of Neurological Surgeons When encountering a patient with SAH, a clinician
Committee on a Universal Subarachnoid Hemorrhage Grading Scale. J needs to focus attention on assessing the cause of SAH
Neurosurg. 1988 Jun;68(6):985-6; used with permission.
(Box 58.1). The pattern of SAH can sometimes be helpful in

Mayo Clinic Neurology Board Review, edited by Kelly D. Flemming, Oxford University Press, Incorporated, 2021. ProQuest Ebook Central, http://ebookcentral.proquest.com/lib/cuhk-ebooks/detail.action?docID=6746419.
Created from cuhk-ebooks on 2023-09-02 09:10:26.
Chapter 58. Nontraumatic Subarachnoid Hemorrhage 481

Management
Box 58.1 • Causes of Subarachnoid Hemorrhage
Initial Treatment and Securing the
Aneurysm Aneurysm
Traumatic brain injury
An early goal in management of aSAH is to identify and
Ruptured mycotic (distal) aneurysm
treat the aneurysm early when an aneurysm is present. After
tPA use associated with treatment of STEMI
this step, the goal is to limit secondary brain injury that can
Arteriovenous malformation occur as a result of many complications (Figure 58.3).
Cavernous malformation For people who arrive in the emergency setting with
Vasculitis suspected SAH, basic management principles apply for
Bleeding diathesis stabilization of the patient with attention to airway, breath-
Reversible cerebral vasoconstriction syndrome ing, and circulation (also known as the ABCs). After the
Amyloid angiopathy diagnosis of SAH is secured, blood pressure should be
controlled with a titratable agent and a goal systolic pres-
Abbreviations: STEMI, ST-​segment elevation myocardial sure of less than 160 mm Hg to balance the risk of rebleed-
infarction; tPA, tissue plasminogen activator.
ing and for maintenance of cerebral perfusion pressure.
Antifibrinolytics, such as tranexamic acid and aminoca-
proic acid, reduce the risk of rebleeding by about 40%.
distinguishing aSAH from SAH due to other causes. An Their temporary use (<72 hours) should be considered for
aneurysmal pattern of SAH consists of a diffuse distribution patients at high risk for rebleeding, particularly when any
of blood in the basal cisterns. A more benign radiographic delay occurs in securing the aneurysm.
variant of SAH is the perimesencephalic (around the mesen- The risk of aneurysmal rebleeding—​a feared event with
cephalon), or the pretruncal (around the truncus cerebri or high risk of death or disability—​is highest within 24 hours
brainstem), pattern, which shows a focal hemorrhage ante- of SAH onset. This risk drives the emphasis on early aneu-
rior to the midbrain or upper pons, or both (Figure 58.2). A rysmal treatment. Early conventional angiography is often
cerebral angiogram continues to be necessary to definitively performed to assess for aneurysms. An aneurysm can be
exclude posterior circulation aneurysm in patients with secured with either percutaneous endovascular coiling or
perimesencephalic hemorrhage. Patients with perimesence- microsurgical clipping. If a ruptured aneurysm is deemed
phalic hemorrhage generally have good outcomes. technically amenable to both coiling and clipping, coiling
A lumbar puncture should be considered when suspi- should be considered. When aneurysms are not amenable
cion of SAH is high despite normal CT. The lumbar punc- to either, flow-​ diverting stents and other endovascular
ture may be used to evaluate cerebrospinal fluid (CSF) for options are considered (Figure 58.4).
the presence of xanthochromia, a yellowish discoloration
of CSF caused by bilirubin (a product of red blood cell Hydrocephalus
[RBC] breakdown). One needs to wait at least 6 hours after
Hydrocephalus is a common early complication of SAH,
symptom onset to check reliably for xanthochromia. If CSF
affecting approximately 20% of patients. Ventricular
is obtained within 6 hours, it may yield false-​ negative
enlargement may initially be evident as mild dilatation of
results because of insufficient time for RBC breakdown.
the temporal horns of the lateral ventricles or be readily
Laboratories have 2 methods by which to assess xan-
Copyright © 2021. Oxford University Press, Incorporated. All rights reserved.

apparent in patients who present comatose. Hydrocephalus


thochromia: the more commonly used visual assessment
typically presents clinically as an impairment of con-
of centrifuged CSF (by holding test tubes against a light
sciousness with possible vertical gaze restriction. Acute
source or white paper) and spectrophotometry. Relying
hydrocephalus responds rapidly to ventricular drainage
merely on a decreasing RBC count between tubes 1 and 4
and can be accompanied by striking clinical improvement.
is not sufficient to distinguish SAH from a traumatic lum-
bar puncture. As time passes and CSF recirculates, the
Vasospasm
acute hemorrhage grows more difficult to detect on imag-
ing. CT findings will normalize in up to 10% of patients by Vasospasm of cerebral arteries after aSAH rupture is com-
day 3 and in about 50% of patients by day 7. If the diagno- mon, with a peak occurrence at 7 to 10 days after rupture
sis of SAH is strongly suspected clinically, additional test- and resolving spontaneously after 21 days. Patients must
ing is needed if the head CT is unrevealing. be monitored for delayed cerebral ischemia (DCI) while
receiving prophylaxis for vasospasm. Nimodipine, a cal-
• For patients with suspected aSAH and negative head cium channel blocker, has been shown to improve neuro-
CT, lumbar puncture should be performed to assess for logic outcome after aSAH but not after cerebral vasospasm.
xanthochromia at least 6 hours after symptom onset. Angiographic vasospasm is seen in about two-​ thirds of

Mayo Clinic Neurology Board Review, edited by Kelly D. Flemming, Oxford University Press, Incorporated, 2021. ProQuest Ebook Central, http://ebookcentral.proquest.com/lib/cuhk-ebooks/detail.action?docID=6746419.
Created from cuhk-ebooks on 2023-09-02 09:10:26.
482 Section IV. Neurologic Intensive Care Disorders

A B

Figure 58.2 Patterns of Subarachnoid Hemorrhage. A, Subarachnoid blood fills cisterns and fissures. Of note, dilatation of
the temporal horns of the lateral ventricles is shown, indicating early hydrocephalus. B, Focal area of subarachnoid blood
(arrow) anterior to the pons in a pretruncal or perimesencephalic pattern, which is a benign form of subarachnoid
hemorrhage, usually associated with a good outcome.
(From Flemming KD. Principles of critical care neurology. In: Mowzoon N, Flemming KD, editors. Neurology board review: an illustrated
study guide. Rochester [MN]: Mayo Clinic Scientific Press and Florence [KY]: Informa Healthcare USA; c2007. p. 401-​34; used with
permission of Mayo Foundation for Medical Education and Research.)

patients with aSAH, but only one-​half of these patients are Maintenance of euvolemia is recommended to prevent
symptomatic. DCI, but prophylactic hypervolemia or balloon angioplasty
The best independent predictor for the development of before the development of angiographic spasm is not rec-
vasospasm is the total volume of subarachnoid blood on ommended. After a patient has DCI, induced hypertension
Copyright © 2021. Oxford University Press, Incorporated. All rights reserved.

the initial head CT. Patients with poor clinical grade, intra- is recommended unless baseline hypertension is present
ventricular hemorrhage, and a history of cigarette smoking or the patient’s cardiac status precludes it. For deficits that
tend to have more severe vasospasm. DCI occurs in up to do not improve with hemodynamic augmentation, endo-
33% of patients and is characterized with neurologic defi- vascular treatments may be considered, such as angio-
cits that appear gradually over hours. These deficits may plasty or intra-​arterial vasodilator therapy.
manifest as focal deficits (25%) or a decreased level of con-
sciousness (25%), or both (50%). Transcranial Doppler Intracranial Hypertension
ultrasonography may be used for vasospasm surveillance,
Intracranial hypertension in comatose patients can be
although it is limited to visualizing only the large cerebral
caused by hydrocephalus, global edema, or space-​occupying
vessels.
hemorrhage. For patients with poor clinical grades, ICP and
Conventional cerebral angiography is the gold standard
cerebral perfusion pressure (CPP) (CPP = mean arterial
for the diagnosis of arterial vasospasm, but the procedure
pressure − ICP) should be monitored to maintain CPP
is costly and invasive. The routine use of serial conven-
greater than 60 mm Hg. Patients with a space-​occupying
tional angiography to screen for vasospasm after aSAH is
hemorrhage may be considered for craniotomy and surgical
not recommended.

Mayo Clinic Neurology Board Review, edited by Kelly D. Flemming, Oxford University Press, Incorporated, 2021. ProQuest Ebook Central, http://ebookcentral.proquest.com/lib/cuhk-ebooks/detail.action?docID=6746419.
Created from cuhk-ebooks on 2023-09-02 09:10:26.
Chapter 58. Nontraumatic Subarachnoid Hemorrhage 483

Subarachnoid hemorrhage

SBP Early EVD if Antifibrinolytics


ABCs
<160 mm Hg needed as indicated

Assess for and


secure aneurysm

Monitor for
complications

Cardiorespiratory
Seizure Vasospasm Hydrocephalus Hyponatremia DVT or PE
complications

Figure 58.3 Overview of Management of Subarachnoid Hemorrhage. ABC indicates airway, breathing, and circulation; DVT,
deep vein thrombosis; EVD, external ventricular drain; PE, pulmonary embolism; SBD, systolic blood pressure.

B
Copyright © 2021. Oxford University Press, Incorporated. All rights reserved.

Figure 58.4 Aneurysmal Endovascular Treatment on Angiography. A, Aneurysm of the posterior communicating segment
(arrow). B, Aneurysm treated with a flow-​diverting stent. Left, Before treatment. Right, After stent placement.
(Courtesy of Giuseppe Lanzino, MD, Mayo Clinic, Rochester, Minnesota; used with permission.)

Mayo Clinic Neurology Board Review, edited by Kelly D. Flemming, Oxford University Press, Incorporated, 2021. ProQuest Ebook Central, http://ebookcentral.proquest.com/lib/cuhk-ebooks/detail.action?docID=6746419.
Created from cuhk-ebooks on 2023-09-02 09:10:26.
484 Section IV. Neurologic Intensive Care Disorders

decompression if clinical deterioration or refractory intra-


Table 58.3 • Complications of Aneurysmal Subarachnoid
cranial hypertension develops. An increased ICP may be
Hemorrhage and Their Management
managed with head-​of-​bed elevation, hyperosmolar ther-
apy, normothermia, and avoidance of hyponatremia. Complication Management

Seizures Antiseizure medication


Hyponatremia No prophylaxis unless seizure
Phenytoin should be avoided when
Hyponatremia due to cerebral salt wasting occurs in
possible because of worse reported
approximately 20% to 40% of aSAH patients. The use of neurologic outcomes with its use
fludrocortisone acetate and hypertonic saline is reasonable
Hyperthermia Aggressively target normothermia
for prevention and correction of hyponatremia.
Glucose Strict avoidance of hypoglycemia (serum
management glucose <80 mg/​dL)
Other Complications
Maintain serum glucose <200 mg/​dL
The management of other neurologic and systemic compli- Neurogenic Avoid volume overload
cations of aSAH is outlined in Table 58.3. Clinical seizures pulmonary Use higher levels of positive end-​
are uncommon after initial aneurysmal rupture, occurring edema expiratory pressure
in 1% to 7% of patients. Identified risk factors for develop- Stress ECG monitoring for T-​wave inversions,
ment of early seizures include aneurysm in the middle cere- cardiomyopathy ST-​segment depression or elevation,
bral artery, thickness of an aSAH clot, associated intracerebral due to prolonged QT, increased troponin
catecholamine
hemorrhage, rebleeding, infarction, poor neurologic grade,
surge
and history of hypertension. Seizure prophylaxis may be
Anemia Transfuse packed red blood cells for
used in the immediate posthemorrhagic period, although its
hemoglobin ≤8 g/​dL
use has weak supportive evidence.
Deep venous Heparin or low-​molecular-​weight heparin
thrombosis after aneurysm is secured
• Early complications of aSAH are rebleeding and
hydrocephalus, and delayed complications include Abbreviation: ECG, electrocardiography.
vasospasm and cerebral salt wasting.
• Hypervolemia should be avoided in patients
admission recover without major cognitive or physical defi-
with aSAH.
cits. The chance of a patient surviving aSAH has increased
• Hemodynamic augmentation for patients with
by nearly 20% over the past 3 decades, which may be due to
symptomatic vasospasm may be indicated, as well as
the emergence of dedicated neurologic-​neurosurgical inten-
endovascular treatment of cases refractory to
sive care units, emphasis on early aneurysmal treatment,
hemodynamic augmentation.
and advances in endovascular therapies. The percentage of
• Fludrocortisone and hypertonic saline may be
patients who regain independence within the first year after
indicated for patients with hyponatremia due to
aSAH is estimated to be 30% to 50%. Some survivors of
cerebral salt wasting.
aSAH may have cognitive deficits, particularly with execu-
• Seizures are uncommon in patients with aSAH.
tive function and memory, which can lead to difficulties
with activities of daily living and a poorer quality of life.
Copyright © 2021. Oxford University Press, Incorporated. All rights reserved.

Prognosis • Of patients who are comatose on initial presentation


after aSAH, 20% have good functional outcomes.
One of the most important predictors of clinical outcome is
the patient’s clinical condition at the time of hospital admis-
• Within the first year after aSAH, 30% to 50% of
patients regain independence.
sion. Of importance, 20% of patients who are comatose at

Mayo Clinic Neurology Board Review, edited by Kelly D. Flemming, Oxford University Press, Incorporated, 2021. ProQuest Ebook Central, http://ebookcentral.proquest.com/lib/cuhk-ebooks/detail.action?docID=6746419.
Created from cuhk-ebooks on 2023-09-02 09:10:26.
Anoxic-​Ischemic Encephalopathya
59 TIA CHAKRABORTY, MD; JENNIFER E. FUGATE, DO

Introduction occurs, characterized by excess free radical formation,


nitric oxide toxicity, and further glutamate release, leading
Anoxic-​ ischemic brain injury occurs when no blood is to worsening brain edema and microhemorrhages.
flowing to the brain. Neurologists commonly encounter Gross examination of the brain that was damaged by
this clinical state when evaluating comatose patients who anoxic-​ischemic injury may show diffuse cerebral edema
have had a cardiac arrest and prolonged cardiopulmonary with loss of gray matter–​white matter differentiation, corti-
resuscitation attempts. Anoxic-​ischemic injury may also cal laminar necrosis, watershed infarctions, and hippo-
occur in primary respiratory arrest or severe hypoxemia campal sclerosis in the subacute or chronic stages.
(eg, asphyxia, anaphylaxis, drug intoxication), but it is less Microscopically, neuronal loss and gliosis may be seen in
well understood in these circumstances. This chapter vulnerable areas such as the CA1 region of the hippocam-
reviews the pathophysiologic factors, clinical manage- pus, basal ganglia, thalami, cerebellar Purkinje cells, and
ment, and prognostic factors in anoxic-​ ischemic brain cortex. Pyknotic nuclei and eosinophilic cytoplasm (so-​
injury. called red, dead neurons) are early pathologic characteris-
tics of ischemic neurons (Figure 59.1).

Pathophysiologic Factors • Pathologic changes following anoxic-​ischemic injury


may include loss of gray matter–​white matter
The brain is very susceptible to ischemic damage because differentiation, watershed infarcts, microhemorrhages,
of its high metabolic demand. Animal studies have shown and hippocampal sclerosis.
that brain concentrations of glucose, adenosine triphos-
Copyright © 2021. Oxford University Press, Incorporated. All rights reserved.

phate, and phosphocreatine decrease almost immediately


after the cessation of cerebral blood flow and are nearly
depleted within 10 minutes, leading to cell death. In addi- Clinical Manifestations
tion, the excitotoxic neurotransmitter glutamate is released The clinical manifestations of anoxic-​ischemic encepha-
and destructive lipases, proteases, and nucleases are acti- lopathy are heterogeneous and generally correlate with the
vated, which cause further tissue destruction. duration of the ischemic event. With brief episodes of car-
Even after blood flow has been restored successfully diac arrest, patients may present with a syncopal episode
after ischemia, brain circulation continues to be impaired. only, which can be associated with clonic movements
Transient hyperemia is quickly followed by more persis- (called convulsive syncope) and may be mistaken for sei-
tent hypoperfusion in the microcirculation (the so-​called zure activity. In the most severe cases, patients continue to
no-​reflow phenomenon). Furthermore, reperfusion injury be comatose for days or weeks and never regain meaningful

a
Portions previously published in Fugate JE, Wijdicks EFM. Anoxic-​ischemic encephalopathy. In: Daroff RB, Jankovic J, Mazziotta JC,
Pomeroy SL, editors. Bradley’s neurology in clinical practice. 7th ed. (London; NY): Elsevier; c2016. p. 1201–​8.e2 and Fugate JE. Anoxic-​
ischemic brain injury. Neurol Clin. 2017 Nov;35(4):601-​11; used with permission.
Abbreviations: EEG, electroencephalography; NSE, neuron-​specific enolase; SSEP, somatosensory-​evoked potential

485

Mayo Clinic Neurology Board Review, edited by Kelly D. Flemming, Oxford University Press, Incorporated, 2021. ProQuest Ebook Central, http://ebookcentral.proquest.com/lib/cuhk-ebooks/detail.action?docID=6746419.
Created from cuhk-ebooks on 2023-09-02 09:10:26.
486 Section IV. Neurologic Intensive Care Disorders

consciousness, transitioning to a persistent vegetative state


A or a minimally conscious state.
The clinical examination is focused on brainstem
reflexes, the presence of generalized myoclonus, and
motor responses to noxious stimuli (Table 59.1).
An important clinical sign is status myoclonus, defined
as spontaneous, vigorous, and continuous multifocal jerk-
ing, which may be an ominous sign. This motion should
be differentiated from the occasional myoclonic jerk that
does not have prognostic importance. Previous studies
have associated myoclonic status with poor outcome,
although cases of favorable outcomes despite myoclonic
status have been described more recently.
Watershed syndromes, including man-​in-​the-​barrel
syndrome (bilateral watershed infarct between anterior
B and middle cerebral arteries) and Balint syndrome (water-
shed region between middle and posterior cerebral arterial
territories), may confer better prognoses. Movement disor-
ders due to basal ganglia and cerebellar injury and cogni-
tive impairment due to hippocampal susceptibility to
injury are included.

• Brainstem reflexes, the presence of myoclonus, and


motor response to noxious stimuli are important
neurologic prognosticators after cardiac arrest.

Clinical Management
After cardiac resuscitation and the acute management of
C cardiovascular collapse with blood pressure optimization,

Table 59.1 • Clinical Findings and Manifestations of


Anoxic-​Ischemic Encephalopathy
Finding Manifestation
Copyright © 2021. Oxford University Press, Incorporated. All rights reserved.

Brainstem reflexes Often normal because the cortex is


more vulnerable to anoxic injury
Motor response Variable depending on severity
(extensor posturing has poor
prognosis)
Figure 59.1 Anoxic Brain Injury. A, Gross autopsy specimen Status myoclonus may have poor
with diffuse global edema of the brain after cardiac arrest. prognosis
Microscopic examination showed (B) laminar necrosis of Movement disorders Lance-​Adams syndrome (delayed-​
the cerebral cortex and (C) ischemic neurons in the CA1 onset stimulus-​induced action
myoclonus)
sector of the hippocampus. Ischemic hippocampal neurons
Parkinsonism
have pyknotic nuclei and densely eosinophilic cytoplasm. Dystonia
Most of these neurons have nuclei that are more eosinophilic Cerebellar outflow tremor
and appear to be assimilated into the background of
Cognitive impairment Amnestic syndromes
eosinophilic cytoplasm.

Mayo Clinic Neurology Board Review, edited by Kelly D. Flemming, Oxford University Press, Incorporated, 2021. ProQuest Ebook Central, http://ebookcentral.proquest.com/lib/cuhk-ebooks/detail.action?docID=6746419.
Created from cuhk-ebooks on 2023-09-02 09:10:26.
Chapter 59. Anoxic-Ischemic Encephalopathy 487

the use of targeted temperature management or therapeutic


Table 59.2 • Factors Associated With Poor Outcome for
hypothermia is recommended, particularly for patients
Patients With Anoxic-​Ischemic
who cannot follow basic commands or perform purposeful
Encephalopathy
movements. Induced hypothermia inhibits apoptosis and
reduces free radical formation and release of excitatory Characteristic Related to Poor
neurotransmitters and has been shown to reduce the mor- Factor Outcome
tality rate and improve neurologic outcomes.
Pupillary light reflex Absent bilaterally at ≥72 h
Recent trials have criticized the benefit of induced
Corneal reflex Absent bilaterally at ≥72 h
hypothermia. However, early fever has been associated
with worse outcomes after cardiac arrest. A target tempera- Motor response to Extensor or absent <72 h without TH
ture of 32°C to 36°C as soon as possible for 12 to 24 hours pain or TTM
and avoidance of fever are recommended to minimize neu- Status myoclonus Present early (<48 h) with epileptiform
rologic damage. This approach has become the standard of EEG
care for patients with out-​of-​hospital cardiac arrests pre- Highly malignant Generalized suppression
senting with shockable rhythms (ventricular fibrillation or EEG pattern Suppression with periodic discharges
Burst suppression
pulseless ventricular tachycardia). Hypothermia treatment
of patients with cardiac arrests resulting from nonshock- ≥2 malignant EEG Periodic or rhythmic patterns,
able rhythms (asystole or pulseless electrical activity) may patterns including electrographic seizures
Discontinuous or low-​voltage
also be beneficial.
background
Nonreactive EEG
• Instituting targeted temperature management as soon Imaging Head CT: loss of gray matter–​white
as possible after cardiac arrest, particularly for patients matter differentiation
with shockable rhythms who had out-​of-​hospital Brain MRI: widespread cortical
cardiac arrest, may reduce death and improve restricted diffusion
neurologic outcome. SSEP Bilateral absent N20 responses with
normothermia at 24-​72 h
Serum NSE Concentration >80 μg/​L within 72 h
Prognostication Abbreviations: CT, computed tomography; EEG, electroencephalography;
MRI, magnetic resonance imaging; NSE, neuron-​specific enolase;
The neurologic examination is crucial in determining the SSEP, somatosensory-​evoked potential; TH, therapeutic hypothermia;
prognosis. Yet, an examination within 72 hours in an TTM, targeted temperature management.
intensive care unit can be confounded, when multisystem
organ failure, metabolic abnormalities, and lingering
effects of sedating medications are common. For patients
not treated with hypothermia, the prognosis is poor if the In addition to imaging, neurophysiologic studies such as
motor response to pain is extensor posturing or absent at electroencephalography (EEG) and somatosensory-​evoked
day 3 after resuscitation. For those with induced hypother- potentials (SSEPs) are often used for prognosis (Table 59.2).
mia, this motor response at day 3 is not as reliable for prog- These include malignant patterns on EEG. With the increas-
nostication, potentially because of greater amounts of ing use of therapeutic hypothermia (which requires seda-
Copyright © 2021. Oxford University Press, Incorporated. All rights reserved.

sedatives and analgesics that accompany hypothermia tion and neuromuscular blockade), interest has grown in
­protocols. continuous EEG monitoring during cooling. Nevertheless, it
Several diagnostic tests may be useful as supplements is not clear whether this costly test provides additional
for the neurologic examination (Table 59.2). Neuroimaging information that cannot be gained with shorter electrocar-
is potentially useful for prognosis in anoxic-​ ischemic diograms. Although longer monitoring almost certainly
injury. However, literature is sparse and is limited by rela- increases detection of epileptiform activity, no evidence
tively small sample sizes. Magnetic resonance imaging can shows that earlier detection and treatment of seizures can
document the extent of anoxic-​ ischemic injury with alter the outcome in this clinical setting.
diffusion-​weighted sequences (Figure 59.2). Quantitative SSEPs are not influenced by drugs, temperature, or
diffusion-​weighted imaging and whole-​brain apparent dif- acute metabolic derangements. They often are obtained for
fusion coefficient values have been shown to correlate a comatose patient after cardiac resuscitation between
with outcomes. Computed tomographic studies are often days 1 and 3. SSEP requires stimulation of the median
normal but may show cerebral edema in more extreme nerve that then results in a potential at the brachial plexus
cases. Effacement of the gray matter–​white matter junction and the cervical spinal cord and, finally, the bilateral cor-
is associated with failure to awaken. tex potentials (N20). The absence of both N20s implies an

Mayo Clinic Neurology Board Review, edited by Kelly D. Flemming, Oxford University Press, Incorporated, 2021. ProQuest Ebook Central, http://ebookcentral.proquest.com/lib/cuhk-ebooks/detail.action?docID=6746419.
Created from cuhk-ebooks on 2023-09-02 09:10:26.
488 Section IV. Neurologic Intensive Care Disorders

A B

Figure 59.2 Neuroimaging in Anoxic-​Ischemic Encephalopathy. Diffusion-​weighted (A) and apparent diffusion coefficient
(B) magnetic resonance imaging sequences show diffuse restricted diffusion affecting the entire cortex, reflecting cortical
laminar necrosis, of a patient with anoxic injury after cardiac arrest.

invariably poor prognosis, and the patient likely will never outcome compared with the use of clinical information
regain consciousness. alone and had higher accuracy than serum NSE.
Investigators are searching for a serum biomarker that
could indicate the degree of brain injury and potentially • Extensor posturing, no motor response, or absent
assist in prognostication for comatose survivors. The most pupillary or corneal reflexes at day 3 may serve as
studied biomarkers have been serum neuron-​specific eno- examination findings that imply poor neurologic
lase (NSE) and S100. A serum NSE level greater than prognosis if confounding factors are eliminated.
33 μ/​L at day 1 through day 3 after cardiac arrest was tra- • Neuroimaging may show restricted diffusion on
ditionally a predictor of poor outcome. However, several diffusion-weighted and apparent diffusion coefficient
studies have confirmed that this cutoff level for NSE is not magnetic resonance imaging sequences, and head CT
reliable for patients who have undergone hypothermia may show cerebral edema in anoxic injury.
protocols. Measuring serial NSE levels for these patients • The absence of bilateral N20 potentials with SSEPs
and following their trends may be more informative than a portends a poor neurologic prognosis.
Copyright © 2021. Oxford University Press, Incorporated. All rights reserved.

single isolated value, but this has not been sufficiently • Malignant EEG patterns include generalized
studied to make firm recommendations. suppression, suppression with periodic discharges, or
Recent interest also exists in serum tau as a predictor in burst suppression, with rhythmic patterns of discharges,
cardiac arrest. A subanalysis from a large randomized a low-​voltage background, or nonreactive EEG pattern
study showed that serum tau improved prediction of poor may be poor electrographic prognosticators.

Mayo Clinic Neurology Board Review, edited by Kelly D. Flemming, Oxford University Press, Incorporated, 2021. ProQuest Ebook Central, http://ebookcentral.proquest.com/lib/cuhk-ebooks/detail.action?docID=6746419.
Created from cuhk-ebooks on 2023-09-02 09:10:26.
Traumatic Brain Injury
60 LUCAS P. CARLSTROM, MD, PHD; EELCO F. M. WIJDICKS, MD, PHD

Introduction outcome after TBI, even after adjustment for potential con-
founders.

T
raumatic brain injury (TBI) continues to be a lead-
ing cause of long-​term morbidity and death world- Diagnostic Approach to Patients With
wide. Each year, an estimated 1.7 million persons Head Trauma
in the United States sustain TBIs, leading to 275,000 hos-
Universal implementation of advanced trauma life support
pitalizations and 52,000 deaths annually. In addition to
protocols has helped standardize trauma care and improve
high personal costs, the direct and indirect societal expen-
prehospital and primary evaluation and patient outcome.
ditures are estimated to be $60 billion per year.
Rapid stabilization and transfer of a patient to a trauma
This chapter reviews the diagnostic approach to head
care facility are critical to improve chances of survival.
trauma and prognosis in brain injury and addresses spe-
Initial management is based on advanced trauma life sup-
cific conditions, such as concussions and intracranial
port guidelines from the American College of Surgeons
hemorrhages.
and includes airway (create or maintain airway with neu-
tral cervical spine precautions), breathing (assess oxygen
saturation and evaluate and treat chest injuries), circula-
Traumatic Brain Injury tion, disability (neurologic examination to assess for brain
injury), and environmental exposure to cold (assess tem-
Epidemiologic Factors
perature and achieve normothermia).
Improved utilization rates of seat belts in cars and improved Patients also should be evaluated for external signs of
helmet laws have reduced the number of serious injuries in basal skull fractures, such as periorbital ecchymoses (rac-
Copyright © 2021. Oxford University Press, Incorporated. All rights reserved.

developed countries. At the same time, the incidence of coon eyes), postauricular ecchymoses (Battle sign), cere-
TBI has increased in developing countries. TBI has a brospinal fluid (CSF) rhinorrhea or otorrhea, facial
bimodal incidence distribution across the age spectrum, fractures, and physical signs of spine trauma. Patients
with increased rates in children and adolescents and the should be immobilized in a cervical collar, and evalua-
elderly persons. Most TBIs are attributed to falls (elderly tions should follow proper spine precautions until appro-
persons), motor vehicle collisions (youth), and blunt force priate trauma evaluation is completed to determine the
trauma. In the military, blast injury is most common (eg, status of the cervical, thoracic, and lumbar spine. Clinical
improvised explosive devices). Age and injury severity are assessment should include serial Glasgow Coma Scale
recognized generally as the strongest predictors of health (GCS) examinations. Although not a perfect clinical

The editors and authors acknowledge the contributions of Jeffrey T. Jacob, MD, to the previous edition of this chapter.
Abbreviations: CSF, cerebrospinal fluid; CT, computed tomography; CTE, chronic traumatic encephalopathy; DAI, diffuse axonal injury;
EDH, epidural hematoma; GCS, Glasgow Coma Scale; ICH, intracerebral hemorrhage; ICP, intracranial pressure; LOC, loss of conscious-
ness; MRI, magnetic resonance imaging; SAH, subarachnoid hemorrhage; SDH, subdural hemorrhage; TBI, traumatic brain injury; tICH,
traumatic intracerebral hemorrhage

489

Mayo Clinic Neurology Board Review, edited by Kelly D. Flemming, Oxford University Press, Incorporated, 2021. ProQuest Ebook Central, http://ebookcentral.proquest.com/lib/cuhk-ebooks/detail.action?docID=6746419.
Created from cuhk-ebooks on 2023-09-02 09:10:26.
490 Section IV. Neurologic Intensive Care Disorders

correlate, GCS provides a preliminary predictive evalua- head elevation to promote venous outflow, h ­ yperventilation
tion and assists with patient stratification. The TBI can be to induce vasoconstriction and therefore reduced intracra-
considered as mild (GCS score 14 or 15 plus brief loss of nial volume, and sedation or use of paralytics to lower oxy-
consciousness [LOC] or impairment of mentation), moder- gen demand. Hyperventilation should be used with caution
ate (GCS score 9-​13, LOC >5 min, or focal neurologic defi- beyond acute temporization for management of transtento-
cit), or severe (GCS score <9). rial herniation or worsening neurologic decline, because it
Radiographic imaging should be done on the basis of a can predispose to cerebral ischemia and poor neurologic
patient’s mechanism of injury or clinical condition and outcomes. More targeted measures may include hyperosmo-
should include non–​contrast medium computed tomogra- lar therapy with hypertonic saline and mannitol (see Chapter
phy (CT) of the head and spinal radiography. Brain mag- 56 “Principles and Management of Increased Intracranial
netic resonance imaging (MRI) is often not appropriate in Pressure” for further detail). Acute corticosteroid treatments,
the acute setting but may be helpful for prognostication similar to management of spinal cord injuries, have not been
after the initial acute time window. One study identified shown to consistently improve outcomes or reduce ICP in
no new lesions on MRI that had not already been detected trauma. Preliminary evidence has suggested that corticoste-
with head CT and required operative intervention. MRI is roids may have some benefit. However, the large multicenter
also less sensitive than CT for acute blood visualization. randomized controlled Corticosteroid Randomization After
Vascular imaging, such as CT angiography or digital Significant Head Injury trial was stopped early because of
subtraction angiography, may be helpful to rule out under- worse outcomes in the treated cohort.
lying vascular pathologic characteristics. This applies par- Neurosurgical intervention is largely guided by imag-
ticularly for penetrating trauma or suspected arterial ing and clinical examination results. In many cases, serial
dissection. imaging may be valuable to accurately assess evolving
All patients should undergo basic hematologic and contusions, intracranial hemorrhages, or cerebral edema
electrolyte profile testing, specifically to assess hemoglo- and determine the timing and discrete operative interven-
bin, platelet count, and serum sodium values. Additionally, tion that may be warranted. Decompressive surgery is
laboratory evaluation should include serum prothrombin often a definitive treatment of increasing ICP, and gener-
time (international normalized ratio) and activated partial ally the threshold should be low when indicated for appro-
thromboplastin time. Establishment of whether a patient priate patient populations.
takes an anticoagulant (vitamin K or a non–​ vitamin K
antagonist) is crucial because emergent reversal is impera- Prognosis After TBI
tive in prevention of intracranial hemorrhage expansion.
Clinical and radiographic tools have been developed to
Antiplatelet agents may affect the stability of acute hemor-
assess prognosis after TBI. Examples of these tools are
rhage. However, exogenous platelets are of limited use for
GCS, abbreviated injury scale, Marshall CT score, and
patients who do not have thrombocytopenia. A toxicology
Rotterdam CT score. Consistent factors that have been pre-
screen and known drug screen can be helpful as well.
dictive of poor outcome include increased ICP, increased
Neurosurgical consultation should be obtained to assist
age, and hypotension.
appropriate triage of medical or surgical interventions, or
both. It may include intracranial pressure (ICP) and tissue
• Studies have shown that increased intracranial
oxygen monitoring devices.
pressure (ICP), increased age, and hypotension are
among factors that portend an unfavorable outcome.
Copyright © 2021. Oxford University Press, Incorporated. All rights reserved.

Treatment of Head Trauma


Age is generally recognized as the strongest predictor
The primary goals of TBI interventions, as with other of outcome after traumatic brain injury, even after
trauma forms, are prevention and minimization of second- adjustment for potential confounders.
ary insult related to sequelae of the initial injury. At the • Rapid hemodynamic stabilizing measures, followed by
first medical assessment, the primary injury has already appropriate radiographic and clinical evaluations, are
occurred. Yet, secondary injury often results in sequelae essential to etiologic identification and management.
that are highly morbid and occasionally fatal and require • Failure to aggressively control ICP is common and,
timely identification and intervention. particularly in the first hours after traumatic brain
Clinical decision-​
making depends on the traumatic injury, could lead to irreversible early secondary
mechanism and specific intracranial pathologic factors. It brainstem and cortical injury.
centers on hemodynamic ICP and hematologic monitoring • Temporizing measures for lowering ICP may include
and regulation. Failure to aggressively control ICP—​often to reduction of intracranial blood volume through head
less than 20 mm Hg—​in the first hours could lead to early elevation and hyperventilation, utilization of
secondary injury of the brainstem and cortical area. hyperosmolar therapy, and occasionally sedation or
Fundamental temporizing measures to decrease ICP include paralytics. Corticosteroids do not improve outcome or

Mayo Clinic Neurology Board Review, edited by Kelly D. Flemming, Oxford University Press, Incorporated, 2021. ProQuest Ebook Central, http://ebookcentral.proquest.com/lib/cuhk-ebooks/detail.action?docID=6746419.
Created from cuhk-ebooks on 2023-09-02 09:10:26.
Chapter 60. Traumatic Brain Injury 491

reduce ICP in acute trauma and may lead to worse baseline with reassurance and symptomatic treatment after
outcomes. 1 to 2 weeks. In the emergent setting, identification is impor-
tant of patients who require head imaging. Unnecessary
imaging can subject patients to needless radiation and risk,
Specific Conditions of TBI identifying incidental findings that can mandate further
evaluation and management but often without clear benefit.
Concussion The New Orleans Criteria (Box 60.1) is highly sensitive
and specific for identifying patients with concussion or
Definition with mild TBI who may have clinically important intracra-
Concussion is a subset of mild TBI (GCS score 14 or 15). nial lesions. This model includes 7 variables. If all 7 vari-
Multiple definitions of the term concussion have been put ables are absent, CT is not needed. Absence of all 7 has a
forth by various medical societies and groups. Importantly, sensitivity of 100% for detecting intracranial injury, defined
new definitions do not require LOC as was the case in the as epidural hematoma, subdural hematoma, depressed
past. In addition, a direct head blow is not required for a skull fracture, cerebral contusion, and subarachnoid hem-
concussion if an injury to the body transmits an impulsive orrhage (SAH). However, if 1 factor is present, the specific-
force to the head. ity is only 24%. The Canadian Head CT Rule also may be
The Quality Standards Subcommittee of the American useful for emergency departments, to help guide clinical
Academy of Neurology defines TBI concussion as “trauma-​ decision-​making. The rule consists of such factors as GCS,
induced alteration in mental status that may or may not age, suspected calvarial fractures, emesis, amnesia, and
involve loss of consciousness.” The American Congress of trauma mechanism.
Rehabilitation defines concussion as a trauma-​ induced Patients who are candidates for a concussion diagnosis
disruption of brain function that involves 1 or more of the may be discharged from the emergency department or clinic
following characteristics: any period of LOC, any loss of setting if they have low clinical risk with no findings on
memory for events immediately before and after accidents, radiographic evaluation and have close observational sup-
alteration in mental status (eg, dazed, disoriented), or focal port. A patient may warrant at least overnight observation
neurologic deficit (persistent or transient). However, the in the hospital if they continue to be neurologically altered
severity does not exceed LOC for 30 minutes or less or, or have positive radiographic findings or if additional com-
after 30 minutes of an initial GCS score of 13 to 15, post- plications arise or a reliable observer is not available.
traumatic amnesia lasting 24 hours or less.
Concussions must have negative structural findings on • A concussion is a trauma-​induced alteration in mental
head CT. Repeated concussions can result in some degree status that may or may not involve loss of
of permanent neurologic impairment. consciousness.
• If all 7 items of the New Orleans Criteria (Box 60.1) are
Symptoms absent, a head CT is often not clinically indicated.
Acutely, persons with a concussion may have headaches,
nausea or vomiting, imbalance and dyscoordination, and a
lack of awareness of their surroundings. Over hours to
days, they also may have mood and cognitive disturbances,
phonophobia, photophobia, and insomnia. Approximately
Copyright © 2021. Oxford University Press, Incorporated. All rights reserved.

10% to 15% of persons with a concussion have symptoms


beyond the first week, a clinical situation referred to as Box 60.1 • Clinical Characteristics in the New
postconcussion syndrome. Symptoms of postconcussion Orleans Criteriaa
syndrome include headache, fatigue, insomnia, dizziness,
Headache
and cognitive and emotional changes. Risk factors for post-
Vomiting
concussion syndrome include age older than 40 years,
female sex, and prior history of concussion. The severity of Age >60 years
the injury is generally not a risk factor for postconcussion Drug or alcohol intoxication
syndrome. Up to 90% of patients with postconcussion syn- Persistent anterograde amnesia (deficits in short-​term
drome improve within 1 year. memory)
Evidence of traumatic soft-​tissue or bone injury above
clavicles
Diagnosis and Management
The nonspecific nature of these symptoms often makes the Seizure (suspected or witnessed)
diagnosis of concussion difficult and perhaps controversial. a
If all 7 characteristics are absent, head computed tomography
Symptomatic therapy is advocated for postconcussion is not required.
symptoms, and the parameters of most patients return to

Mayo Clinic Neurology Board Review, edited by Kelly D. Flemming, Oxford University Press, Incorporated, 2021. ProQuest Ebook Central, http://ebookcentral.proquest.com/lib/cuhk-ebooks/detail.action?docID=6746419.
Created from cuhk-ebooks on 2023-09-02 09:10:26.
492 Section IV. Neurologic Intensive Care Disorders

Skull Fractures Location and Potential Symptoms


Skull fractures may result in various symptoms depending
Definition
on location (Figure 60.2). Basilar skull fractures most com-
Skull fractures are seen radiographically as linear fracture
monly involve the petrous portion of the temporal bone,
lines with distinct margins (Figure 60.1). Fractures may be
anterior cranial fossa, or posterior or lateral skull base.
considered closed or open depending on whether the epi-
When violation of aerated sinuses in the skull occurs along
dermal and dermal layers are violated. If these layers are
with dural lacerations, patients may have CSF otorrhea or
fragmented, the fracture is termed comminuted.
rhinorrhea. Avulsion of olfactory fibrils from the cribriform
True fractures often are difficult to radiographically dis-
plate by the shearing forces of a blunt impact rarely causes
tinguish from suture lines and vascular grooves. Unlike
rhinorrhea in the absence of a fracture. A fracture of the
vascular markings, fracture lines generally are straight, do
posterior wall of the frontal sinus may allow CSF to track
not taper or branch, and are thin. Fractures often appear
through the frontonasal duct. Fractures through the petrous
darker than vessel grooves on plain radiographs. Suture
bone that extend to the middle ear may lead to either otor-
lines are wide and jagged and join with other, similar-​
rhea if the tympanic membrane is torn or otorhinorrhea if
appearing suture lines.
leakage occurs via the eustachian tube into the nasophar-
Skull fractures may result in tearing of the underlying
ynx. CSF can be assayed for β2-​transferrin, which is unique
dura and brain parenchyma. This outcome causes extra-​
to CSF and the vitreous of the eye.
axial hemorrhages (eg, epidural, subdural) or intra-​axial
hemorrhages (eg, focal contusions, traumatic SAH).
Management
Depressed skull fractures often require operative interven-
Prophylactic antibiotics are no longer recommended for
tion if they are open, comminuted, or associated with
patients with CSF leaks that do not communicate with the
underlying intracranial hemorrhage or disruption of the
external environment. Closed nondisplaced fractures can
dura, or both. Operative intervention is warranted early to
be managed conservatively. Open depressed fractures gen-
reduce the risk of infection, and patients should continue
erally require open elevation of bone fragments, débride-
to receive appropriate empirical antibiotic coverage.
ment of jagged skin edges and injured brain parenchyma,

A B
Copyright © 2021. Oxford University Press, Incorporated. All rights reserved.

Figure 60.1 Skull Fracture on Computed Tomography. A, Anteriorly displaced fracture (arrows) of the frontal bone with
extension superiorly into the right coronal sutures. B, Anteriorly displaced fracture (arrows) of the frontal bone with
extension superiorly into sagittal sutures.

Mayo Clinic Neurology Board Review, edited by Kelly D. Flemming, Oxford University Press, Incorporated, 2021. ProQuest Ebook Central, http://ebookcentral.proquest.com/lib/cuhk-ebooks/detail.action?docID=6746419.
Created from cuhk-ebooks on 2023-09-02 09:10:26.
Chapter 60. Traumatic Brain Injury 493

injury (eg, basilar skull fracture, penetrating trauma, surgi-


A
cal intervention). Often, first-​line agents are vancomycin
paired with cefepime, ceftazidime, or meropenem.

• In patients with skull fractures, nasal fluid drainage


may be assayed for β2-​transferrin, which is unique to
CSF and the vitreous of the eye.
• Open, depressed skull fractures generally require
surgical intervention and empirical antibiotic
coverage.

Epidural Hematoma
Definition and Epidemiologic Factors
Epidural hematomas (EDHs) occur in approximately 1% of
head injuries but account for a large proportion of fatalities
from such injuries. The incidence is highest in men
younger than 40 years. Occurrence is rare in children
younger than 2 years and in adults older than 60 years
because of stronger adherence of the dura to the inner table
of the calvarium. EDHs result from vascular injuries to
dural vessels or from depressed skull fractures.
Arterial insults make up 85% of EDH cases and fre-
JVH quently occur peripterional with the epicenter in the mid-
© MAYO
2014 dle fossa, often because of injury to the middle meningeal
artery where it runs through the foramen spinosum.
However, EDH can occur because of physical disruption to
meningeal veins, dural sinuses, or diploic vessels in the
B skull. In some cases, EDHs occur in the posterior fossa and
the occipital and frontal regions. Diagnostic evaluation
should be expanded if no clear precipitating trauma is
detected, because EDH can occur rarely from a nontrau-
matic cause, such as infections, coagulopathic factors, vas-
cular malformations, and neoplastic lesions.

Symptoms
The triggering injury of an EDH often causes an initial brief
LOC. A relatively minimal potential cranial space can ini-
JVH
© MAYO tially quell major hematoma expansion, which results in
2014
the classic lucid interval where the patient is often at neu-
Copyright © 2021. Oxford University Press, Incorporated. All rights reserved.

rologic baseline. However, as the hematoma enlarges, the


Figure 60.2 Basilar Skull Fractures. A basilar fracture level of consciousness can deteriorate rapidly, resulting in
involves at least 1 of 5 bones that make up the skull base. mortality rates up to 50%. However, this classic and long-​
These bones are cribriform plate, orbital plate of frontal attributed sequence of clinical signs occurs only for some
bone, temporal bone, sphenoid bone, and occipital bone. A, patients and with considerable clinical heterogeneity.
Ecchymosis of the mastoid region (Battle sign) and otorrhea As the hematoma further enlarges, the temporal lobe is
(temporal bone fracture) may occur with basilar skull pushed medially, causing herniation of the uncus and
fracture. B, Periorbital ecchymosis (raccoon eyes) may mesial structures over and through the tentorial notch.
occur with basilar skull fracture or anterior or middle fossa Uncal herniation is characterized by decreasing conscious-
facial trauma. ness, early dilatation of the ipsilateral pupil, and contralat-
eral hemiparesis. The hemiparesis usually is contralateral
because of the decussation of the descending pyramidal
repair of dural injury, hemostasis, cranioplasty, and skin tracts. Ipsilateral hemiparesis may result if the opposite
reapproximation. Broad-​ spectrum antibiotic coverage cerebral peduncle is compressed against the tentorial edge
should be tailored to patient age, risk factors, and type of (Kernohan sign). The Kernohan notch is the indentation in

Mayo Clinic Neurology Board Review, edited by Kelly D. Flemming, Oxford University Press, Incorporated, 2021. ProQuest Ebook Central, http://ebookcentral.proquest.com/lib/cuhk-ebooks/detail.action?docID=6746419.
Created from cuhk-ebooks on 2023-09-02 09:10:26.
494 Section IV. Neurologic Intensive Care Disorders

the cerebral peduncle caused by the displacement of the


brainstem against the opposite tentorium during tentorial A
herniation.

Diagnosis and Management


A head CT without contrast medium should be prudently
obtained, which would show a convex, hyperdense lesion
that does not cross suture lines unless a fracture disrupts
the dural attachment (Figure 60.3). Vigilance should be
undertaken when reviewing the imaging because concur-
rent subdural hematomas or cortical SAHs may be present.
Surgical evacuation is most often indicated after initial
stabilization and reversal of any coexisting coagulopa-
thies. Patients with small EDHs, often of the venous type,
who have normal neurologic examinations may be
observed in an appropriately monitored setting with serial
neurologic examinations. If the EDH volume exceeds 30
cm3, immediate surgical evacuation should be undertaken
B
regardless of examination results.

• Epidural hematomas result from vascular injuries to


dural vessels or from calvarial fractures.
• The temporal fossa epidural hematoma is the most
common and results from damage to the middle
meningeal artery.
• The injury may result in an initial period of
unconsciousness, subsequently followed by a lucid
interval due to relatively adherent dura to the inner
table of the calvarium, limiting hematoma expansion.
As the hematoma enlarges, resulting in brain
compression and herniation of the cerebral peduncle,
the patient may have contralateral hemiparesis,
ipsilateral mydriasis, nausea and vomiting, and
seizures. With compression of the contralateral
cerebral peduncle against the tentorium, paradoxical
ipsilateral hemiparesis may also occur
(Kernohan sign).
• Rapid surgical evacuation is imperative after
hemodynamic stabilization and reversal of any
Copyright © 2021. Oxford University Press, Incorporated. All rights reserved.

coexisting coagulopathies, to prevent death.

Acute Subdural Hemorrhage


Figure 60.3 Epidural Hemorrhage. A, Gross specimen. B,
Definition and Epidemiologic Factors On computed tomography, epidural hemorrhage appears as
The term subdural hemorrhage (SDH) refers to accumula- a biconvex hyperdense lesion, often with substantial mass
tion of blood products between the arachnoid and dura. effect (shown herein). Small low-​density areas within the
Acute SDHs most commonly occur with falls, motor vehi- hemorrhage (arrowheads) are likely hyperacute, unretracted
cle or motorcycle collisions, and blunt force trauma (Figure semiliquid blood building up rapidly.
60.4). Rarely, an acute SDH may occur spontaneously (or (From Flemming KD. Principles of critical care neurology. In:
after minor trauma) for patients receiving long-​term antico- Mowzoon N, Flemming KD, editors. Neurology board review: an
agulation therapy or with coagulopathies or after rupture illustrated study guide. Rochester [MN]: Mayo Clinic Scientific
of a cerebral aneurysm. Most of the acute SDHs result from Press and Florence [KY]: Informa Healthcare USA; c2007. p. 401-​
venous vascular injury at the brain surface, which is par- 34; used with permission of Mayo Foundation for Medical
ticularly common in elderly persons with cerebral atrophy Education and Research.)
that results in stretch and traction forces applied to fragile

Mayo Clinic Neurology Board Review, edited by Kelly D. Flemming, Oxford University Press, Incorporated, 2021. ProQuest Ebook Central, http://ebookcentral.proquest.com/lib/cuhk-ebooks/detail.action?docID=6746419.
Created from cuhk-ebooks on 2023-09-02 09:10:26.
Chapter 60. Traumatic Brain Injury 495

bridging veins. Less commonly, interhemispheric fissure


A or tentorial SDHs are also seen. Serial imaging is often
important to distinguish a small SDH from calcification or
imaging artifact in these regions.

Symptoms
Common presenting symptoms of acute SDH depend on
the size and location of the SDH as well as any additional
traumatic injury. Frequent clinical presentations include
headaches, weakness, and sensory changes, altered menta-
tion including coma, seizures, and altered language if the
SDH occurs over the dominant hemisphere. Because of the
nature of injury in patients with acute SDH, they may have
symptoms related to associated cerebral or brainstem con-
tusions. The latter may account for the high mortality rate
(50%-​90%) associated with acute SDH.
Acute SDHs are highly epileptogenic, and clinical sei-
B zures occur in at least 10% of patients. However, emerging
data suggest that subclinical seizures may be even more
prevalent than previously thought.

Management
Any coagulopathy should be reversed. Symptomatic acute
SDHs—​more than 1 cm at the thickest point and more than
5 mm of midline shift—​should be given strong consider-
ation for surgical evacuation after stabilization and rever-
sal of coagulopathy as indicated. Acute SDHs require
hemicraniotomy or hemicraniectomy, both of which are
invasive procedures, particularly for elderly persons.
Ideally, in a patient with mild or minimal neurologic defi-
cits whose status is not deteriorating, it is clinically benefi-
cial to wait for the SDH to become chronic and begin to
liquefy, allowing for a much less invasive burr hole evacu-
ation. All patients with acute SDHs with or without surgi-
cal evacuation should receive pharmacologic seizure
prophylaxis. Patients with a low GCS score of 3 on admis-
sion have mortality rates up to 90%. Functional survival is
less than 20% if the GCS score is less than 6.
Copyright © 2021. Oxford University Press, Incorporated. All rights reserved.

• Symptomatic acute subdural hemorrhages of more


than 1 cm in maximal diameter and more than 5 mm
in maximal midline shift often warrant surgical
consideration.
Figure 60.4 Acute Subdural Hemorrhage. A, Gross specimen.
• Acute subdural hemorrhages are highly epileptogenic,
B, On computed tomography, acute subdural hemorrhage
and patients should be treated with pharmacologic
appears as a hyperdense lesion over the convexity, shown
seizure prophylaxis.
here bilaterally.
• Spontaneous acute subdural hemorrhage should be
(A is from Flemming KD. Principles of critical care neurology. In:
evaluated for underlying vascular abnormality or
Mowzoon N, Flemming KD, editors. Neurology board review: an
coagulopathy.
illustrated study guide. Rochester [MN]: Mayo Clinic Scientific
Press and Florence [KY]: Informa Healthcare USA; c2007. p. 401-​
Subacute and Chronic SDH
34; used with permission of Mayo Foundation for Medical
Education and Research.) Definition
A patient with a subacute or chronic SDH presents days to
weeks after the time of injury and shows subacute to

Mayo Clinic Neurology Board Review, edited by Kelly D. Flemming, Oxford University Press, Incorporated, 2021. ProQuest Ebook Central, http://ebookcentral.proquest.com/lib/cuhk-ebooks/detail.action?docID=6746419.
Created from cuhk-ebooks on 2023-09-02 09:10:26.
496 Section IV. Neurologic Intensive Care Disorders

long-​term blood products on imaging. Chronic SDH is most Traumatic Intracerebral Hemorrhage
common among elderly patients and persons with alcohol-
Definition
ism. These persons usually have some brain atrophy with
Traumatic intracerebral hemorrhages (tICHs) account for
a resultant increased subdural space. Patients receiving
20% of all traumatic intracranial hemorrhages. tICHs occur
long-​term anticoagulation therapy or those who harbor
within the brain parenchyma and appear hyperdense on
blood dyscrasia are also at high risk for SDH. The precipi-
noncontrast head CT. Spontaneous intracerebral hemor-
tating trauma may be minimal; in many cases, no direct
rhage (ICH) often occurs at the parietal and occipital lobes,
blunt force trauma to the head has occurred. Chronic trau-
basal ganglia, thalamus, or cerebellum. By comparison,
matic subdural hygromas and excessive subdural effusions
tICH usually develops in the temporal and frontal lobes
may occur from tearing the arachnoid membrane. SDHs
after trauma. Coup and countrecoup findings (Figure 60.6)
may also result from low CSF pressure syndromes or over­
are highly suggestive of trauma and are related to the sud-
shunting through a ventriculoperitoneal catheter. Post­
den acceleration and deceleration forces applied on the
meningitis subdural hygromas can occur and should be
brain. These focal brain injuries typically result in contu-
considered if a recent infectious history is obtained.
sions or tICH. A tICH can be seen in conjunction with an
overlying SDH or SAH.
Symptoms and Presentation
Subacute or chronic SDHs generally develop within 7 to 10
Pathologic and Pathophysiologic Factors
days after injury. The signs and symptoms are similar to
Brain contusions represent focal regions of subpial hemor-
those of acute SDH, but the course is generally slower and
rhage and swelling. Contusions are most common in
the mortality rate correspondingly lower. In chronic SDH,
regions that contact bony surfaces in the cranial vault dur-
the signs and symptoms may be confused with ischemic
ing trauma: frontal and temporal poles, orbitofrontal gyri,
phenomena or encephalopathy and can fluctuate. Patients
perisylvian cortices, and inferolateral temporal lobe sur-
may have seizures.
faces. These contusions can be seen on the same side
(coup) or the opposite side (contrecoup) of impact. On CT,
Diagnosis and Management
contusions are commonly associated with areas of relative
The diagnosis of SDH can be made from the CT appearance
hypodensity in the white matter structures, an indication
of isodense or hypodense fluid collections along the cere-
of posttraumatic cerebral edema.
bral convexity (Figure 60.5). A small hemorrhage fills the
tICHs differ from cerebral contusions in that a large pro-
preexisting subdural space, and over several weeks, a vas-
portion of tICHs are composed of blood. Yet, they often
cular membrane forms around the collection. Chronic
result from growth or coalescence of smaller cerebral con-
SDHs occur bilaterally in 20% to 25% of cases.
tusions. tICHs are more common in patients with calvarial
For patients with subacute or chronic SDH, coagulation
fractures. About half of patients with severe TBI and skull
abnormalities should be corrected. The duration of rever-
fracture have a sizable tICH on initial head CT. Patients
sal should be individualized on the basis of the associated
receiving long-​term anticoagulation therapy are at increased
risk of premorbid conditions. Symptom severity and the
risk for tICH, after even a mild head injury.
SDH amount may determine whether observation or surgi-
cal intervention is warranted. Both subacute and chronic
Symptoms
SDH may be amenable to burr hole drainage when symp-
Because the common location of contusions includes the
tomatic or when the SDH size is moderate to large. Often,
Copyright © 2021. Oxford University Press, Incorporated. All rights reserved.

bifrontal and temporal regions, symptoms are often behav-


a subdural drain is left in place temporarily to reduce the
ioral and cognitive, which may impair return to normal
risk of reaccumulation, and patients may be kept lying flat
personal and professional activities. Patients may also
to promote brain reexpansion. In some cases, middle men-
have headaches, nausea and vomiting, and seizures.
ingeal artery embolization may be useful in treating
patients with chronic SDH who are at high risk for under-
Management
going surgery. Subdural hygromas are often monitored and
Monitoring and treatment of increased ICP are essential in
rarely require operative intervention unless symptoms are
tICH. Symptomatic patients with large-​volume clots (>50
severe.
cm3) and a midline shift greater than 5 mm should be con-
sidered for clot evacuation. Trials such as the Surgical Trial
• Subacute or chronic subdural hematomas generally
in Lobar Intracerebral Hemorrhage I and II and Early
develop within 7 to 10 days after injury.
Minimally Invasive Removal of Intracerebral Hemorrhage
• The severity of symptoms and amount of subdural
provide insight to whether patients may benefit from surgi-
hematoma may determine whether observation,
cal intervention and operative approaches for evacuation
surgical intervention, or middle meningeal artery
of the clot. However, these trials address spontaneous ICH,
embolization is warranted.
which may not be generalizable.

Mayo Clinic Neurology Board Review, edited by Kelly D. Flemming, Oxford University Press, Incorporated, 2021. ProQuest Ebook Central, http://ebookcentral.proquest.com/lib/cuhk-ebooks/detail.action?docID=6746419.
Created from cuhk-ebooks on 2023-09-02 09:10:26.
Chapter 60. Traumatic Brain Injury 497

A
R L

Figure 60.5 Chronic Subdural Hematoma on Computed


Tomography. On the right (R) side of the image, a chronic
subdural hemorrhage appears hypodense. On the left (L), a B
subacute hematoma is modestly isodense.
(From Flemming KD. Principles of critical care neurology. In:
Mowzoon N, Flemming KD, editors. Neurology board review: an
illustrated study guide. Rochester [MN]: Mayo Clinic Scientific
Press and Florence [KY]: Informa Healthcare USA; c2007. p. 401-​
34; used with permission of Mayo Foundation for Medical
Education and Research.)

Younger patients with superficial lobar hematomas


tend to benefit from surgery. In large, compressive hemor-
rhages, CSF diversion with an external ventricular drain
Copyright © 2021. Oxford University Press, Incorporated. All rights reserved.

may have a role in acutely temporizing high ICP and allow


continued ICP monitoring. Additional types of ICP moni- Figure 60.6 Contusion and Traumatic Intracerebral
tors may have a role for these patients. Care should be Hemorrhage. A, Axial computed tomography of the head
taken not to implant the monitors into contusion epicen- without contrast medium shows a right frontal contusion
ters because the risk of further hemorrhage is extremely and traumatic intracerebral hemorrhage. B, Gross specimen
high, and with external ventricular drains, the catheter with bifrontal contusions related to trauma.
fenestrations may become obstructed. (B is from Flemming KD. Principles of critical care neurology. In:
Mowzoon N, Flemming KD, editors. Neurology board review: an
• Traumatic ICHs usually develop within the temporal illustrated study guide. Rochester [MN]: Mayo Clinic Scientific
and frontal lobes after trauma. Press and Florence [KY]: Informa Healthcare USA; c2007. p. 401-​
• Contusions can be seen on the same side (coup) or 34; used with permission of Mayo Foundation for Medical
opposite side (contrecoup) of impact. Education and Research.)
• Traumatic ICHs are more common in patients with
skull fractures.

Mayo Clinic Neurology Board Review, edited by Kelly D. Flemming, Oxford University Press, Incorporated, 2021. ProQuest Ebook Central, http://ebookcentral.proquest.com/lib/cuhk-ebooks/detail.action?docID=6746419.
Created from cuhk-ebooks on 2023-09-02 09:10:26.
498 Section IV. Neurologic Intensive Care Disorders

• Symptomatic patients who have traumatic ICH and self-​inflicted mechanisms. Projectiles can injure intracra-
large-​volume clots (>50 cm ) with greater than a 5-​mm
3 nial contents directly and indirectly through secondary
midline shift should be considered for clot evacuation pressure waves in adjacent brain and vascular tissue from
or CSF diversion. high-​velocity projectiles.

Diffuse Axonal Injury Management


After initial resuscitation and imaging, the treatment
Definition of projectile injuries in the brain depends largely on
Diffuse axonal injury (DAI) results from severe rotational
the extent of injury and the neurologic examination.
acceleration-​deceleration forces that deliver sheer and ten-
Management of penetrating injury is more nuanced,
sile forces to axons.
particularly if the diencephalic structures and mesen-
cephalon are affected. Bullets or bullet fragments may
Pathology and Clinical Presentation lacerate important arteries or venous structures. Bullet
The histologic findings of DAI include disruption and
fragments may also migrate as a result of vascular and
swelling of axons, retraction balls (swollen proximal ends
CSF pulsations. Neurosurgical intervention may be
of severed axons), and punctate hemorrhages in the dorso-
indicated for 1) ICP monitoring or CSF diversion, 2)
lateral pons and midbrain, as well as the corpus callosum.
depressed, open skull fractures, 3) intracranial hemor-
DAI is often attributed to the immediate disruption in cor-
rhage evacuation, or 4) intracranial decompression.
tical function resulting in LOC immediately following an
Patients with epidural hematomas greater than 30 cc
insult. DAI may be mild, moderate, or severe, with severe
should be considered for surgical evacuation indepen-
cases having prolonged coma, flexor or extensor posturing,
dent of clinical examination and GCS. Because of the
and dysautonomia. These severe impairments may occur
open wounds created by penetrating intracranial inju-
even without commonly associated parenchymal contu-
ries, patients require empirical antibiotic coverage until
sions or hematoma. The location and severity of axonal
further individualization can occur.
injuries are important determinants of functional recovery.

Diagnosis
DAI lesions are often difficult to visualize on conventional Traumatic Abuse: Physical Abuse
CT and are better seen on MRI. T2-​weighted gradient-​echo
Physical abuse is now recognized as a major cause of seri-
imaging is particularly sensitive for hemorrhagic lesions
ous head injury in children and is second only to motor
after DAI (Figure 60.7). Advancements in MRI, particularly
vehicle crashes as a cause of traumatic death in the pediat-
with 7-​T ultra-​high-​
field MRIs, may provide additional
ric population. In many institutions, a team approach to
insight to the true extent of TBI and of DAI.
manage cases of suspected nonaccidental trauma provides
an organized and standardized means of addressing the
Management
frequently complex issues involved in caring for these
Monitoring and treatment of increased ICP are essential in
patients.
DAI (see Chapter 56 “Principles and Management of
A skeletal survey is a mandatory part of the evaluation
Increased Intracranial Pressure,” for further detail). These
of suspected nonaccidental injury in infants and young
are often performed with ventricular catheters that have
children, and bone scanning may be useful when results of
Copyright © 2021. Oxford University Press, Incorporated. All rights reserved.

the benefit of CSF diversion to treat resultant hydrocepha-


plain radiography are equivocal. Ophthalmic examination
lus. However, parenchymal pressure monitors also are
may be helpful in looking for retinal hemorrhages. Studies
beneficial and are mildly less invasive.
for anemia, thrombocytopenia, additional hematologic
abnormalities, and toxicologic factors should be per-
• Diffuse axonal injury (DAI) results from severe
formed, along with standard complete blood cell counts
acceleration-​deceleration forces that deliver sheer and
and chemistry panels.
tensile forces to axons.
Certain types of injuries that occur with greater fre-
• DAI can result in severe neurologic sequela related to
quency in the physical abuse setting should raise a physi-
trauma, with or without associated contusions or
cian’s level of suspicion for nonaccidental causation (Box
hemorrhages.
60.2). Characteristic findings in shaken baby syndrome
include retinal hemorrhages, SDH (Figure 60.8), and cere-
Penetrating Injuries
bral edema. Other signs of abuse may or may not be pres-
Definition ent. A low threshold of suspicion should be maintained
Penetrating brain injuries are most commonly associated for children and vulnerable adults with acute traumatic
with gunshot wounds to the head, either from assault or injuries.

Mayo Clinic Neurology Board Review, edited by Kelly D. Flemming, Oxford University Press, Incorporated, 2021. ProQuest Ebook Central, http://ebookcentral.proquest.com/lib/cuhk-ebooks/detail.action?docID=6746419.
Created from cuhk-ebooks on 2023-09-02 09:10:26.
Chapter 60. Traumatic Brain Injury 499

A B

Figure 60.7 Diffuse Axonal Injury on Magnetic Resonance Imaging. A, Susceptibility-​weighted image shows extensive
scattered foci of cortical and subcortical susceptibility effect compatible with microhemorrhages related to diffuse axonal
injury. B, Diffusion-​weighted image shows cortical and subcortical patchy areas of restricted diffusion consistent with
diffuse axonal injury.

Box 60.2 • Injuries Potentially Related to


Chronic Traumatic Encephalopathy
Traumatic Abuse That May Raise Suspicion of An emerging area of TBI epidemiology is chronic traumatic
Nonaccidental Causes encephalopathy (CTE). It originally was described in
retired professional boxers. More recently, it has become
General further associated with professional athletes of various
Spiral fracture of humerus sports and other persons who receive repeat head injuries
Metaphyseal fractures in infants and concussions, particularly soldiers.
Copyright © 2021. Oxford University Press, Incorporated. All rights reserved.

Duodenal hematomas CTE continues to be a pathologic diagnosis. Phosphorylated


Frenulum tears in nonambulatory infants tau is aggregated in neurons, astrocytes, and cell processes in
Retinal hemorrhages the cortical sulci, a pattern distinct from Alzheimer disease.
Immersion burns Patients with these findings often have cognitive, behavioral,
Long-​bone injuries
and emotional symptoms before death. Debate continues on
whether selection bias exists in autopsy studies and how the
Rib fractures
clinical symptoms and pathologic findings correlate. In addi-
Cutaneous bruises
tion, research continues to focus on the identification of these
Patterned bruises
patients before death.
Neurologic
Subgaleal hemorrhage
Subperiosteal hemorrhage
Future Research
Skull fracture
Focal brain contusion Ongoing research is aimed at better diagnosis of the sever-
ity of injury, reduction in pathophysiologic and clinical

Mayo Clinic Neurology Board Review, edited by Kelly D. Flemming, Oxford University Press, Incorporated, 2021. ProQuest Ebook Central, http://ebookcentral.proquest.com/lib/cuhk-ebooks/detail.action?docID=6746419.
Created from cuhk-ebooks on 2023-09-02 09:10:26.
500 Section IV. Neurologic Intensive Care Disorders

extents of insult, and improvement of outcomes. Axonal


and glial injury products are potential avenues to more
sensitively detect a neurologic injury. In addition, certain
markers—​ such as tau, neurofilament light, Aβ, neuron-​
specific enolase, glial fibrillary acidic protein, S100β, and
spectrin fragments—​are under investigation in CSF and
blood. However, no markers have been routinely used in
the clinical practice.
In various forms, therapeutic hypothermia has been tri-
aled for acute neurologic injury. Although positive benefits
are routinely seen in preclinical trials, human studies so
far have been underwhelming. A large prospective, multi-
center randomized trial examining the effects of mild ther-
apeutic hypothermia was stopped early because of worse
outcomes in the treatment group. Preliminary evidence is
suggestive that targeted hypothermia to the brain may min-
imize the systemic adverse effects that often confound the
potential benefits.
Various medications championed initially for neurode-
generative disease are finding their way to TBI applica-
tions and may hold promise for the future.

Figure 60.8 Evidence of Physical Abuse. Left subdural


hygroma (arrowhead) and depressed skull fracture (arrow)
are seen in this computed tomogram of a 3-​year-​old patient.
Copyright © 2021. Oxford University Press, Incorporated. All rights reserved.

Mayo Clinic Neurology Board Review, edited by Kelly D. Flemming, Oxford University Press, Incorporated, 2021. ProQuest Ebook Central, http://ebookcentral.proquest.com/lib/cuhk-ebooks/detail.action?docID=6746419.
Created from cuhk-ebooks on 2023-09-02 09:10:26.
Acute Spinal Cord Compression, Spinal
61 Cord Trauma, and Peripheral
Neural Injury
CHRISTOPHER R. MARCELLINO, MD; EELCO F. M. WIJDICKS, MD, PHD

Introduction structural injury due to swelling and hemorrhage, which


can exacerbate tissue injury.

A
cute spinal cord compression with myelopathy is The loss of autoregulation in the microcirculation of
a neurologic emergency. Recognition of spinal the gray and white matter results in local ischemia,
cord compression, timely imaging, and treatment infarction, and hemorrhagic necrosis adjacent to the pri-
are important to restore and preserve neurologic function. mary injury. These changes lead to disruption of cellular
This chapter reviews the causes and clinical approach membrane potentials and ion-​mediated cell damage with
to spinal cord compression. Traumatic and nontraumatic the influx of calcium. Neuronal dysfunction results in
causes of spinal cord compression are addressed together excitotoxicity due to glutamate release, oxidative stress,
because of their overlapping symptoms and management. and, eventually, cell death. Macrophage invasion leads to
The chapter concludes with a brief discussion of periph- cavitation and disruption of tissue architecture and may
eral nerve injury. contribute to postinjury hydromyelia. Microglial cell
­
activation initiates neuroinflammation with glial scarring
that can become a barrier to axonal regrowth. Myelin
Spinal Cord Compression, Spinal Cord debris, including such proteins as Nogo-​ A, has been
shown to inhibit axonal repair and has become a thera-
Injury, and Vertebral Fracture peutic target. Chondroitin sulfate proteoglycans are
Pathophysiologic Factors secreted by reactive astrocytes and have been shown to be
Copyright © 2021. Oxford University Press, Incorporated. All rights reserved.

a barrier to axonal regrowth. Over time, this proinflam-


Spinal cord injury (SCI) has a primary and secondary
matory environment leads to wallerian degeneration and
injury process. Primary injury is an initial traumatic insult
further loss of ­function.
to the spinal cord that results in disruption of the long
axons, myelin, vasculature, and supporting spine that pro-
Clinical Presentation and Localization
vide motor, sensory, and autonomic function to the body.
Secondary injury follows primary injury and can lead to Patients present with deficits reflective of the spinal cord
progressive tissue damage for weeks to months after the level or levels of the compression. Paraparesis is common
initial SCI. The secondary form includes preventable (ie, for thoracic and lumbar lesions) and may be symmetri-
causes, such as hypotension and hypoxia, and further cal or asymmetrical. Additional characteristics may include

The editors and authors acknowledge the contributions of Patrick R. Maloney, MD, and Jeffrey T. Jacob, MD, to the previous edition of
this chapter.
Abbreviations: ASIA, American Spinal Cord Injury Association; CSF, cerebrospinal fluid; CT, computed tomographic, computed tomog-
raphy; MRI, magnetic resonance imaging; SCI, spinal cord injury

501

Mayo Clinic Neurology Board Review, edited by Kelly D. Flemming, Oxford University Press, Incorporated, 2021. ProQuest Ebook Central, http://ebookcentral.proquest.com/lib/cuhk-ebooks/detail.action?docID=6746419.
Created from cuhk-ebooks on 2023-09-02 09:10:26.
502 Section IV. Neurologic Intensive Care Disorders

bilateral Babinski sign, hyperreflexia (or areflexia acutely), The American Spinal Cord Injury Association (ASIA)
some degree of associated sensory loss or paresthesia, uri- grading system should be used to document the extent of
nary retention (increased postvoid residual volume), and injury and localize the level of the lesion (Figure 61.1).
decreased rectal tone. Respiratory compromise and dysau- The system grades are A (complete loss of sensory and
tonomia (eg, hypotension, bradycardia) may also occur. If motor function below the level of injury) to E (normal
the cervical cord is involved, patients may have quadripa- motor and sensory function below the level of injury). It is
resis, and respiratory compromise is more common. also useful for prognostication.
A careful neurologic examination is important for doc-
umentation of neurologic deficits and to localize the level Differential Diagnosis of Spinal Cord
of the SCI or compression. Important findings may include Compression
paradoxical breathing (abdominal breathing due to dener-
The most common cause of spinal cord compression is
vation of thoracic musculature); reduced inspiratory
trauma, of which motor vehicle crashes are the most com-
capacity due to diaphragm involvement (cervical segments
mon source, followed by falls, gunshot wounds, and sports
3, 4, and 5); autonomic instability; flaccid limbs with
injuries. Nontraumatic causes include spontaneous or
reduced reflexes (hyperreflexia and spasticity several days
anticoagulation-​ associated epidural hematoma, epidural
to weeks after the injury); specific, dermatomal sensory
abscess, and expanding or structurally destructive meta-
level; urinary retention; priapism; bowel or bladder incon-
static lesions. In traumatic SCIs, the male to female ratio is
tinence; or sudden step-​off of spinous processes, indica-
approximately 4:1.
tive of fractures.
Paraparesis from spinal cord compression should be
Cauda equina syndrome is distinguished from conus
distinguished from other causes (Table 61.2). Guillain-​
medullaris syndrome by its lower motor neuron characteris-
Barré syndrome may manifest with paraparesis, but are-
tics and is caused by compression of the postganglionic
flexia and absent sensory changes distinguish it from cord
intraspinal nerve rootlets of the lumbar spine in the common
compression. An anterior spinal cord syndrome sparing
dural sac (Table 61.1). Conus medullaris syndrome results in
the dorsal columns (and thus sparing the fine touch, vibra-
a mix of upper and lower motor neuron disturbances.
tion, and proprioception) is more suggestive of an isch-
emic cause. Other spinal cord syndromes (Figure 61.2)
may be suggestive of trauma or other causes. Central cord
Table 61.1 • Distinguishing Characteristics of Cauda syndrome is an incomplete SCI with disproportionately
Equina and Conus Medullaris Syndromes greater motor deficits in the upper extremities than the
Syndrome lower extremities, often associated with hyperextension
injury. Brown-​Séquard syndrome involves half of the spi-
Characteristic Cauda Equina Conus Medullaris
nal cord on 1 side typically because of hemisection or
Onset/​course Gradual, asymmetrical Sudden, bilateral injury to 1 lateral half of the cord. It presents with ipsilat-
Pain Severe May or may not be eral motor deficit, proprioceptive and vibration deficit,
Radicular and low present and contralateral loss of pain and temperature sensation.
back (See Chapter 85, “Myelopathies.”)
Weakness Often asymmetrical Often symmetrical
Depending on Imaging and Diagnostic Tests
chronicity, may
Copyright © 2021. Oxford University Press, Incorporated. All rights reserved.

have associated Patients who are severely injured or had high-​risk trauma
atrophy (eg, high-​velocity injuries, falls from height, blast injuries)
Reflexes Reduced patellar and May solely affect generally undergo urgent computed tomography (CT),
ankle reflexes ankle reflexes without contrast medium, of the entire neuraxis in
Sensory loss Saddle numbness; may Perianal, symmetrical resource-​intensive settings. Otherwise, in the absence of an
be asymmetrical obvious mechanism, imaging choice is directed by clinical
and dermatomal examination findings, neurologic deficits, and pain.
Bowel/​bladder Urinary retention Urinary retention When a deficit is not explained by CT findings and
Impotence Impotence common acute cord compression is suspected, emergency magnetic
occasionally resonance imaging (MRI) of the spine is necessary (Figure
Level L1 or L2 L2 and belowa 61.3). MRI has the added benefit of characterizing edema,
a
In adults, considerable variation can occur in the level of termination of hemorrhage, and demyelination and can delineate the
the conus medullaris. nature of nontraumatic compressive lesions.

Mayo Clinic Neurology Board Review, edited by Kelly D. Flemming, Oxford University Press, Incorporated, 2021. ProQuest Ebook Central, http://ebookcentral.proquest.com/lib/cuhk-ebooks/detail.action?docID=6746419.
Created from cuhk-ebooks on 2023-09-02 09:10:26.
Chapter 61. Acute Spinal Cord Compression, Spinal Cord Trauma, and Peripheral Neural Injury 503

Figure 61.1 American Spinal Injury Association Classification of Spinal Cord Injury. DIP indicates distal interphalangeal;
IP, interphalangeal; L, left; LEMS, lower extremity motor sensory; LT, light touch; LTL, light touch left; LTR, light touch right;
max, maximum; MCP, metacarpophalangeal; MP, metacarpophalangeal; PIP, proximal interphalangeal; PP, pin prick; PPL,
pin prick left; PPR, pin prick right; R, right; SCI, spinal cord injury; UEMS, upper extremity motor sensory.
(From International Standards for Neurological Classification of Spinal Cord Injury [ISNCSCI] [Internet]. Atlanta [GA]: American Spinal
Injury Association. c2019. [cited 2020 Jan 7]. Available from: https://​ asia-​
spinalinjury.org/​
international-​
standards-​
neurological-​
Copyright © 2021. Oxford University Press, Incorporated. All rights reserved.

classification-​sci-​isncsci-​worksheet/​; used with permission.)

Mayo Clinic Neurology Board Review, edited by Kelly D. Flemming, Oxford University Press, Incorporated, 2021. ProQuest Ebook Central, http://ebookcentral.proquest.com/lib/cuhk-ebooks/detail.action?docID=6746419.
Created from cuhk-ebooks on 2023-09-02 09:10:26.
504 Section IV. Neurologic Intensive Care Disorders

Figure 61.1 Continued

Table 61.2 • Differential Diagnosis of Acute Myelopathy


Pathophysiologic Cause Specific Cause
Copyright © 2021. Oxford University Press, Incorporated. All rights reserved.

Vascular Spinal cord ischemia


Inflammatory or infectious Inflammatory: systemic lupus erythematosus
(transverse myelitis) Infectious: HIV, HTLV-​1, HSV-​2, rabies, West Nile virus, TB (Pott disease), Lyme disease, syphilis,
yaws, mycoplasma, leptospirosis, brucellosis, schistosomiasis, filariasis, eosinophilic meningitis
Demyelinating: multiple sclerosis, neuromyelitis optica, acute demyelinating encephalomyelitis
Structural (cord compression) Epidural compression: metastases, lymphoma, multiple myeloma, disk protrusion, epidural abscess
or bleed, spondylolysis, atlantoaxial subluxation (rheumatoid arthritis)
Extramedullary, intradural compression: meningioma, neurofibroma
Intramedullary expansion: glioma, ependymoma, arteriovenous malformation
Paraneoplastic Multiple antibodies and associated neoplasms
Toxic or metabolic Arsenic
Heroin
Acute B12 deficiency (nitrous oxide exposure)
Radiation
Trauma —​a
Abbreviations: HSV-​2, herpes simplex virus type 2; HTLV-​1, human T-​lymphotropic virus 1; TB, tuberculosis.
a
No subsets of causes in trauma.

Mayo Clinic Neurology Board Review, edited by Kelly D. Flemming, Oxford University Press, Incorporated, 2021. ProQuest Ebook Central, http://ebookcentral.proquest.com/lib/cuhk-ebooks/detail.action?docID=6746419.
Created from cuhk-ebooks on 2023-09-02 09:10:26.
Chapter 61. Acute Spinal Cord Compression, Spinal Cord Trauma, and Peripheral Neural Injury 505

D.F.
© MAYO
1998

Complete transection Advanced central lesion Brown-Séquard


(T10) (T10) (T3)

Combined loss

Analgesia
Loss of vibratory
and position sense

Cauda equina Anterior spinal artery


lesion syndrome (T4)

Figure 61.2 The Major Spinal Cord Syndromes.


Copyright © 2021. Oxford University Press, Incorporated. All rights reserved.

(From Wijdicks EFM. The practice of emergency and critical care neurology. 2nd ed. New York [NY]: Oxford University Press; 2016.
Chapter 36, Acute spinal cord disorders; p. 500-​16; used with permission of Mayo Foundation for Medical Education and Research.)

Patients who cannot undergo MRI can receive CT than otherwise anticipated. Additionally, patients with
myelography, but caution should be used because intradu- advanced ankylosing spondylitis have a lower sensitivity
ral pressure shifts have been shown to exacerbate neuro- for fracture detection with CT and are at higher risk for
logic deficits. If MRI or myelography does not show an unstable fractures, even in apparently minor trauma. They
abnormality, lumbar puncture and laboratory studies are should be evaluated with MRI after a fall or other injury,
often performed to look for an inflammatory or infectious especially when presenting with acute back pain.
cause of the myelopathy. (See Chapter 85, “Myelopathies.”)
In trauma, 20% of patients with a major SCI have a sec- Management
ond spine injury at a noncontiguous level. Therefore, they
Initial management should begin with assessment of air-
should undergo a careful neurologic evaluation, and they
way, breathing, and circulation. Intubation may be neces-
may require more complete imaging of the entire spine
sary when cervical or thoracic injury compromises the

Mayo Clinic Neurology Board Review, edited by Kelly D. Flemming, Oxford University Press, Incorporated, 2021. ProQuest Ebook Central, http://ebookcentral.proquest.com/lib/cuhk-ebooks/detail.action?docID=6746419.
Created from cuhk-ebooks on 2023-09-02 09:10:26.
506 Section IV. Neurologic Intensive Care Disorders

multiple levels. Certain types of dislocation or ligamen-


tous injury can be treated with external traction or ­bracing.
Exquisitely radiation-​ responsive tumors (eg, plasmacy-
toma) and metastatic disease of patients who are medi-
cally unable to tolerate surgery are better treated with
urgent radiation therapy.
Even in the absence of cord compression, traumatic
injuries that cause fractures or a ligamentous injury that
destabilizes the spine require immobilization (surgical
fusion of multiple vertebral segments, external bracing, or
halo stabilization) to allow for healing and to prevent sec-
ondary injuries. Multiple classifications are used for the
evaluation of spine fractures, including the thoracolumbar
injury classification and severity score. Pathologic frac-
tures from malignancy or osteoporosis are handled simi-
larly, but surgical decision-​ making is generally more
nuanced. Yet, because of the general absence of soft tissue
injuries, pathologic fractures are often less likely to be
unstable than their traumatic counterparts.

Cervical Spine Immobilization and Precaution


Clearance
Unlike the thoracic and lumbar spine, the cervical spine is
Figure 61.3 Cord Compression on T2-​Weighted Magnetic more frequently fractured or its stabilizing ligaments dis-
Resonance Imaging. A dorsal epidural extension of this rupted in trauma because of its exposure. Early immobili-
patient’s known right-​sided Pancoast tumor extends from zation of the cervical spine is important to prevent
C2-​
C3 through T1-​ T2, with moderate associated cord secondary injuries.
compression from C4 through T1. In low-​velocity blunt injuries, patients with no neuro-
logic deficit who are alert, compliant, and not intoxicated
and have no distracting injuries are evaluated for the
diaphragm or thoracic musculature or in the clinical set- absence of midline posterior cervical tenderness to palpa-
ting of obtundation from concomitant traumatic brain tion and the ability to complete a range-​of-​motion exami-
injury or hemorrhagic shock. nation. If the patient has neither cervical tenderness to
Cervical spine immobilization is performed with use of palpitation nor a full range of motion, trained providers
a rigid collar at first patient contact if indicated by guide- may safely clear the cervical spine from immobilization
lines (see the Cervical Spine Immobilization and Precaution without radiographic evaluation on the basis of validated
Clearance section below). The immobilization can prevent guidelines. Most commonly used are the Canadian C-​
secondary injuries in the event that the spine is structurally Spine Rule (available at: https://​ jamanetwork.com/​ jour-
unstable because of fracture or severe ligamentous injury. nals/​jama/​fullarticle/​194296) and the National Emergency
Copyright © 2021. Oxford University Press, Incorporated. All rights reserved.

In addition, for high-​risk trauma mechanisms, the patient is X-​Radiography Utilization Study guidelines.
treated with total spine precautions—​laying supine, trans- Patients who do not meet the above criteria and have
ferring while affixed to a rigid board, and so-​called log roll- not had CT performed require a 3-​view cervical radiograph
ing for any turns to prevent secondary injuries to the series. They also need a so-​called swimmer’s view if the
thoracic and lumbar spine (and cervical spine precau- lateral view does not include the C7-​ T1 interface. For
tions)—​until with imaging and examination findings the patients with degenerative disease of the cervical spine, a
precautions can be cleared. plain radiograph series is often inadequate to assess for
Further management depends on the cause of com- injury, and CT must be performed. In addition, for cases of
pression. If the cause of the myelopathy is structural, high-​velocity injuries, CT is indicated even if the clinical
immediate neurosurgical consultation is indicated. Cord examination is negative, to rule out fracture before remov-
compression due to fracture, dislocation, and structural ing the stabilizing collar. If pain is elicited, however, and
lesions such as an epidural hematoma, large herniated the radiograph or CT does not show fracture, either MRI or
disk, or certain tumors generally necessitates urgent sur- delayed flexion-​ extension radiographic imaging is per-
gery. The most common surgical options, depending on formed to evaluate for unstable ligamentous injury. If frac-
the myelopathy cause, are laminectomy, diskectomy, and tures or ligamentous injuries are identified, neurosurgical
corpectomy in combination with a segmental fusion of consultation is warranted.

Mayo Clinic Neurology Board Review, edited by Kelly D. Flemming, Oxford University Press, Incorporated, 2021. ProQuest Ebook Central, http://ebookcentral.proquest.com/lib/cuhk-ebooks/detail.action?docID=6746419.
Created from cuhk-ebooks on 2023-09-02 09:10:26.
Chapter 61. Acute Spinal Cord Compression, Spinal Cord Trauma, and Peripheral Neural Injury 507

The cervical spine must stay immobilized until the chain travels in the lateral gray column of the spinal cord
clinical examination becomes possible if the patient has a from T1 to L3 and is subject to injury at these levels.
head injury with altered sensorium, is intoxicated, has Although hypotension is often exacerbated by spinal
been given potent analgesics, or is comatose for any rea- shock, the term spinal shock is frequently misused to
son. For an obtunded trauma patient, the optimal approach describe severe hypotension due to SCI in lieu of neuro-
to cervical spine clearance continues to be controversial. genic shock.
Prolonged use of hard cervical collars can lead to pressure Neurogenic shock due to cervical and high thoracic
sores, which ultimately can lead to infection, need for sur- SCI is treated with combination α-​ and β-​adrenergic ago-
gical débridement, and, in rare cases, death. In our prac- nists such as norepinephrine or dopamine to counteract
tice, we perform cervical spine MRI to rule out ligamentous unopposed vagal tone and reduced cardiac contractility.
injury, unless contraindicated, after CT rules out fracture The opposite is true for lower SCI because hypotension is
but the patient cannot be clinically evaluated in 48 hours generally due to vasodilatation, and nonionotropic agents
to rule out ligamentous injury (alert, reliable, not dis- such as phenylephrine are recommended. Still, various
tracted, and no midline tenderness or limitation in range vasopressors are used clinically because of practical con-
of motion). However, recent studies suggest that high-​ siderations about obtaining central access. Hydration is
quality thin-​cut CT interpreted by a neuroradiologist or a an important adjunct, but it must be used judiciously to
neurosurgeon is adequate for both fracture and ligamen- avoid volume overload and subsequent pulmonary edema
tous injury clearance in patients without ankylosed spines. and to prevent medical complications. Hemodynamic
This approach is controversial, however. augmentation should target a mean arterial pressure of 85
mm Hg or higher for the first 7 days after SCI to minimize
Corticosteroids in SCI secondary injury due to hypotension if the patient is
The use of corticosteroids is not recommended in traumatic unable to maintain a normal blood pressure. Abdominal
SCI by neurosurgical societies. Yet, this topic continues to binders and pressure garments are useful in the manage-
be debated. Corticosteroid administration is associated ment of chronically symptomatic postural hypotension
with higher rates of medical complications. The best evi- after SCI.
dence in support of corticosteroids shows only a marginal Regardless of the etiologic factors, supportive manage-
functional benefit based on a post hoc subgroup analysis, ment may include placement of a urinary catheter because
no change in death rate overall, and an increased death rate urinary retention is common. It may include monitoring
for patients with concomitant traumatic brain injury. and treating dysautonomia (with atropine or transcutane-
However, corticosteroids are commonly used for cord com- ous pacing for severe bradycardia) and monitoring respira-
pression due to neoplastic compression. (See Chapter 105, tory status (especially for a high cervical lesion). Intradural
“Spinal Cord Tumors.”) lumbar drains are used after iatrogenic vascular injuries to
the spinal cord and prophylactically during high-​ risk
Complications of SCI endovascular surgery for thoracic and thoracoabdominal
aortic aneurysms. Cerebrospinal fluid (CSF) diversion low-
Spinal Shock
ers intrathecal pressure and provides a gradient for
Spinal shock refers to the findings of areflexia or hypore-
increased perfusion to minimize the risk of infarction.
flexia and autonomic dysfunction seen early in acute SCI,
Research is actively being conducted to determine whether
which may be associated with a profound transient loss of
CSF diversion is efficacious for SCI patients, given the
some or all spinal cord function caudal to the level of
Copyright © 2021. Oxford University Press, Incorporated. All rights reserved.

risks of secondary injury by cord hypoperfusion.


injury. This paradoxical areflexia occurs immediately and
lasts between 24 and 72 hours after injury. Reflexes return
Outcomes
over the course of days and ultimately reflect hyperreflexia
and spasticity. A transient paralysis due to spinal shock Common complications after SCI include bowel or bladder
with complete recovery has been described in younger dysfunction, deep vein thrombosis, dysautonomia, and
patients with athletic injuries and provides an important pneumonia. Pulmonary complications are the most com-
aspect to the usual poor prognosis of complete (ASIA mon cause of death after SCI in both the short and long
A) SCI. term. Aggressive prevention and treatment of these com-
plications are essential.
SCI-​Associated Hypotension and Neurogenic Shock The most important prognostic factors determining out-
In the clinical setting of acute SCI, hypotension can be comes in patients with SCI are age and extent of injury.
caused by interruption of sympathetic autonomic function Persons with paraplegia have life expectancies similar to
that results in loss of vasomotor tone, bradycardia due to their age-​matched cohorts. However, the 10-​year mortality
unopposed parasympathetic action, and hypothermia—​all rate for quadriplegia with onset at an age greater than 50
of which contribute to secondary injury. The sympathetic years is near 50%; the 20-​year mortality rate for patients

Mayo Clinic Neurology Board Review, edited by Kelly D. Flemming, Oxford University Press, Incorporated, 2021. ProQuest Ebook Central, http://ebookcentral.proquest.com/lib/cuhk-ebooks/detail.action?docID=6746419.
Created from cuhk-ebooks on 2023-09-02 09:10:26.
508 Section IV. Neurologic Intensive Care Disorders

age 30 through 49 years has historically been approxi- • Conus medullaris and cauda equina syndromes are
mately 70%. Patients who are ventilator dependent have distinct, resulting from compression of the lumbar
approximately half the life expectancy of those with other- spinal cord and postganglionic intradural nerve
wise comparable injuries who do not have such depen- rootlets, respectively (Table 61.1).
dency. Most of the recovery a patient achieves takes place • Spinal shock describes the paradoxical areflexia or
within the first year of injury. Research is ongoing in stem hyporeflexia that can occur generally within the first 24
cell therapy and other biologic approaches to improve the hours after spinal cord injury, before development of
regeneration capabilities within the spinal cord. hyperreflexia.
After major SCI, syringomyelia can develop in up to • Hypotension and neurogenic shock can occur after
30% of patients because of disrupted CSF flow and cause acute spinal cord injury and are treated aggressively.
further neurologic deficits (Figure 61.4). Typically, ascend- • Syringomyelia can develop in up to 30% of patients
ing symptoms develop insidiously years after a thoraco- after major spinal cord injury, causing further
lumbar injury that initially caused complete paraparesis neurologic deficits.
but later leads to a cervical syrinx that can extend into the • Validated protocols exist for clearing cervical spine
brainstem and even cause bulbar symptoms. Early surgical precautions.
treatment of complete SCI to restore CSF flow and later • The most important prognostic factors of patients with
CSF diversion for symptomatic syrinxes are used to pre- spinal cord injury are age and extent of injury. The 10-​
vent worsening deficits. year mortality rate is near 50% in quadriplegia with
onset when the patient is older than 50 years.
• Trauma is the most common primary cause of acute
spinal cord compression.
• Neurologic deficits reflect the spinal level of injury and Radiculopathy and Peripheral
should be evaluated further with CT and MRI.
Alternative causes of paraparesis and quadriparesis can Neural Trauma
be identified through history, examination findings not Degenerative Spine Disease
consistent with upper motor neuron signs, and imaging
findings. Degenerative spine disease, or spondylosis, is more preva-
• Surgical treatment is indicated for traumatic spinal lent with age. Spondylosis involves hypertrophy of the fac-
cord compression and often is needed for unstable ets, lamina, and ligaments, along with intervertebral disk
fractures and dislocations, even without cord degeneration, osteophyte formation, and loss of cervical and
compression. lumbar lordosis, and results in multiple pain generators. In

A B C
Copyright © 2021. Oxford University Press, Incorporated. All rights reserved.

Figure 61.4 Symptomatic Cervical Syrinx. A symptomatic C3-​C6 cervical syrinx was discovered 20 years after a traumatic
injury at the C7 level due to a fall from a bridge. The patient presented with a 6-​month history of episodes of hypotension
(systolic pressure <60 mm Hg) and right upper limb paresthesia associated with neck flexion. A, Sagittal T2-​weighted
magnetic resonance scan of the cervical spine. B, Cerebrospinal fluid egress under pressure after posterior midline
myelotomy. C, Synthetic shunt tube placement to allow drainage into the subarachnoid space.
(From Brodbelt AR, Stoodley MA. Post-​traumatic syringomyelia: a review. J Clin Neurosci. 2003 Jul;10[4]‌:401-​8; used with permission.)

Mayo Clinic Neurology Board Review, edited by Kelly D. Flemming, Oxford University Press, Incorporated, 2021. ProQuest Ebook Central, http://ebookcentral.proquest.com/lib/cuhk-ebooks/detail.action?docID=6746419.
Created from cuhk-ebooks on 2023-09-02 09:10:26.
Chapter 61. Acute Spinal Cord Compression, Spinal Cord Trauma, and Peripheral Neural Injury 509

the absence of a progressive neurologic deficit, an initial The lack of an advancing Tinel sign is indicative of a poor
course of nonoperative management is indicated in patients prognosis.
with minor or no neurologic deficits. These therapies
include analgesics, epidural corticosteroid injections, oral Brachial Plexus Injuries
anti-​inflammatory agents, physical therapy, weight loss, and
The brachial plexus is often traumatically injured because
muscle relaxants. Indications for surgery include cauda
of penetrating trauma, traction injuries, first-​rib fractures,
equina syndrome (Table 61.1), progressive neurologic defi-
and expansile hematomas. The 2 common brachial plexus
cit, intractable pain, and spinal deformity.
injuries are given the eponyms Erb palsy and Klumpke
palsy (Table 61.3).
Radiculopathy
The severity of the deficit tends to be worst at initial
Intervertebral disk herniations or facet narrowing may presentation and improves with time. It generally requires
result in nerve root compression. Symptoms may include no intervention, especially for traction and compression
back or neck pain, individual nerve root pain, and sen- injuries. Most injuries are managed conservatively for up
sory or motor deficits related to the individual root (see to 5 months. Progressive deficits are concerning for a vas-
Chapter 6, “Anatomy of Peripheral Nerves, Neuromuscular cular injury (pseudoaneurysm, hemorrhage, or posttrau-
Junctions, and Skeletal Muscles,” and Chapter 87, matic arteriovenous fistula) and should be surgically
“Peripheral Nerve Disorders”). The majority of cervical explored urgently. Sharp transections are also repaired
disk herniations occur at C6-​C7 (C7 radiculopathy) and surgically because only in the first 24 to 48 hours can the
C5-​C6 (C6 radiculopathy). nerve endings be sutured together to prevent a permanent
Lumbar disk herniation most commonly occurs at L4-​ deficit. For up to a year after injury, nerve grafts can be
L5 and L5-​ S1. MRI is the diagnostic study of choice, performed for patients without clinical improvement to
although CT myelography may be useful. repair a neuroma in an otherwise continuous nerve, or
Disk herniations often improve with nonsurgical thera- portions of other functional nerves can be transferred to
pies. Indications for surgery include progressive neuro- the donor nerve. An example is the transfer of 1 or 2 fas-
logic deficit, large disk herniation, cauda equina syndrome, cicles from the ulnar nerve to the median nerve in the
and persistent severe radicular pain despite a period of Oberlin procedure to restore elbow flexion. Occasionally,
conservative management. other muscles such as the trapezius are transposed into the
arm to restore elbow flexion (in lieu of shoulder elevation)
Peripheral Nerve Injury when no other options exist to allow use of the arm.

For evaluation of traumatic injury to the peripheral nerves,


the clinical history, physical examination, and diagnostic
testing provide insight on the injury location and the extent Table 61.3 • Brachial Plexus Injury Syndromes
of injury. A detailed motor and sensory examination of the
Characteristic Erb-​Duchenne Palsy Klumpke Palsy
affected limb is performed to differentiate preganglionic
from postdorsal root ganglion injuries because the former Nerve roots C5-​C6 a
C8-​T1
cannot be repaired surgically. Mechanism Neck hyperextension, Shoulder
Electrodiagnostic studies are useful in the preoperative especially from hyperextension or
and intraoperative evaluations of a peripheral nerve falls, and shoulder neck stretching or
Copyright © 2021. Oxford University Press, Incorporated. All rights reserved.

injury. They can help determine the nerves injured, the dystocia in birth avulsing lower
canal injuries from roots
location of the injury, and the presence of regeneration.
traction, stretching
Electromyography can produce useful information imme- upper roots
diately after injury, but some findings require at least 3
Weakness Deltoid, rotator cuff, Wrist flexors and
weeks after injury to allow for complete wallerian degen- elbow flexors, wrist hand intrinsic
eration, such as fibrillation potentials. MRI can be a useful and hand muscle
adjunct when the nature of the injury is unclear. Resting position Shoulder adducted Hand supinated,
Regenerating nerve fibers develop mechanosensitivity. and internally wrist extended,
Percussion over the course of the nerve produces tingling rotated (waiter’s and digits flexed
paresthesia in the sensory distribution of the nerve (Tinel tip) (claw hand)
sign). Starting distally and progressing toward the lesion Sympathetic None Horner syndrome
site, tingling is perceived as soon as the tip of the down-​ involvement
growing hypersensitive but not yet myelinated fibers is Prognosisb Excellent Poor
met. An advancing Tinel sign does not indicate the num- a
Sometimes considered to include C7.
ber or quality of regenerating axons and does not guarantee b
Combined injuries to the total plexus (C5-​T1) have the worst prognosis
functional outcome. However, the contrapositive is true: and result in a flaccid arm.

Mayo Clinic Neurology Board Review, edited by Kelly D. Flemming, Oxford University Press, Incorporated, 2021. ProQuest Ebook Central, http://ebookcentral.proquest.com/lib/cuhk-ebooks/detail.action?docID=6746419.
Created from cuhk-ebooks on 2023-09-02 09:10:26.
510 Section IV. Neurologic Intensive Care Disorders

• Indications for surgery in a degenerative disk include • Many peripheral nerve injuries improve with time and
cauda equina syndrome, progressive neurologic deficit, are conservatively managed for up to 5 months.
intractable pain, and spinal deformity. Multiple surgical techniques are used for persistent
• Peripheral nerve injuries produce deficits concordant deficits thereafter.
with their anatomical function. • Sharp transections and progressive deficits require
immediate surgical exploration and repair.
Copyright © 2021. Oxford University Press, Incorporated. All rights reserved.

Mayo Clinic Neurology Board Review, edited by Kelly D. Flemming, Oxford University Press, Incorporated, 2021. ProQuest Ebook Central, http://ebookcentral.proquest.com/lib/cuhk-ebooks/detail.action?docID=6746419.
Created from cuhk-ebooks on 2023-09-02 09:10:26.
Neuromuscular Disease in the
62 Neuroscience Intensive Care Unit
MAXIMILIANO A. HAWKES, MD; EELCO F. M. WIJDICKS, MD, PHD

Introduction Scalene and sternocleidomastoid muscles are active during


breathing, and their contraction can be palpated. Weakness

N
eurologists may meet patients with neuromuscu- leads to smaller tidal volume breaths and poor cough.
lar disease and subsequent respiratory failure in Despite increasing respiratory frequency, atelectasis ensues,
the inpatient setting. Common clinical conditions causing pulmonary shunt and resultant hypoxemia. Poor
include Guillain-​Barré syndrome (GBS) and myasthenia secretion clearance may lead to upper airway obstruction
gravis (MG). Recognition of impending respiratory failure and aspiration.
for such patients is of great importance. Early signs of respiratory failure include anxiety, rest-
This chapter describes general features of neuromuscu- lessness, forehead sweating, tachycardia, tachypnea, and a
lar weakness and neuromuscular respiratory failure. In tendency to sit upright (Box 62.1). Speech is hesitant and
addition, it reviews the clinically important aspects of interrupted. Early on, patients may compensate and main-
GBS, MG, and botulism. Finally, the chapter outlines the tain a normal-​appearing arterial blood gas (ABG) value.
clinical symptoms, diagnosis, and treatment of critical ill- Nocturnal hypoventilation and hypercapnia may be pres-
ness polyneuropathy, a commonly encountered condition ent at this stage as a result of relaxation of accessory mus-
in the medical intensive care unit (ICU). cles at night and worse diaphragmatic efficiency in the
supine position. Hypercarbia is a late phenomenon.
Bedside respiratory tests, ABG tests, and a chest radio-
Neuromuscular Respiratory Failure graph are important adjuncts to the physical examination.
Bedside respiratory tests include measurement of vital
Brainstem respiratory centers, upper airway, cervical capacity and maximal inspiratory and expiratory pres-
Copyright © 2021. Oxford University Press, Incorporated. All rights reserved.

spine, phrenic nerve motoneurons, and respiratory mus- sures. Among patients with the various causes of primary
cles must act in concert to provide adequate minute venti- neuromuscular respiratory failure, the maximal inspira-
lation to maintain oxygenation and carbon dioxide tory and expiratory pressures (bugle pressures) appear to
clearance (Figure 62.1). have greater predictive value than vital capacity. They are
When respiratory muscle weakness occurs, chest wall effort-​and examiner-​dependent. Additionally, when facial
mechanics change. Poor diaphragmatic action can be com- weakness is prominent, poor mouth sealing around the
pensated by accessory muscle contraction. This causes an spirometer compromises test reliability.
upward displacement of the rib cage and a decrease in intra-​ The 20-​30-​40 rule is a useful predictor of respiratory
abdominal pressure. As the chest rises, the abdomen is failure in GBS. A patient is likely to require endotracheal
drawn inward, a phenomenon called paradoxic breathing. intubation if vital capacity is less than 20 mL/​kg, maximal

The editors and authors acknowledge the contributions of Philippe Couillard, MD, to the previous edition of this chapter.
Abbreviations: ABG, arterial blood gas; CMAP, compound muscle action potential; CSF, cerebrospinal fluid; GBS, Guillain-​Barré syn-
drome; ICU, intensive care unit; MG, myasthenia gravis; PLEX, plasma exchange; SNAP, sensory nerve action potential

511

Mayo Clinic Neurology Board Review, edited by Kelly D. Flemming, Oxford University Press, Incorporated, 2021. ProQuest Ebook Central, http://ebookcentral.proquest.com/lib/cuhk-ebooks/detail.action?docID=6746419.
Created from cuhk-ebooks on 2023-09-02 09:10:26.
512 Section IV. Neurologic Intensive Care Disorders

Acute brainstem
lesion
(eg, stroke,
demyelination,
poliomyelitis,
West Nile virus)

Acute spinal
Acutely blocked cord lesion
neuromuscular
junction traffic (eg, trauma,
demyelination,
(eg, myasthenia gravis, infarction,
organophosphates, postinfectious)
botulism, tick paralysis)

Acute peripheral
nerve disease
(eg, Guillain-Barré
syndrome, phrenic
nerve lesion,
acute porphyria)
Copyright © 2021. Oxford University Press, Incorporated. All rights reserved.

D.F.
© MAYO
2017

Figure 62.1 Causes of Neuromuscular Respiratory Failure.


(From Wijdicks EFM. The neurology of acutely failing respiratory mechanics. Ann Neurol. 2017;81[4]‌:485-​94; used with permission.)

inspiratory pressure is less than −30 cm H2O, and maximal decrease exceeding 20% from upright to supine also is
expiratory pressure is less than 40 cm H2O (Table 62.1). A worrisome. The predictive value of bugle pressures in MG
reduction of 30% of the baseline vital capacity or a is poor because of marked disease fluctuation. Study

Mayo Clinic Neurology Board Review, edited by Kelly D. Flemming, Oxford University Press, Incorporated, 2021. ProQuest Ebook Central, http://ebookcentral.proquest.com/lib/cuhk-ebooks/detail.action?docID=6746419.
Created from cuhk-ebooks on 2023-09-02 09:10:26.
Chapter 62. Neuromuscular Disease in the Neuroscience Intensive Care Unit 513

less than 20 mL/​kg, maximal inspiratory pressure is


Box 62.1 • Clinical Characteristics of Imminent less than −30 cm H2O, and maximal expiratory
Neuromuscular Respiratory Failure pressure is less than 40 cm H2O, then the patient likely
will require endotracheal intubation.
Dyspnea at low levels of work
Restlessness
Tachycardia (heart rate >100 beats per minute)
Tachypnea (respiratory rate >20 breaths per minute)
Guillain-​Barré Syndrome
Use of sternocleidomastoid, scalene muscles (by Overview and Epidemiologic Factors
palpation alone)
Many patients with GBS present with acute to subacute
Forehead sweating
flaccid paralysis. The rapidly progressive weakness can
Staccato speech
lead to quadriplegia, bulbar and respiratory muscle weak-
Asynchronous (paradoxic) breathing
ness, and dysautonomia. The annual disease incidence is
about 1 case per 100,000 persons, and the syndrome is the
Modified from Wijdicks EFM. The practice of emergency and
critical care neurology. 2nd ed. New York (NY): Oxford most common cause of acute flaccid paralysis worldwide.
University Press; c2016. Chapter 10, Short of breath; p. 78-​ Antecedent viral upper respiratory tract prodrome is
91; used with permission of Mayo Foundation for Medical
present for two-​thirds of GBS patients, but other infectious
Education and Research.
agents can be associated with the illness. Campylobacter
jejuni (30%) and cytomegalovirus (10%) are most com-
mon, followed in frequency by Epstein-​Barr virus, vari-
results may be at reference values for a patient with early
cella zoster virus, and Mycoplasma pneumoniae. Other
myasthenic crises whose condition later deteriorates to
reported precipitants include vaccination, surgery, and,
intubation.
more recently, Zika virus infection.
The ABG profile differs depending on the stage of the
neuromuscular weakness. In early stages, the ABG values
are characteristically at reference levels or show mild
Pathophysiologic Factors
hypocapnia with respiratory alkalosis. When mild hypox- GBS results from an autoimmune response leading to
emia is observed, basal atelectasis may be suspected. In the myelin damage. Autoantibodies are thought to bind to
clinical setting of tachypnea, a normal Pco2 means that the myelin antigens and activate complement. Vesicular degen-
system’s ability to compensate is overwhelmed. Established eration ensues because of the formation of a membrane-​
hypoxemia and hypercapnia are late findings. For this rea- attack complex on the outer surface of Schwann cells.
son, no indication exists for waiting until this point to Macrophages subsequently invade myelin and remove
decide about ventilatory support. myelin debris.
Lastly, a chest radiograph is necessary to exclude atel- Similarities between microbial and host proteins may
ectasis, pneumonia, and other forms of cardiopulmonary result in cross-​reacting immune responses and autoim-
disease. Presence of any of these findings lowers the mune disease, a response called molecular mimicry. Some
threshold for initiation of mechanical ventilation and evidence supports molecular mimicry between ganglio-
guides therapeutic efforts. sides and antecedent infectious agents to explain demye-
lination in GBS. Gangliosides—​an aggregate of sialic acid,
Copyright © 2021. Oxford University Press, Incorporated. All rights reserved.

• The 20-​30-​40 rule is a useful predictor of respiratory oligosaccharides, and ceramide—​ are important compo-
failure in Guillain-​Barré syndrome. If vital capacity is nents of the peripheral nerves. Depending on the site of the
immune attack, 2 main variants are described: acute
Table 62.1 • Laboratory Values Used in Monitoring inflammatory demyelinating polyneuropathy and acute
Respiratory Failure motor axonal neuropathy.

Factor Reference Value Critical Value


Clinical Presentation
Vital capacity, mL/​kg 40–70 <20
Paresthesia in the feet is often the first manifestation of
Maximal inspiratory Male, >−100 <−30
pressure, cm H2O Female, >−70
GBS. Nonspecific neck or back pain is frequently present
and is dismissed as trivial by patients. These first symp-
Maximal expiratory Male, >200 <40 toms occur mostly 2 weeks after a viral illness. Early in the
pressure, cm H2O Female, >140
process, the deep tendon reflexes are reduced or absent and
Modified from Wijdicks EFM. The practice of emergency and critical often not evaluated for a patient presenting to the emer-
care neurology. 2nd ed. New York (NY): Oxford University Press;
c2016. Chapter 10, Short of breath; p. 78-​91; used with permission of
gency department with solely tingling. Weakness in the
Mayo Foundation for Medical Education and Research. proximal muscles soon follows and may appear ascending.

Mayo Clinic Neurology Board Review, edited by Kelly D. Flemming, Oxford University Press, Incorporated, 2021. ProQuest Ebook Central, http://ebookcentral.proquest.com/lib/cuhk-ebooks/detail.action?docID=6746419.
Created from cuhk-ebooks on 2023-09-02 09:10:26.
514 Section IV. Neurologic Intensive Care Disorders

The degree of proximal involvement of upper extremities suspected depending on the clinical scenario (Table 62.2).
often mirrors respiratory muscle weakness and should Some helpful distinctions from GBS include the lack of sen-
alarm the clinician to seek signs, subtle and overt, of respi- sory changes of patients with hypophosphatemia or hypo-
ratory failure preemptively. Weakness severity of GBS can kalemia, MG, and botulism. In addition, for hospitalized
vary among patients, from mild to a virtual locked-​in state. patients with acute myopathy, the deep tendon reflexes are
Because progression is expected to occur for 1 to 4 generally retained, unlike with GBS where reflexes are gen-
weeks after symptom onset, hospitalization is the rule for erally absent. Lyme disease and HIV infection may present
observation, monitoring, and treatment. Findings on phys- with acute polyradiculopathy and may need to be assessed
ical examination are generally symmetrical weakness, are- depending on the clinical situation and risk factors.
flexia in weak muscles, and minimal loss of sensation Laboratory test results are generally unremarkable except
despite paresthesias. Variable cranial nerve dysfunction in for normovolemic hyponatremia due to an occasional syn-
the form of ptosis, facial diplegia, and dysphagia can be drome of inappropriate antidiuretic hormone level, moder-
present. Attention needs to be paid to abnormal vital signs, ate increase of the erythrocyte sedimentation rate, and
to detect frequent involvement of the autonomic nervous occasional perturbations of liver function. If results of liver
system. Common autonomic symptoms include cardiac function tests show increased concentrations, a hepatitis
conduction arrhythmias, orthostatic hypotension, hyper- panel should be obtained. Creatine kinase is in the normal
tension, and adynamic ileus. Early signs of respiratory fail- range or slightly increased for patients with pain. In cases of
ure are recognized from shortness of breath (often, pausing suspected respiratory failure, ABG testing and chest radiog-
in sentences), mild tachypnea, and mild tachycardia. raphy are appropriate to look for atelectasis and characteris-
Patients who have the Miller Fisher variant of GBS tics suggestive of aspiration among patients with dysphagia.
present with ophthalmoplegia, ataxia, and areflexia. Most Baseline electrocardiography is advisable. Brain and spinal
patients with Miller Fisher syndrome have autoantibodies imaging may also be useful to rule out a structural cause,
against the GQ1b ganglioside. Other, less common variants depending on the presentation of findings.
of GBS include acute motor axonal neuropathy, acute Electromyography most often shows slowed motor con-
motor and sensory axonal neuropathy, pandysautonomia, duction velocities and distal latencies, motor conduction
and pharyngeal-​cervical-​brachial involvement. block, temporal dispersion, and slowed or absent F waves.
It also can have characteristics of axonal degeneration.
Diagnostic Approach to Suspected GBS Cerebrospinal fluid (CSF) analysis shows albuminocyto-
logic dissociation. Most patients who are seen within the
Because of its high frequency, GBS should always be consid- first week of symptoms have a protein concentration
ered in the differential for acute flaccid paralysis. However, greater than 50 mg/​dL and few lymphocytes in the CSF. A
other conditions can mimic GBS. Hypokalemia should be high leukocyte count in the CSF prompts additional stud-
rapidly ruled out. Acute spinal cord compression should be ies (cytologic and HIV testing). Specific gangliosides can

Table 62.2 • Disorders Frequently Mimicking Guillain-​Barré Syndrome


Disease Relevant Clinical Features Helpful Laboratory Tests

Transverse myelitis Sensory level MRI of spine with gadolinium


Copyright © 2021. Oxford University Press, Incorporated. All rights reserved.

Urinary incontinence CSF: pleocytosis (>200 cells)


No facial or bulbar involvement
Myasthenia gravis Marked fatiguing ptosis and ophthalmoplegia EMG, NCV, repetitive stimulation
Intact tendon reflexes CSF: normal
Masseter weakness Neostigmine test
No dysautonomia
Vasculitic neuropathy History of PAN, SLE, GPA, or RA Chest and sinus radiography or CT of thorax
Pain without paresthesias Nerve and muscle biopsies
Marked asymmetry of weakness
Carcinomatous or lymphomatous Mental changes CSF cytologic assessment
meningitis Radicular pain MRI with gadolinium
Asymmetrical cranial nerve involvement MRI of spine or brain with gadolinium
Abbreviations: CSF, cerebrospinal fluid; CT, computed tomography; EMG, electromyography; GPA, granulomatosis with polyangiitis (formerly called
Wegener granulomatosis); MRI, magnetic resonance imaging; NCV, nerve conduction velocity; PAN, polyarteritis nodosa; RA, rheumatoid arthritis;
SLE, systemic lupus erythematosus.
Modified from Wijdicks EFM. The practice of emergency and critical care neurology. 2nd ed. New York (NY): Oxford University Press; c2016. Chapter 42,
Guillain-​Barré syndrome; p. 587-​607; used with permission of Mayo Foundation for Medical Education and Research.

Mayo Clinic Neurology Board Review, edited by Kelly D. Flemming, Oxford University Press, Incorporated, 2021. ProQuest Ebook Central, http://ebookcentral.proquest.com/lib/cuhk-ebooks/detail.action?docID=6746419.
Created from cuhk-ebooks on 2023-09-02 09:10:26.
Chapter 62. Neuromuscular Disease in the Neuroscience Intensive Care Unit 515

be found in these tests—​for example, GQ1b in the Miller Intravenous immunoglobulin and plasma exchange
Fisher variant. (PLEX) are accepted treatment choices for patients who
cannot walk unassisted. These treatments can speed up
Management and Treatment strength recovery and reduce the number of days of
mechanical ventilation. Re-​treatment can be considered
Respiratory failure and dysautonomia are important con-
if clinical symptoms progress after reaching a plateau
cerns and necessitate cardiac and respiratory monitoring.
with treatment. Corticosteroid therapy has no role in GBS.
Admission to the ICU is needed for patients with rapid dis-
ease progression, bulbar or facial dysfunction, severe dys-
Outcome and Prognosis
autonomia, and respiratory failure (Table 62.3). Preemptive
elective intubation is preferred if the patient shows the ini- Studies have shown that older age (>40 years), preced-
tial signs of respiratory failure and will decrease the emer- ing diarrhea, and severe weakness (low Medical Research
gency intubations. Counsel sum score) on admission and 1 week after onset
Electrocardiography, blood pressure, oxygen satura- predict lower probability of the ability to walk not only
tion, vital capacity, and swallowing should be checked within 4 weeks but also at 3 and 6 months. The mortality
regularly. The most common complications of GBS relate rate has been estimated at 3%. Yet, the rate doubles for
to disability incurred from the disease and the critical care patients who need long-​term mechanical ventilation and
environment. These complications are pneumonia, sepsis, may even approach 20%. Approximately 80% to 85% of
severe dysrhythmias, adynamic ileus and bowel perfora- patients walk independently at 1 year after diagnosis,
tion, deep vein thrombosis with pulmonary embolism, and full recovery occurs for approximately 60%.
gastrointestinal tract bleeding, pseudomembranous colitis, Relapses are rare but can occur in up to 5% of patients.
and complications of tracheostomy.
Treatment intensity depends on patient needs but • The rapidly progressive weakness of Guillain-​Barré
includes temporary cardiac pacing, mechanical ventila- syndrome (GBS) can lead to quadriplegia, bulbar and
tion, and orogastric or nasogastric enteral nutrition (Table respiratory muscle involvement, and dysautonomia.
62.3). Prophylaxis for gastrointestinal tract and venous • Campylobacter jejuni (30%) and cytomegalovirus
thromboemboli and use of ventilator-​associated pneumonia-​ (10%) are the most common among the infectious
reduction bundles are essential. Judicious antibiotic use is agents associated with GBS.
required for nosocomial infections. • Paresthesia in the feet and neck or back pain is often
the first manifestation of GBS.
• Patients who have the Miller Fisher variant of GBS
present with ophthalmoplegia, ataxia, and areflexia
Table 62.3 • Management of Guillain-​Barré Syndrome (GQ1b autoantibody test is often positive).
Type of Management Treatment • CSF analysis shows albuminocytologic dissociation
in GBS.
Airway, respiratory Intubate if vital capacity ≤20 mL/​kg
and maximal inspiratory pressure
• Intravenous immunoglobulin or plasma exchange is an
≤−30 mm Hg accepted treatment choice for patients with GBS who
Monitor bugle pressures cannot walk unassisted.
Monitor for aspiration • Approximately 80% to 85% of patients with GBS walk
independently at 1 year after diagnosis, and full
Copyright © 2021. Oxford University Press, Incorporated. All rights reserved.

Cardiac monitor Monitor for dysautonomia and


arrhythmia recovery occurs for approximately 60%.
Temporary cardiac pacing as needed
Prophylaxis DVT prophylaxis with subcutaneous
heparin
GI tract prophylaxis (critically ill
Myasthenic Crisis
patients) Overview and Epidemiologic Factors
Nursing/​rehabilitation Frequent turning of the patient to
avoid decubitus ulcers MG is an autoimmune neuromuscular junction disorder
Physical and occupational therapy (see also Chapter 88, “Neuromuscular Junction Disorders”).
Nutrition Enteral nutrition if swallowing is
Anti-​acetylcholine receptor and anti–​muscle-​specific kinase
impaired are the 2 most common causative antibodies. The disorder
manifests as fluctuating extraocular, bulbar, cervical, or
Immunomodulation IVIG, 0.4 g/​kg for 5 d
or
proximal weakness. Marked respiratory muscle weakness
PLEX every other day for 5 d can lead to respiratory failure, whereas profound bulbar
weakness can lead to marked difficulty with secretion clear-
Abbreviations: DVT, deep vein thrombosis; GI, gastrointestinal; IVIG,
intravenous immunoglobulin; PLEX, plasma exchange. ance, can diminish airway tone, and can hinder normal

Mayo Clinic Neurology Board Review, edited by Kelly D. Flemming, Oxford University Press, Incorporated, 2021. ProQuest Ebook Central, http://ebookcentral.proquest.com/lib/cuhk-ebooks/detail.action?docID=6746419.
Created from cuhk-ebooks on 2023-09-02 09:10:26.
516 Section IV. Neurologic Intensive Care Disorders

deglutition. The term myasthenic crisis is used when the


Table 62.4 • Differentiation of Cholinergic and
condition of patients with MG decompensates with respira-
Myasthenic Crises
tory and bulbar weakness, often requiring intubation.
Factor Cholinergic Crisis Myasthenic Crisis
Etiologic Factors
Frequencya Rare Common
Under several conditions, myasthenic crisis may be pre- Trigger Overdose, drug therapy Infection, certain
cipitated for patients with MG. These conditions include for MG drugs,
infection and fever, use of certain drugs (ie, aminoglyco- corticosteroids
sides, magnesium, and quinine), surgery, other systemic Pupils Miosis Mydriasis
illnesses, and pregnancy (Box 62.2) (see also Chapter 88, Respiration Bronchus plugging and Diaphragm failure
“Neuromuscular Junction Disorders”). Patients with recent spasm, marked
thymectomy may have this crisis, and weaning them from salivation
ventilator use is difficult. Fasciculations, Present Absent
Myasthenic crisis can develop abruptly with few clini- cramps
cal signs because of chronic immunosuppression or mus-
Diarrhea Present Absent
cle weakness that masks the usual signs of respiratory
Abbreviation: MG, myasthenia gravis.
failure. Moreover, oxygen desaturation is expected late in a
The combination of both crises is often encountered clinically.
the crisis state, sometimes shortly before respiratory arrest. From Wijdicks EFM. The practice of emergency and critical care
neurology. 2nd ed. New York (NY): Oxford University Press; c2016.
Diagnosis Chapter 43, Myasthenia gravis; p. 608-​25; used with permission of
Mayo Foundation for Medical Education and Research.
The diagnosis of myasthenic crisis is typically made on the
basis of symptoms in the clinical setting of known MG.
However, patients may present with their first serious crisis monitored setting for myasthenic crisis. Endotracheal intu-
before diagnosis. In these cases, the finding of fatigable weak- bation should be considered with deterioration in vital
ness during the examination is an important clinical hint. capacity or bugle pressures, atelectasis on chest radiogra-
A myasthenic crisis must be distinguished from a cholin- phy, or inability to handle secretions. However, a trial of
ergic crisis since they both may present with weakness (Table biphasic positive airway pressure before development of
62.4). A cholinergic crisis may occur because of excess ace- hypercapnia has been shown to prevent intubation in cer-
tylcholinesterase inhibitor use for patients with MG. Yet, tain cases.
other medications can result in a similar presentation (eg,
organophosphate poisoning). Serologic tests, electromyogra- Outcome and Prognosis
phy with repetitive stimulation, and single-​fiber electromy- About 40% of patients with generalized MG may have a
ography may allow for confirmation of the diagnosis. severe crisis at some point that requires endotracheal intu-
bation and mechanical ventilation, mostly in the first 2
Management years after onset. The quality of life after MG is largely
Cholinesterase inhibitors are not sufficient to treat myas- determined by the severity of muscle weakness (which
thenic crisis. In fact, their use is often stopped during may include neck muscle weakness and constant head
mechanical ventilation to improve secretion management. drop), dysphagia and chewing problems, ptosis, diplopia,
Copyright © 2021. Oxford University Press, Incorporated. All rights reserved.

Patients are best treated with PLEX or intravenous immu- and a speech impediment.
noglobulin. Corticosteroids may be given concurrently
with PLEX therapy. • Myasthenic crisis may be precipitated by certain
Frequent clinical assessments and bedside pulmonary medications, infection and fever, pregnancy, certain
function testing and ABG analysis are essential in a systemic illness, and recent thymectomy.
• A myasthenic crisis must be distinguished from a
cholinergic crisis because often, they both may present
with weakness.
Box 62.2 • Potential Risk Factors for Development • Patients with myasthenic crisis are best treated with
of Myasthenic Crisis several courses of plasma exchange or intravenous
Concurrent infection or fever
immunoglobulin.
Thymectomy or other surgery
Certain medications
Pregnancy Botulism
Systemic illness Clostridium botulinum is part of a family of bacteria that
are Gram positive, spore forming, and anaerobic.

Mayo Clinic Neurology Board Review, edited by Kelly D. Flemming, Oxford University Press, Incorporated, 2021. ProQuest Ebook Central, http://ebookcentral.proquest.com/lib/cuhk-ebooks/detail.action?docID=6746419.
Created from cuhk-ebooks on 2023-09-02 09:10:26.
Chapter 62. Neuromuscular Disease in the Neuroscience Intensive Care Unit 517

The toxin produced by these bacteria prevents acetylcho-


line exocytosis from the motor nerve terminals and
A
autonomic ganglia, resulting in motor weakness and
autonomic ­dysfunction.
Detection of the toxin in serum or isolation of the
spore in feces can be useful in making the diagnosis.
Treatment is supportive. Antitoxin can be delivered to
patients age 1 year and older and botulism immunoglobu-
lin given to infants younger than 1 year. This topic is cov-
ered in more detail in Chapter 88, “Neuromuscular
Junction Disorders.”

Critical Illness Neuropathy


Overview and Epidemiologic Factors
Critical illness neuropathy occurs in more than 50% of
patients with systemic inflammatory response syndrome. B
This syndrome may occur in patients with severe infec-
tion, septic shock, or severe trauma.

Clinical Manifestations
Because patients are critically ill, often the earliest indica-
tor of the disease is difficulty weaning from ventilator use.
Serious muscle weakness develops, and atrophy, often of
all 4 limbs (Figure 62.2), and respiratory muscle weakness
occur. Tendon reflexes are reduced, and distal sensation is
lost. This loss of distal sensation is sometimes difficult to
test in a critically ill patient. Cranial nerves often have nor-
mal responses.

Diagnosis
The diagnosis of critical illness neuropathy is made with
electromyography and laboratory studies. Nerve conduc-
tion studies show reduced amplitude of sensory nerve
action potentials (SNAPs) and compound muscle action
potentials (CMAPs), with relatively preserved distal
Copyright © 2021. Oxford University Press, Incorporated. All rights reserved.

latencies and conduction velocities. These findings are


suggestive of an axonal pattern. Sensory nerve responses
can be normal in the early stages. Needle examination
shows fibrillation potentials and reduced recruitment. A
concomitant myopathy sometimes occurs, evidenced by
rapid recruiting and short-​ duration motor unit poten-
tials. Creatine kinase and CSF values are usually at refer-
ence levels. Rarely, the CSF protein value is mildly
increased.
If pathologic examination is done, it shows axonal
degeneration of motor and sensory fibers. No inflammatory Figure 62.2 Patient With Polyneuropathy of Critical Illness.
infiltrates are seen. A, Marked atrophy of interossei. B, Marked atrophy of
tibialis and calf muscles.
Treatment and Prognosis (From Wijdicks EFM. Neurologic complications of critical illness.
3rd ed. Oxford [UK] & New York [NY]: Oxford University Press;
Treatment is supportive. Recovery occurs over months and c2009. Chapter 4, Generalized weakness in the intensive care unit;
may not be complete. p. 59-​72. [Contemporary Neurology series; 74]; used with permission
of Mayo Foundation for Medical Education and Research.)

Mayo Clinic Neurology Board Review, edited by Kelly D. Flemming, Oxford University Press, Incorporated, 2021. ProQuest Ebook Central, http://ebookcentral.proquest.com/lib/cuhk-ebooks/detail.action?docID=6746419.
Created from cuhk-ebooks on 2023-09-02 09:10:26.
518 Section IV. Neurologic Intensive Care Disorders

• Critical illness neuropathy is rare, but it may occur in weaning from use of the ventilator. Facial weakness, but
more than 50% of patients with systemic inflammatory rarely oculomotor weakness, can occur. Sensory changes,
response syndrome. if able to be tested, are typically absent. However, critical
• Often the earliest indicator of the disease is difficulty illness myopathy and neuropathy may overlap, and thus
weaning from use of the ventilator in the ICU. sensory abnormalities may occur in the latter.
• Electromyography in critical illness neuropathy may
show reduced SNAPs and CMAPs with preserved Diagnosis
distal latencies suggestive of an axonal neuropathy.
The diagnosis is made with electromyography. Nerve con-
duction studies often show low-​amplitude, long-​duration
CMAPs with relative preservation of sensory responses.
Critical Illness Myopathy Needle electromyography shows increased insertional
activity with fibrillation potentials and positive sharp
Overview and Epidemiologic Factors
waves. The creatine kinase value may be increased.
Critical illness myopathy is often recognized in the ICU as
diffuse flaccid paresis and difficulty weaning from use of Treatment and Prognosis
the ventilator. It can occur in isolation or concomitantly
Supportive treatment and prevention of complications are
with critical illness neuropathy. Critical illness myopathy
the mainstays of management.
often develops in the clinical setting of multiorgan failure
(renal) and sepsis or systemic inflammatory response.
Additional risk factors include corticosteroid use, use of
• Critical illness myopathy is often recognized in the ICU
as diffuse flaccid paresis and difficulty weaning from
neuromuscular junction blocking agents, chronic obstruc-
use of the ventilator.
tive pulmonary disease, and liver failure.
• Risk factors for critical illness myopathy include
multiorgan failure, sepsis, corticosteroid use, use of
Clinical Manifestations
neuromuscular junction blocking agents, and chronic
Patients typically have diffuse weakness of all 4 limbs that obstructive pulmonary disease.
is proximal more than distal, and they have difficulty
Copyright © 2021. Oxford University Press, Incorporated. All rights reserved.

Mayo Clinic Neurology Board Review, edited by Kelly D. Flemming, Oxford University Press, Incorporated, 2021. ProQuest Ebook Central, http://ebookcentral.proquest.com/lib/cuhk-ebooks/detail.action?docID=6746419.
Created from cuhk-ebooks on 2023-09-02 09:10:26.
Acute Hyperthermia Syndromes
63 MAXIMILIANO A. HAWKES, MD; EELCO F. M. WIJDICKS, MD, PHD

Introduction and motor tone. Serotonin syndrome may occur after use
of the combination of medications that block serotonin

A
high core body temperature is a medical emer- metabolism, an effect that leads to excess extracellular 5-​
gency. Neuronal mitochondria and plasma mem- HT (Figure 63.1). For most patients, the syndrome devel-
branes are thought to undergo protein changes at ops within 6 hours after the initial use of the medication,
temperatures higher than 40°C, which lead to brain dam- an overdose, or an increase in dosing.
age or dysfunction. In addition, the effects of hyperther- A thorough review of a patient’s medications is essen-
mia on an already injured brain cause further injury. tial, assessing for use of SSRIs, triptans, opioids, metoclo-
Control of fever is one of the important additional inter- pramide, ondansetron, or inhibitors of cytochrome P450
ventions necessary to control increased intracranial pres- 2D6 and 3A4. Severe cases often involve unsuspected use
sure and status epilepticus and to treat hypertensive of monoamine oxidase inhibitors, such as linezolid or
surges in patients with sympathetic system overdrive. methylene blue (Box 63.1).
In hyperthermia, the thermoregulatory control mecha-
nisms are impaired or overwhelmed, in contrast to fever, Clinical Manifestations
in which the hypothalamic set point is increased.
Hyperthermia can be caused by conditions such as thyro- Mild cases present with tachycardia, shivering, diaphore-
toxicosis, pheochromocytoma, aneurysmal subarachnoid sis, and mydriasis. Tremor, myoclonus, and hyperreflexia
hemorrhage, and meningitis. Drugs that impair thermoreg- are common. Hypertension, hyperthermia, and sounds of
ulation also can have a role. They include amphetamines, a hyperactive bowel complicate the moderate cases.
cocaine, opiates, antihistamines, selective serotonin reup- Hyperreflexia, myoclonus, rigidity, and clonus are strik-
take inhibitors (SSRIs), β-​blockers, and diuretics. ing features that may occur in serotonin syndrome, in
In this chapter, we discuss several acute hyperthermia addition to agitated delirium. Life-​ threatening cases
involve seizures, shock, metabolic acidosis, rhabdomyol-
Copyright © 2021. Oxford University Press, Incorporated. All rights reserved.

syndromes (Table 63.1).


ysis, renal failure, and disseminated intravascular coagu-
lation, which develop in part because of uncontrolled
­hyperthermia.
Serotonin Syndrome
Epidemiologic Factors Treatment
A monoamine, 5-​ hydroxytryptamine (5-​ HT) is derived Treatment is supportive and includes discontinuation of
from tryptophan. Serotoninergic neurons are located in the the precipitating drugs—​a simple approach that is often
brainstem raphe nuclei. This serotoninergic system assists forgotten. In addition, aggressive control of cardiorespira-
in the regulation of wakefulness, affective behavior, food tory and thermal abnormalities is needed. The 5-​ HT2A
intake, thermoregulation, migraine, emesis, nociception, antagonist cyproheptadine is often used for intubated

The editors and authors acknowledge the contributions of Philippe Couillard, MD, to the previous edition of this chapter.
Abbreviations: 5-​HT, 5-​hydroxytriptamine; NMS, neuroleptic malignant syndrome; SSRI, selective serotonin reuptake inhibitor

519

Mayo Clinic Neurology Board Review, edited by Kelly D. Flemming, Oxford University Press, Incorporated, 2021. ProQuest Ebook Central, http://ebookcentral.proquest.com/lib/cuhk-ebooks/detail.action?docID=6746419.
Created from cuhk-ebooks on 2023-09-02 09:10:26.
520 Section IV. Neurologic Intensive Care Disorders

Table 63.1 • Comparison of the Acute Hyperthermia Syndromes


Condition Precipitants Clinical Characteristics Treatment

Serotonin syndrome Drugs resulting in increased Hyperthermia Removal of provoking drug(s)


serotonin (Box 63.1) Hyperreflexia Supportive treatment
Tremor, clonus 5-​hydroxytriptamine2A antagonist
Myoclonus Cyproheptadine or sedation for intubated
Mental status changes patients
Neuroleptic Neuroleptic agents or sudden Hyperthermia Removal of offending agent
malignant withdrawal of a dopamine Tachycardia, diaphoresis Supportive treatment
syndrome agent Rigidity, tremor Bromocriptine, dantrolene, or amantadine
Mental status changes can be considered
Dysphagia Electroconvulsive therapy
Mutism
Paroxysmal Brain trauma Hyperthermia Supportive treatment
sympathetic Severe brain injury (stroke) Tachycardia Morphine, β-​blockers, gabapentin
hyperactivity Hypertension
Tachypnea
Excessive sweating
Dystonic posturing
Malignant Succinylcholine in the clinical Hyperthermia Removal of offending agent
hyperthermia setting of an abnormal Muscle spasms Cooling
ryanodine receptor Trismus Dantrolene

patients, along with sedation and neuromuscular paraly- The syndrome may mimic shock. Leukocytosis and
sis. Use of cyproheptadine is guided by the severity of increased creatine kinase are present. The occurrence of
rigidity (and serum creatine phosphokinase level). Seizures NMS is unpredictable. Its pathophysiologic mechanism is
are treated with intravenous benzodiazepines, which also not fully understood, but dopamine 2 receptor blockade
may help control agitation. has been implicated.

• Serotonin syndrome is often characterized with Treatment


tachycardia, hypertension, hyperthermia, hyperreflexia,
and rigidity. Treatment is supportive: cooling, hydration, and discon-
tinued use of the offending drugs. Bromocriptine, dan-
trolene, and amantadine have been shown to improve
Neuroleptic Malignant Syndrome outcome over supportive care alone. In life-​ threatening
cases, neuromuscular blockade prevents heat generation.
Clinical Manifestations Lorazepam, clonidine, and electroconvulsive therapy are
Copyright © 2021. Oxford University Press, Incorporated. All rights reserved.

Neuroleptic malignant syndrome (NMS) is suspected possible approaches, coupled with discontinuation of drug
when a patient has sudden acute rigidity and increased use and aggressive supportive care.
core temperature and has taken a dopaminergic agent (or
abruptly withdrawn use of dopamine). The onset may not • Neuroleptic malignant syndrome is suspected in a
always be abrupt, and milder forms have been described. patient who has sudden acute rigidity and increased
Various changes in level of consciousness, ranging from core temperature and has used a dopaminergic agent
confusion to coma, are often present. Basal ganglia dys- (or abruptly withdrawn use of dopamine).
function takes the form of tremor, dystonia, and bradyki- • Neuroleptic malignant syndrome is characterized by
netic characteristics, such as dysphagia, sialorrhea, and hyperthermia and rigidity.
mutism. • Bromocriptine, dantrolene, and amantadine have been
When NMS is severe, autonomic dysfunction is present shown to improve outcome over supportive care alone
with diaphoresis, tachycardia, and labile blood pressure. in patients with neuroleptic malignant syndrome.

Mayo Clinic Neurology Board Review, edited by Kelly D. Flemming, Oxford University Press, Incorporated, 2021. ProQuest Ebook Central, http://ebookcentral.proquest.com/lib/cuhk-ebooks/detail.action?docID=6746419.
Created from cuhk-ebooks on 2023-09-02 09:10:26.
Chapter 63. Acute Hyperthermia Syndromes 521

Normal Presynaptic neuron

Synaptic cleft

Postsynaptic neuron

Symptoms of
serotonin syndrome
Rigidity
Dysautonomia
Myoclonus
Agitation

SSRI

© MAYO
2017

Figure 63.1 Serotonin Syndrome: Blocking of Uptake Receptor by SSRIs. The SSRI blocking action results in increased
Copyright © 2021. Oxford University Press, Incorporated. All rights reserved.

5-​hydroxytriptamine levels and clinical symptoms. SSRI indicates selective serotonin reuptake inhibitors.

Paroxysmal Sympathetic Hyperactivity sudden onset, episodes may be mislabeled as seizure or


simply considered a manifestation of the primary brain
Clinical Manifestations disorder.
Episodes of paroxysmal sympathetic hyperactivity gener-
ally occur in patients with severe traumatic brain injury Treatment
or, less frequently, hypoxic-​ischemic encephalopathy or Management of paroxysmal sympathetic hyperactivity
stroke. Whether this hyperactivity is triggered by stimula- includes adequate hydration, effective analgesia, and
tion or not, its manifestations include hyperthermia, avoidance of such triggers as frequent body positioning
tachycardia, hypertension, tachypnea, excessive sweat- and frequent tracheal suctioning. Morphine sulfate, nonse-
ing, and dystonic posturing. Because of the condition’s lective β-​
blockers, and gabapentin are first-​
line therapy.

Mayo Clinic Neurology Board Review, edited by Kelly D. Flemming, Oxford University Press, Incorporated, 2021. ProQuest Ebook Central, http://ebookcentral.proquest.com/lib/cuhk-ebooks/detail.action?docID=6746419.
Created from cuhk-ebooks on 2023-09-02 09:10:26.
522 Section IV. Neurologic Intensive Care Disorders

Second-​line agents include baclofen, bromocriptine, and


Box 63.1• Drugs and Medications Sometimes clonidine.
Associated With Serotonin Syndrome
• Episodes of paroxysmal sympathetic hyperactivity
Drugs of abuse generally occur in patients with severe traumatic brain
Amphetamines injury or, less frequently, hypoxic-​ischemic
Cocaine encephalopathy or stroke.
3,4-​methylenedioxymethamphetamine (also called
ecstasy)
Lysergic acid diethylamide (LSD)
Malignant Hyperthermia
Pain medications
Tramadol Pathophysiologic Factors
Meperidine Malignant hyperthermia is a muscle disorder caused by a
Pentazocine mutation in the ryanodine receptor. An increased tendency
Fentanyl for excessive calcium release from the sarcoplasmic reticu-
Psychiatric medications lum is the key response to exposure to a trigger such as
Selective serotonin reuptake inhibitors (eg, inhaled anesthetics or the muscle relaxant succinylcho-
citalopram, escitalopram, fluoxetine, fluvoxamine, line. Calcium excess leads to muscle hyperactivity and
paroxetine, sertraline) hypermetabolism, followed by fever and rhabdomyolysis.
Serotonin norepinephrine reuptake inhibitors (eg, Uncoupling of oxidative phosphorylation with metabolic
desvenlafaxine, duloxetine, venlafaxine) breakdown activates the sympathetic nervous system and
Dopamine norepinephrine reuptake inhibitors (eg, causes catecholamine release.
bupropion)
Tricyclic antidepressants (eg, amitriptyline, Clinical Manifestations
amoxapine, clomipramine, desipramine, doxepin,
imipramine, maprotiline, nortriptyline, Generalized muscle spasms, trismus, cutaneous vasocon-
protriptyline, trimipramine) striction, and increased systemic vascular resistance, car-
Serotonin-​modulating agents (eg, nefazodone, diac output, and minute ventilation are the presenting
trazodone, vilazodone)
symptoms and signs of malignant hyperthermia. Adenosine
Monoamine oxidase inhibitors triphosphate production is overwhelmed and ceases, the
Lithium result of which is failure of membrane pumps, leakage of
Amphetamines electrolytes, increased creatine kinase value, myoglobin-
Antiseizure medications uria with cardiac arrhythmia, and death.
Valproate
Carbamazepine Treatment
Other neurologic medications The mainstay of treatment is discontinuation of use of the
Levodopa/​carbidopa offending agent, cooling, cardiovascular support, renal and
Triptans electrolyte management, and administration of dantrolene.
Ergotamine This disorder is more often seen by anesthesiologists than
Copyright © 2021. Oxford University Press, Incorporated. All rights reserved.

Cyclobenzaprine neurologists because of its association with certain types of


Dextromethorphan anesthetic agents.
Gastrointestinal tract medications
Metoclopramide • Malignant hyperthermia is a muscle disorder caused by
Ondansetron a mutation in the ryanodine receptor.
Over-​the-​counter medications or supplements
• In susceptible patients, it can be triggered by certain
inhaled anesthetics or succinylcholine.
St. John’s wort
• Malignant hyperthermia clinically manifests with
Ginseng
hyperthermia, muscle spasm, and trismus. It can
Turmeric deteriorate to cardiac arrhythmia and death.
Nutmeg

Mayo Clinic Neurology Board Review, edited by Kelly D. Flemming, Oxford University Press, Incorporated, 2021. ProQuest Ebook Central, http://ebookcentral.proquest.com/lib/cuhk-ebooks/detail.action?docID=6746419.
Created from cuhk-ebooks on 2023-09-02 09:10:26.
Questions and Answers

Questions IV.6. Which of the following statements is true about Cheyne-​Stokes


respiration?
Multiple Choice (choose the best answer) a. It suggests a supratentorial mass as a cause
b. It is characterized by erratic, irregular bouts of respiration
IV.1. Which of the following examination findings is confirmative of c. It occurs in patients with medullary lesions
brain death? d. It is always due to a brain mass
a. Absent motor response, absent verbal response, and intact corneal e. It is consistent with a poor prognosis
reflex IV.7. An intracranial pressure (ICP) monitor placed in a female patient
b. Presence of alpha waves on an electroencephalogram with a hemorrhage in a brain tumor shows an ICP of 30 mm Hg after
c. During an apnea test, no breathing effort observed when Pco2 is a ventriculostomy is positioned. The patient’s mean arterial pressure
greater than 60 mm Hg or increases 20 mm Hg from baseline (MAP) is 90 mm Hg. What is her cerebral perfusion pressure (CPP)?
d. Core body temperature less than 32°C a. 20 mm Hg
e. Presence of decerebrate posturing b. 60 mm Hg
IV.2. In confirming brain death, which of the following should be cor- c. 120 mm Hg
rected before an apnea test? d. Cannot calculate CPP because the autoregulation curve has
a. Core body temperature greater than 36°C shifted to the right
b. Pulse rate less than 100 beats per minute e. Cannot calculate CPP without cerebral blood flow
c. Head of bed elevated to 30° IV.8. A patient with a rapidly expanding right frontal cortical mass
d. Correction of coagulopathies undergoes surgery and the mass is removed. On awakening from
e. Absence of antipyretic medications surgery, the patient has reduced motivation to eat and speak and
IV.3. A 23-​year-​old farm worker is brought to the emergency depart- seems to stare blankly. He can name items and comprehend sim-
ment after being found unresponsive. The patient is generally ple commands. What is a potential cause of these new symptoms?
weak with pinpoint pupils, diaphoresis, and excessive salivation. a. Subfalcine herniation
Glucose is at the reference level. Naloxone results in no response. b. Downward herniation before surgery resulting in mutism from
Head computed tomography is negative. The most likely cause brainstem injury
of this patient’s coma is which of the following? c. Tonsillar herniation with cerebellar mutism
a. Methanol poisoning d. Uncal herniation with resultant aphasia
b. Organophosphate poisoning e. Upward herniation
c. Hallucinogen overdose IV.9. The Cushing reflex consists of which of the following items?
Copyright © 2021. Oxford University Press, Incorporated. All rights reserved.

d. Carbon monoxide exposure a. Cerebral perfusion pressure = mean arterial pressure –​intracra-
e. Tricyclic antidepressant overdose nial pressure (ICP)
IV.4. A comatose patient is intubated and triggers the ventilator. b. Total intracranial content volume = blood + cerebrospinal fluid
Motor response is decorticate. Pupils are midposition and fixed. + brain volume
The horizontal oculocephalic reflex is intact. The examination is c. Hypertension, tachycardia, and Cheyne-​Stokes breathing
suggestive of a supratentorial lesion with transtentorial herni­ d. Increased ICP, reduced cerebral blood flow, and elevated P1
ation affecting which of the following structure(s)? waveform
a. Midbrain e. Irregular respirations, bradycardia, and hypertension
b. Midbrain and pons IV.10. Which of the following therapies could be used to treat refrac-
c. Midbrain, pons, and medulla tory increased intracranial pressure (ICP) due to malignant cere-
d. Midbrain, pons, medulla, and cervicomedullary junction bral ischemia?
e. Midbrain, pons, and cerebellum a. High-​dose corticosteroids
IV.5. After assessment of airway, breathing (oxygen), and circulation, b. Prolonged hyperventilation
what measure(s) should be considered immediately for a patient c. Hypertonic saline
with coma? d. Decompressive craniotomy
a. Urine toxin screen e. Propofol
b. Finger-​stick glucose, thiamine, naloxone, and intravenous glucose IV.11. Which of the following statements about status epilepticus (SE)
c. Head computed tomography without contrast medium is true?
d. Point-​of-​care international normalized rate a. The majority of SE episodes occur in patients with no history of
e. Serum electrolytes and kidney and liver function seizures

523

Mayo Clinic Neurology Board Review, edited by Kelly D. Flemming, Oxford University Press, Incorporated, 2021. ProQuest Ebook Central, http://ebookcentral.proquest.com/lib/cuhk-ebooks/detail.action?docID=6746419.
Created from cuhk-ebooks on 2023-09-02 09:10:26.
524 Section IV. Neurologic Intensive Care Disorders

b. The highest incidence occurs within the first year of life and after c. The anterior cerebral artery territory
age 60 years d. The posterior fossa
c. SE is diagnosed after 30 minutes of continued seizure activity e. The watershed distribution between the anterior and middle
d. Most seizures, if untreated, lead to SE cerebral artery territories
e. The diagnosis of SE relies on electroencephalography IV.20. Which of the following statements is true regarding the utility of
IV.12. For a patient with status epilepticus (SE), what is the next step tests in the assessment of prognosis for patients with anoxic
after assessment of airway, breathing, and circulation? encephalopathy?
a. Establish intravenous access a. Neuroimaging is not useful in prognostication
b. Administer fosphenytoin 20 mg/​kg intramuscularly b. Somatosensory-​evoked potential (SSEP) may be influenced by
c. Assess a finger-​stick glucose and oxygen saturation hypothermia and medications used in an intensive care unit and
d. Administer 2 mg of lorazepam intramuscularly should not be used for prognostication
e. Obtain an arterial blood gas c. Computed tomographic scan of the brain is 85% sensitive and
IV.13. A 60-kg patient with generalized convulsive status epilepticus 90% specific in identification of anoxic brain injury
receives 6 mg of lorazepam and 20 mg/kg of intravenous fos- d. A neuron-​specific enolase (NSE) value greater than 33 μg/​L at day
phenytoin. The patient is intubated with etomodate, fentanyl, 1 is uniformly predictive of a poor prognosis
and recuronium, and an electroencephalography monitor is e. An electroencephalography (EEG) showing burst suppression is
placed showing frequent generalized seizures. The next best suggestive of a poor prognosis
choice of therapy is which of the following? IV.21. A 15-​year-​old boy is struck on the left side of the head by an 80-​
a. Lacosamide mile-​per-​hour baseball pitch. Although initially he dropped to
b. Levetiracetam the ground, he got up and was able to walk to the ambulance. On
c. Inhalation anesthetic arrival to the emergency department, his level of consciousness
d. Vagal nerve stimulation deteriorates. A computed tomographic (CT) scan is shown in the
e. Midazolam image below. The best course of action is which of the following?
IV.14. Which of the following medications should be avoided as an
antiepileptic agent for aneurysmal subarachnoid hemorrhage
because of its potential systemic adverse effects?
a. Levetiracetam
b. Phenytoin
c. Lacosamide
d. Lamictal
e. Phenobarbital
IV.15. Abrupt, early hydrocephalus typically presents as which of the
following conditions?
a. Cranial nerve VII palsy
b. Nystagmus
c. Impaired consciousness
d. Horner’s syndrome
e. Hemiparesis
IV.16. The percentage of patients who regain functional independence
within the first year after aneurysmal subarachnoid hemor-
rhage is approximately which of the following ranges?
a. 1% to 2%
b. 5% to 10%
c. 10% to 20%
d. 30% to 50%
e. 60% to 70%
IV.17. For a patient with aneurysmal subarachnoid hemorrhage and
Copyright © 2021. Oxford University Press, Incorporated. All rights reserved.

hyponatremia, all of the following steps are indicated except


which step?
a. Free water restriction
b. Use of fludrocortisone acetate
c. Use of hypertonic saline solution
d. Use of normal saline a. Burr hole evacuation
e. Liberal free water intake b. Emergent craniotomy and evacuation
IV.18. Which of the following might confer a better prognosis in c. No surgery; admission to the neuroscience intensive care unit
anoxic-​ischemic injury? and administration of mannitol
a. Burst suppression pattern on electroencephalography d. No surgery; admission to the general neurology floor service for
b. Localization to painful stimuli close observation
c. Serum neuron-​specific enolase of 80 μ/​L at day 1 to day 3 after e. No surgery; admission to the general neurology floor service and
cardiac arrest administration of intravenous corticosteroids
d. Sustained upward gaze IV.22. A 37-​year-​old man involved in a motor vehicle crash sustains
e. Status myoclonus facial trauma and a brief loss of consciousness. He now reports
IV.19. Man-​in-​the-​barrel syndrome localizes to which of the following headache, nausea, and clear nasal drainage. To determine
areas? whether the nasal drainage is due to cerebrospinal fluid (CSF)
a. The watershed distribution between the middle and posterior leak, which of the following tests can be used on the fluid?
cerebral artery territories a. Vascular endothelial growth factor
b. The middle cerebral artery territory b. β-​2 Transferrin

Mayo Clinic Neurology Board Review, edited by Kelly D. Flemming, Oxford University Press, Incorporated, 2021. ProQuest Ebook Central, http://ebookcentral.proquest.com/lib/cuhk-ebooks/detail.action?docID=6746419.
Created from cuhk-ebooks on 2023-09-02 09:10:26.
Questions and Answers 525

c. Neuron-​specific enolase e. After the patient is intubated, bedside pulmonary function tests
d. Glial fibrillary acidic protein should be measured daily to evaluate disease progression and
e. Xanthochromia readiness for extubation
IV.23. Which of the following statements is true about concussion? IV.29. A 70-​kg patient comes into the emergency department with
a. By definition of concussion, patients must lose consciousness, ascending weakness over the past 3 days and shortness of
even if briefly breath. Respiratory rate is about 25 breaths per minute. Pulse is
b. A direct head strike is required 100 beats per minute. Chest radiography is negative for con-
c. Patients must have associated nausea and vomiting cerns. The patient’s vital capacity is 1,050 mL; inspiratory pres-
d. The Glasgow Coma Scale (GCS) score must be 15 sure, ​–20 cm H2O; and expiratory pressure, 30 cm H2O. What is
e. Concussion can occur in a patient who sustains body trauma that important in the next steps of treatment of this patient?
transmits an impulsive force to the head a. Spine imaging
IV.24. Which of the following statements is true regarding assessment b. Admission to general floor with daily assessment of vital capac-
of concussion in the emergency department? ity, inspiratory, and expiratory pressures
a. Patients younger than 15 years should be admitted and observed c. Spinal fluid evaluation
for postconcussion brain swelling d. Arterial blood gas and consideration of intubation
b. Patients should undergo neurologic evaluation that includes e. Assessment of urine toxicology screen
assessment of cognition and a computed tomographic (CT) scan IV.30. Which of the following characteristics might distinguish a cho-
of the head linergic crisis from a myasthenic crisis?
c. Blood should be drawn for serum biomarkers to determine a. Respiratory failure
which patients should be admitted and observed b. Generalized weakness, including distal muscles
d. Selected patients should undergo CT scan of the brain c. Presence or absence of excessive salivation and diarrhea
e. Seizure prophylaxis for 3 days should be implemented d. Presence or absence of reflexes
IV.25. A herniated disk at the L5-​S1 interspace is compressing the con- e. Presence or absence of ptosis
tents of the spinal canal. Which of the following characteristics IV.31. Which of the following complications is not common in Guillan-​
would not be typically associated with this lesion? Barré syndrome (GBS)?
a. Asymmetrical weakness a. Takotsubo cardiomyopathy
b. Upgoing plantar responses b. Arrhythmia
c. Back pain (lumbago) c. Deep vein thrombosis
d. Reduced ankle (Achilles) reflexes d. Paroxysmal fluctuations in blood pressure
e. Saddle anesthesia e. Respiratory failure
IV.26. A man is stabbed in the chest near his axilla and has arm weak- IV.32. Which of the following drugs could increase the risk of sero-
ness. What is the best course of action? tonin syndrome in a patient already taking a selective serotonin
a. Cast immobilization reuptake inhibitor (SSRI)?
b. Nonsteroidal anti-​inflammatory drug therapy a. Quetiapine
c. Urgent surgical consultation b. Phenytoin
d. Physical therapy c. Tramadol
e. Traction d. Marijuana
IV.27. A man who fell from a grain elevator had a complete spinal cord e. Levetiracetam
injury at the T1 level (American Spinal Cord Injury Association IV.33. A 75-​year-​old patient with advanced Parkinson disease is admit-
A). Twenty years later, he has progressive bilateral hand weak- ted with abdominal pain, nausea, and vomiting. He receives the
ness first and then has arm weakness. Which of the following is diagnosis of small-​bowel obstruction and is administered noth-
the most likely cause of his new symptoms? ing by mouth (NPO). On the second day of admission, the patient
a. Ligamentous laxity has hyperthermia, rigidity, and mental status changes. The most
b. Scoliosis likely cause of these symptoms is which of the following?
c. Cervical radiculopathies due to spine instability a. Concomitant pneumonia
d. Syringomyelia b. Reaction to the antiemetic
e. Amyotrophic lateral sclerosis c. Abrupt withdrawal of dopamine
Copyright © 2021. Oxford University Press, Incorporated. All rights reserved.

IV.28. Which of the following statements about Guillain-​Barré syn- d. Malignant hyperthermia
drome (GBS) is not true? e. Ischemic stroke
a. The presence of deep tendon reflexes in the initial evaluation IV.34. Which of the following statements is true about the cause of
rules out GBS malignant hyperthermia?
b. The combination of intravenous immunoglobulin (IVIG) and a. Abrupt withdrawal of dopamine may cause this syndrome
plasma exchange (PLEX) is superior to either treatment alone b. A mutation in the ryanodine receptor in combination with suc-
c. Noninvasive ventilation should be avoided in the treatment of cinylcholine use may cause this syndrome
respiratory failure c. Cocaine use may result in this syndrome
d. High cellularity in the cerebrospinal fluid should prompt ques- d. Intramuscular haloperidol treatment may result in this ­syndrome
tioning of the diagnosis of GBS e. This disorder may result from severe head trauma

Mayo Clinic Neurology Board Review, edited by Kelly D. Flemming, Oxford University Press, Incorporated, 2021. ProQuest Ebook Central, http://ebookcentral.proquest.com/lib/cuhk-ebooks/detail.action?docID=6746419.
Created from cuhk-ebooks on 2023-09-02 09:10:26.
526 Section IV. Neurologic Intensive Care Disorders

Answers IV.10. Answer d.


If ICP does not decrease with the simple interventions, a
IV.1. Answer c. large bone flap must be removed, allowing brain swelling to
Brain death is the irreversible cessation of all brain and occur without the confinement of the skull and thereby
brainstem function due to a known, irreversible injury. reducing ICP.
When an apnea test is performed to confirm brain death, the IV.11. Answer b.
arterial blood gas test is considered positive if no breathing SE is a medical and neurologic emergency defined as per-
effort occurs when the arterial Pco2 is greater than 60 mm sistent seizure activity lasting more than 5 minutes or recur-
Hg or there is an increase of 20 mm Hg in Pco2 above a nor- rent seizures without recovery of consciousness in between.
mal baseline value. The highest incidence of SE is within the first year of life
IV.2. Answer a. and after age 60.
Requirements for the apnea test, in addition to a general IV.12. Answer c.
brain death examination to exclude brain or brainstem Although many activities occur simultaneously during sta-
functioning, include a core temperature of more than 36°C, bilization of a patient with SE, immediate assessment of a
a positive fluid balance and no recent polyuric episode, finger-​stick glucose and oxygen saturation may help iden-
Paco2 of more than 40 mm Hg, and Pao2 of more than 200 tify a potentially reversible cause of SE. While these initial
mm Hg. Severe metabolic derangements should be cor- assessments are occurring, establishment of intravenous
rected with replacement or toxin removal as necessary; access is crucial.
there should be no effect of sedating or paralytic drugs. IV.13. Answer e.
Systolic blood pressure should be greater than 100 mm Hg. If seizures persist after adequate doses of benzodiazepine and
No pulse rate requirements are noted. of fosphenytoin or valproic acid, consideration should be
IV.3. Answer b. given to intubation, patient transfer to an intensive care unit,
Patients with organophosphate poisoning may present with infusion of an anesthetic agent (midazolam, propofol,
coma in addition to miosis, urination, diarrhea, diaphore- ­ketamine, or pentobarbital), and continuous electroencepha-
sis, lacrimation, excitation, and salivation (mnemonic lography. Alternative antiepileptic agents, including lacos-
MUDDLES). amide and levetiracetam, are currently not recommended as
IV.4. Answer a. third-line agents for control of convulsive SE. Although they
Midposition, fixed pupils indicate disruption of parasym- are widely used, their effectiveness and safety are not clearly
pathetic and sympathetic tone and disruption of the effer- established in this clinical setting. Inhalation anesthetic or
ent pathway of the pupillary light reflex (cranial nerve III). surgical procedures are reserved for selected cases of super
They can be indicative of severe midbrain dysfunction, refractory SE.
such as dysfunction from an infarct or transtentorial hernia- IV.14. Answer b.
tion. Normal horizontal oculocephalic reflex is suggestive Phenytoin should be avoided in patients with aneurysmal
that the afferent limb (cranial nerve VIII) and efferent limb subarachnoid hemorrhage because some evidence suggests
(cranial nerve VI) of this pathway are intact. These cranial that worse functional and cognitive outcomes at 14 to 30 days
nerves are at the level of the pons. Decorticate posturing is are associated with its use. Additional studies have suggested
generally seen with a lesion above the red nucleus. With all a higher adverse effect profile and more drug-​to-​drug interac-
pieces of the patient parameters taken together, the mid- tions than levetiracetam. In addition, pheny­toin induces the
brain is dysfunctional, but the pons and medulla are intact. metabolism of nimodipine, which may decrease its effect.
IV.5. Answer b. IV.15. Answer c.
For any patient with coma, many tests are simultaneously One of the first signs of abrupt-onset hydrocephalus is a reduced
performed, and it may be important to get all the tests listed level of consciousness. Patients may also have diplopia, vomit-
above as choices. However, it will take time for the results of ing, and vertical gaze restriction.
serum and urine laboratory studies. Of importance are early IV.16. Answer d.
assessment of a finger-​stick glucose level and use of intrave- Within the first year after aneurysmal subarachnoid hemor-
nous thiamine, naloxone, and glucose (50% dextrose), rhage, 30% to 50% of patients regain independence.
which potentially reverse life-​threatening causes of coma. IV.17. Answer e.
Copyright © 2021. Oxford University Press, Incorporated. All rights reserved.

IV.6. Answer b. Hyponatremia due to cerebral salt wasting occurs in


Cheyne-​ Stokes breathing is characterized by a progressive approximately 20% to 40% of patients with aneurysmal
increase in the respiratory volume and rate followed by a subarachnoid hemorrhage. Free water intake will worsen
gradual decrease in the rate and volume, and it may lead to the hyponatremia. The use of fludrocortisone acetate and
brief periods of apnea. It generally is nonlocalizing and can be hypertonic saline is reasonable for prevention and correc-
seen in patients with lesions in multiple locations and with tion of hyponatremia.
nonlesional coma (ie, toxic/​metabolic or systemic illnesses). IV.18. Answer b.
IV.7. Answer b. A localizing motor response may confer a better prognosis,
The calculation CPP = MAP –​ICP applies in this case: CPP whereas the other options may be predictive of a poor neu-
= 90 –​30, or 60 mm Hg. Typically, CPP is between 50 and rologic prognosis after cardiac arrest.
150 mm Hg in a normotensive patient. IV.19. Answer e.
IV.8. Answer a. The man-​in-​the-​barrel syndrome clinically presents as bilat-
Patients with frontal masses can have subfalcine herniation eral proximal upper-​extremity weakness and localizes to the
that results in anterior cerebral artery infarction. This watershed distribution between the anterior and middle
infarction may cause contralateral leg paralysis. If the pre- cerebral artery vascular territories. Hypoperfusion, often in
frontal cortex is affected, abulia is common. the setting of bilateral carotid disease, may cause infarction
IV.9. Answer e. in this territory.
The Cushing reflex consists of hypertension, bradycardia, IV.20. Answer e.
and abnormal breathing (which may include Cheyne-​Stokes A highly malignant pattern on EEG such as burst suppres-
breathing). sion suggests a poor prognosis. Magnetic resonance imaging

Mayo Clinic Neurology Board Review, edited by Kelly D. Flemming, Oxford University Press, Incorporated, 2021. ProQuest Ebook Central, http://ebookcentral.proquest.com/lib/cuhk-ebooks/detail.action?docID=6746419.
Created from cuhk-ebooks on 2023-09-02 09:10:26.
Questions and Answers 527

can document the extent of anoxic-​ischemic injury with IV.31. Answer a.


diffusion-​weighted sequences and has some value in prog- Patients with GBS may have serious cardiac complica-
nostication. SSEPs are not influenced by drugs, tempera- tions—​ most commonly tachyarrhythmias and fluctuating
ture, or acute metabolic derangements. The absence of both blood pressure—​due to autonomic dysfunction. Heart fail-
N20s implies an invariably poor prognosis, and the patient ure is less common, and takotsubo cardiomyopathy has
likely will never regain consciousness. A serum NSE level been associated with GBS only in case reports. Deep vein
greater than 33 μg/​L at day 1 through day 3 after cardiac thrombosis, respiratory failure, adynamic ileus, urinary
arrest was traditionally a predictor of poor outcome. retention, and pain are common complications of GBS.
However, several studies have confirmed that this cutoff IV.32. Answer c.
level for NSE is not reliable for patients who have under- Many substances (see Box 63.1, “Drugs and Medications
gone hypothermia protocols. (See also Table 59.2.) Sometimes Associated With Serotonin Syndrome”) may
IV.21. Answer b. contribute to serotonin syndrome. Of the list provided, tra-
CT scan of the brain shows an acute epidural hemorrhage on madol can increase the risk of serotonin syndrome in a
the left side with overlying soft tissue swelling. The condition patient already taking an SSRI.
of such patients may deteriorate rapidly, and emergent craniot- IV.33. Answer c.
omy and evacuation of the epidural hemorrhage are warranted. A patient with advanced Parkinson disease is likely taking
IV.22. Answer b. levodopa/​ carbidopa. When the patient has the clinical
The fluid can be tested for β-​2 transferrin. This protein is order for NPO, the abrupt withdrawal of dopamine may
found only in CSF and perilymph. result in a neuroleptic malignant–​ type clinical picture.
IV.23. Answer e. Resumption of the medication would be recommended.
Although the exact definition of concussion differs across IV.34. Answer b.
medical societies, the American Academy of Neurology Malignant hyperthermia is a muscle disorder caused by a
defines traumatic brain injury concussion as “trauma-​ mutation in the ryanodine receptor. In susceptible patients,
induced alteration in mental status that may or may not it can be triggered by certain inhaled anesthetics or
involve loss of consciousness.” Concussions do not require ­succinylcholine.
loss of consciousness or a direct head strike. The GCS score
is generally 15 but can range from 13 to 15. SUGGESTED READING
IV.24. Answer d.
Carney N, Totten AM, O’Reilly C, Ullman JS, Hawryluk GW, Bell MJ,
Not all patients with concussion require a CT scan of the brain.
et al. Guidelines for the management of severe traumatic brain
The New Orleans Criteria (see Box 60.1, “Clinical
injury, fourth edition. Neurosurgery. 2017 Jan 1;80(1):6–15.
Characteristics in the New Orleans Criteria”) are highly sensi-
Connolly ES Jr, Rabinstein AA, Carhuapoma JR, Derdeyn CP, Dion
tive and specific for identifying patients with concussion or
J, Higashida RT, et al; American Heart Association Stroke
with mild traumatic brain injury who may have clinically
Council; Council on Cardiovascular Radiology and Intervention;
important intracranial lesions. This model includes 7 vari-
Council on Cardiovascular Nursing; Council on Cardiovascular
ables. If all 7 variables are absent, a head CT scan is not needed.
Surgery and Anesthesia; Council on Clinical Cardiology.
IV.25. Answer b.
Guidelines for the management of aneurysmal subarachnoid
Cauda equina syndrome does not produce upper motor
hemorrhage: a guideline for healthcare professionals from the
neuron signs.
American Heart Association/​ American Stroke Association.
IV.26. Answer c.
Stroke. 2012 Jun;43(6):1711–37. Epub 2012 May 3.
Clean, sharp, lacerating injuries to brachial plexus or termi-
Mattsson N, Zetterberg H, Nielsen N, Blennow K, Dankiewicz J,
nal nerve branches should be explored and repaired end to
Friberg H, et al. Serum tau and neurological outcome in cardiac
end within 24 to 48 hours because with time, the nerve end-
arrest. Ann Neurol. 2017 Nov;82(5):665–75. Epub 2017 Nov 2.
ings become more edematous and difficult to suture.
Rabinstein AA, editor. Neurological emergencies: a practical
Ultimately, they scar and degenerate, requiring more
approach. 1st ed. Cham (Switzerland): Springer Nature; c2020.
advanced and less effective surgical interventions.
Rabinstein AA, Lanzino G, Wijdicks EF. Multidisciplinary manage-
IV.27. Answer d.
ment and emerging therapeutic strategies in aneurysmal sub-
Syringomyelia can develop proximally in the spinal cord in
Copyright © 2021. Oxford University Press, Incorporated. All rights reserved.

arachnoid haemorrhage. Lancet Neurol. 2010 May;9(5):504–​19.


up to 30% of patients after a major spinal cord injury and
Wijdicks EF, Hijdra A, Young GB, Bassetti CL, Wiebe S; Quality
can cause further neurologic deficits. It can cause unilateral
Standards Subcommittee of the American Academy of
or bilateral symptoms, but bilateral symptoms are more spe-
Neurology. Practice parameter: prediction of outcome in coma-
cific for a spinal cord lesion.
tose survivors after cardiopulmonary resuscitation (an evi-
IV.28. Answer b.
dence-​ based review): report of the Quality Standards
IVIG and PLEX are accepted treatment choices for patients
Subcommittee of the American Academy of Neurology.
who cannot walk unassisted. Both treatments are consid-
Neurology. 2006 Jul 25;67(2):203–​10.
ered efficacious, but the combination of the 2 treatments is
Wijdicks EFM. Brain death. 3rd ed. New York (NY): Oxford
not known to be beneficial.
University Press; c2017. 284 p.
IV.29. Answer d.
Wijdicks EFM, Clark SL. Neurocritical care pharmacotherapy: a
For a patient with suspected neuromuscular respiratory fail-
clinician’s manual. 1st ed. New York (NY): Oxford University
ure, the airway, breathing, and circulation should take prior-
Press; c2018. 340 p.
ity to diagnostic imaging and testing. The 20-​30-​40 rule is a
Wijdicks EFM, Findlay JY, Freeman WD, Sen A, editors. Mayo
useful predictor of respiratory failure in Guillain-​Barré syn-
Clinic critical and neurocritical care board review. 1st ed. New
drome. A patient is likely to require endotracheal intubation
York (NY): Oxford University Press; c2019. 1062 p. (Mayo
if vital capacity is <20 mL/​kg; maximal inspiratory pressure,
Clinic Scientific Press series.)
<–​30 cm H2O; and maximal expiratory pressure, <40 cm H2O.
Wijdicks EFM, Rabinstein AA, Hocker SE, Fugate JE. Neurocritical
IV.30. Answer c.
care. 2nd ed. New York (NY): Oxford University Press; c2016.
Table 62.4 lists some clinical and examination findings that
321 p. (What do I do now series.)
may help distinguish cholinergic crisis from myasthenic crisis.

Mayo Clinic Neurology Board Review, edited by Kelly D. Flemming, Oxford University Press, Incorporated, 2021. ProQuest Ebook Central, http://ebookcentral.proquest.com/lib/cuhk-ebooks/detail.action?docID=6746419.
Created from cuhk-ebooks on 2023-09-02 09:10:26.
Copyright © 2021. Oxford University Press, Incorporated. All rights reserved.

Mayo Clinic Neurology Board Review, edited by Kelly D. Flemming, Oxford University Press, Incorporated, 2021. ProQuest Ebook Central, http://ebookcentral.proquest.com/lib/cuhk-ebooks/detail.action?docID=6746419.
Created from cuhk-ebooks on 2023-09-02 09:10:26.

You might also like